Download as pdf or txt
Download as pdf or txt
You are on page 1of 144

1

One of three blood culture bottles drawn from a patient with unexplained fevers reveals gram-positive cocci
growing in clusters. Which of the following tests would be most useful in determining whether this organism is a
part of the normal skin flora?

A. Bacitracin resistance

B. Catalase

C. Coagulase

D. Novobiocin resistance

E. Optochin resistance

Explanation:

The correct answer is C. Gram-positive cocci in clusters are staphylococci. Staphylococcus aureus is a common
pathogen that should not be considered normal skin flora. Other staphylococci, especially Staphylococcus
epidermidis, may contaminate blood cultures and can be differentiated from Staphylococcus aureus by the
coagulase test. S. aureus is the only coagulase-positive staphylococcus.

Bacitracin sensitivity differentiates Streptococcus pyogenes from the other beta-hemolytic streptococci, which
are bacitracin resistant (choice A).

The catalase test (choice B) is used to differentiate streptococci from staphylococci. Staphylococci produce
catalase and can generate oxygen bubbles in hydrogen peroxide, whereas streptococci cannot.

Novobiocin resistance (choice D) differentiates the coagulase-negative staphylococci into S. epidermidis


(novobiocin sensitive) and S. saprophyticus (novobiocin resistant).

Optochin resistance (choice E) differentiates the major pathogenic alpha-hemolytic streptococci. S. pneumoniae
is optochin- and bile-sensitive whereas S. viridans is resistant to both optochin and bile.

A 4-month-old infant presents with failure to thrive, progressive muscular weakness, and poor head control. On
questioning, the mother states that she typically feeds the baby soy-based formula sweetened with honey. Which
of the following organisms is most likely to be responsible for the child's presentation?

A. Clostridium botulinum

B. Clostridium difficile

C. Clostridium perfringens

D. Clostridium tetani

E. Corynebacterium diphtheriae

Explanation:

The correct answer is A. The baby has infant botulism (floppy baby syndrome), which is due to germination of
Clostridium botulinum spores (found in honey) in the baby's gastrointestinal tract. Patients improve when honey
is removed from the diet. This disorder is most common in children under the age of six months; older children
and adults do not appear to be vulnerable to this form of botulism, but are susceptible to botulism caused by
ingestion of preformed toxin.

Clostridium difficile(choice B) causes pseudomembranous colitis, especially after antibiotic therapy.


2

Clostridium perfringens(choice C) causes gas gangrene and gastroenteritis, and it is not associated with
ingestion of honey.

Clostridium tetani (choice D) causes tetanus, and does not cause a food-borne illness in infants.

Corynebacterium diphtheriae(choice E) causes diphtheria in susceptible individuals.

A 35-year-old woman presents to her gynecologist with complaints of burning on urination for the past 2 days.
Dipstick test of her urine demonstrates marked positivity for leukocyte esterase, but no reactivity for nitrite. Urine
culture later grows out large numbers of organisms. Which of the following bacteria are most likely to be
responsible for this patient's infection?

A. Enterobacter sp.

B. Enterococcus faecalis

C. Escherichia coli

D. Klebsiella pneumoniae

E. Pseudomonas aeruginosa

Explanation:

The correct answer is B. The positive leukocyte esterase test indicates the presence of neutrophils in the urine,
suggesting a bacterial infection. The nitrite test exploits the fact that most Enterobacteria (gram-negative enteric
rods) are able to form nitrite from nitrate; thus, the nitrite test is used to diagnose urinary tract infections. One
limitation of this method is the fact that enterococci (gut streptococci) do not produce nitrite from nitrate, but can
nonetheless cause urinary tract infections. Enterococcal urinary tract infections are often nosocomial and
classically acquired in the intensive care unit, although they can occur in other settings.

Enterobacter sp. (choice A), Escherichia coli(choice C), Klebsiella pneumoniae(choice D), and Pseudomonas
aeruginosa(choice E) can cause urinary tract infections and would usually be picked up by the dipstick for
nitrites. False-negative results might still be seen with these organisms if the infection was light, the bladder had
been recently emptied prior to collection, and the urine was "new" and had not yet grown enough bacteria to
produce a positive result.

A 37-year-old woman presents with 3 days of progressive joint pain in her ankles, knees, and wrists. She recalls
three similar episodes over the past several years. On examination, she has a temperature of 38.7 C, her blood
pressure is 110/70 mm Hg, and her heart rate is 90/min. She has a diffuse petechial rash over her trunk and
extensor surfaces. Her ankles and knees are swollen, red, and tender with decreased range of motion, and there
is tenderness over the tendon sheaths of her hands and forearms. Blood cultures are negative. Aspiration of joint
fluid reveals a white cell count of 22,000/mm3 with no visible organisms, but culture on chocolate agar is positive.
Which of the following is an attribute of the causative organism that allows it to produce recurrent infections?

A. It is an intracellular pathogen

B. It is resistant to ceftriaxone

C. It is resistant to complement-mediated lysis

D. Its capsule is not immunogenic


3
E. Its pili undergo antigenic and phase variation

Explanation:

The correct answer is E. This is a case of Neisseria gonorrhoeae arthritis. Patients are continuously susceptible
to reinfection because of antigenic variation and phase variation of the pili.

N. gonorrhoeae is not an intracellular pathogen (choice A), although it may be found intracellularly in
neutrophils after it has been phagocytized.

Ceftriaxone (choice B) is the drug of choice for N. gonorrhoeae.

Gonococci are especially susceptible to complement-mediated lysis, not resistant to it (choice C).

N. gonorrhoeae has an insignificant capsule that does not play a major role in the pathogenesis, but its capsule
is immunogenic (compare with choice D). Streptococcus pyogenes is the best known example of a
nonimmunogenic capsule, made of hyaluronic acid, but virtually all other capsules are immunogenic.

A Pap smear from a woman with chronic cervicitis shows cytoplasmic inclusions within epithelial cells. Fluorescent
antibodies identify both these inclusions and "elementary bodies." The intracellular organisms causing the
infection are unusual because they cannot synthesize which of the following?

A. ATP

B. Cholesterol

C. DNA

D. Proteins

E. RNA

Explanation:

The correct answer is A. The disease is chlamydial cervicitis. This venereally transmitted infection is usually
suspected after treatment for gonorrhea fails to relieve symptoms. However, a few cases are picked up when
cytoplasmic inclusions composed of aggregates of chlamydia are identified on Pap smears. Confirmation can
be made with fluorescent antibodies that pick up both the aggregates and individual bacteria known as
"elementary bodies." The organisms are obligate intracellular parasites because they have lost the ability to
synthesize ATP. Although some authors consider them to be "bacteria," others do not, since this is such a
fundamental difference between the chlamydia and free living bacteria.

Failure to synthesize cholesterol (choice B) is not usually cited as a problem specific to any type of organism,
although viruses are not able to do so without using host machinery.

Some viruses cannot synthesize DNA (choice C), but the disease in question is not caused by a virus.

Protein synthesis (choice D) by viruses requires host ribosomes.

Some viruses cannot synthesize RNA (choice E), but the disease in question is not caused by a virus.

A patient with colorectal cancer develops septicemia complicated by endocarditis. You would expect the blood
cultures to grow
4

A. Streptococcus agalactiae

B. Streptococcus bovis

C. Streptococcus pneumoniae

D. Streptococcus pyogenes

E. Streptococcus viridans

Explanation:

The correct answer is B.Streptococcus bovis is a Group D streptococcus. There is a significant association
between S. bovis bacteremia and endocarditis with carcinoma of the colon and other colonic diseases. Every
patient with S. bovis bacteremia should undergo gastrointestinal and cardiac evaluation. Up to 50% of patients
with S. bovis bacteremia are reported to have underlying colonic malignancies. In another study, 25-50% of
cases of S. bovis bacteremia were associated with endocarditis, especially in patients with preexisting valvular
lesions.

Streptococcus agalactiae(choice A) is an important cause of maternal and neonatal bacteremia and neonatal
meningitis. It is part of the normal flora of the gastrointestinal tract and the female genital tract.

Streptococcus pneumoniae(choice C) is a leading cause of community-acquired pneumonia, meningitis in


adults, otitis media (especially in children), and sinusitis. Spontaneous peritonitis due to S. pneumoniae is
reported in children with ascites from nephrotic syndrome. Asplenia predisposes patients to severe infections
with S. pneumoniae and other encapsulated organisms. S. pneumoniae infections are also more frequent and
unusually severe in patients with sickle cell anemia, multiple myeloma, alcoholism, and hypogammaglobulinemia.
S. pneumoniae is now the leading cause of invasive bacterial respiratory disease in patients with AIDS.

Streptococcus pyogenes(choice D) is the most common cause of bacterial pharyngitis. Complications include
paratonsillar abscesses, otitis media, and sinusitis. Long-term sequelae include rheumatic fever and
poststreptococcal glomerulonephritis. S. pyogenes is also responsible for many skin and soft tissue infections.
The organism also produces many toxins that produce a variety of diseases.

Streptococcus viridans(choice E), or the viridans Streptococci, are the most common cause of subacute
bacterial endocarditis, which should be suspected in cases of viridans streptococcal bacteremia. One species of
viridans Streptococci, Streptococcus milleri, is frequently associated with pyogenic abscesses, especially of the
liver.

A 54-year-old woman suffering from influenza deteriorates and develops shaking chills and a high fever. Physical
examination is remarkable for dullness to percussion at the left base and decreased breath sounds on the left.
Chest x-ray confirms the diagnosis of lobar pneumonia, presumed to be caused by Streptococcus pneumoniae.
The patient has no known drug allergies. Which of the following antibiotics would be most appropriate to treat the
patient's condition?

A. Cefotaxime

B. Chloramphenicol

C. Erythromycin

D. Penicillin

E. Vancomycin
5

Explanation:

The correct answer is D. Penicillin remains the first-line drug of choice for pneumococcal pneumonia, except in
patients with penicillin allergy and in the relatively few areas in which pneumococcal strains with high-level
penicillin resistance exist. Alternative therapies include erythromycin and vancomycin.

The third-generation cephalosporin cefotaxime (choice A) is not usually used for pneumococcal pneumonia.

Chloramphenicol (choice B) is not usually used for pneumococcal pneumonia.

Erythromycin (choice C) is a good alternative therapy for pneumococcal pneumonia, but is usually used only
when a penicillin allergy is present.

Vancomycin (choice E) is not the first-line therapy, but it is a good alternative in patients allergic to penicillin or
when high-level penicillin resistance (relatively uncommon) is present.

An otherwise healthy 3-year-old child is brought to the pediatrician with umbilicated, flesh-colored papules on his
trunk. This condition is related to infection with which of the following viruses?

A. Cytomegalovirus

B. Herpesvirus 6

C. Parvovirus

D. Poxvirus

E. Variola

Explanation:

The correct answer is D. The lesions are characteristic of molluscum contagiosum, which is a typically benign
and self-limited condition caused by a poxvirus. The disease can be transmitted either venereally or through
non-venereal contact. The other viruses listed do not cause similar skin lesions. Patients with advanced HIV
infection may develop a severe, generalized, and persistent eruption, often involving the face and upper body.

Cytomegalovirus (choice A) causes congenital infections and disseminated infections in immunosuppressed


patients.

Herpesvirus 6 (choice B)causes roseola (exanthem subitum).

Parvovirus (choice C) causes aplastic crises in patients with hemolytic anemia.

Variola (choice E) is the smallpox virus.

A 39-year-old black man presents with complaints of anorexia, malaise, fatigue, dark urine, and upper abdominal
discomfort. He admits to homosexuality, but denies blood transfusions, alcohol intake, or intravenous drug
abuse. On physical examination, the patient has a temperature of 100.2 degrees F, scleral icterus, and jaundice.
His liver is palpable below the right costal margin, and there is moderate right upper quadrant tenderness. Liver
function test results are as follows: total bilirubin 12.4%, SGOT 980 units, SGPT 1200 units. Serologic findings
are as follows: anti-hepatitis A IgM negative, HBsAg positive, anti-HBc IgM positive, HBeAg positive, anti-HBsAb
negative, hepatitis C negative. When this patient enters the window period, what would likely be the first change
in his serologic findings?
6

A. He will become HBcAg positive

B. He will become HBc IgG positive

C. He will become HBeAg negative

D. He will become HBsAb positive

E. He will become HBsAg negative

Explanation:

The correct answer is E. This patient has acute hepatitis B. The "window period" refers to that period in
infection when neither hepatitis B surface antigen (HBsAg) nor its antibody (HBsAb) can be detected in the
serum of the patient. It is an immunologically mediated phenomenon caused by the precipitation of
antigen-antibody complexes in their zone of equivalent concentrations and, thereby, their removal from the
circulation. Because of this, the first thing that will happen in the window period is that the serum will become
negative for the surface antigen (HBsAg), as that antigen is precipitated out of the serum by developing levels
of its specific antibody (HBsAb).

HBcAg is not typically measured (choice A) in the serum.

Levels of the c-core antibody HBc IgG (choice B) and HBeAg (choice C) do not have a relationship to the
window period.

Levels of HBsAb (choice D) will not be detectable until there is antibody excess, and the patient is leaving the
window period.

A 15-year-old girl in a rural community has swollen, painful lymph nodes in her right axilla. Physical examination
reveals multiple scratches on her right arm with a papule associated with one of the scratch marks. She states
that the scratches occurred about 5 days ago. What type of animal is the most likely source of the infection?

A. Cat or kitten

B. Chicken

C. Dog or puppy

D. Horse

E. Parrot

Explanation:

The correct answer is A. This patient has the classic symptoms of cat scratch disease caused by the bacillus
Bartonella henselae. The disease is self-limited with the onset of symptoms occurring 3-10 days following an
inoculating scratch. The organism can be isolated from kittens, typically less than 1 year of age, or from fleas. A
history of a new kitten in the house and the papule at the site of a scratch with regional painful adenopathy
defines the classic scenario.

Chickens (choice B) can harbor Salmonella spp. producing a gastroenteritis or enterocolitis. Chicken guano is
also a favorable environment for the fungus Histoplasma capsulatum. The mycelial phase thrives in the rich soil.
The human disease is a granulomatous infection involving the lungs and mimicking tuberculosis.

Dogs or puppies (choice C) carry Capnocytophaga canimorsus as part of the normal flora of the oral cavity.
7
Infections from licking or biting range from a self- limited cellulitis to fatal septicemia. Patients at risk for more
severe infections are those with asplenia, alcoholism, or hematologic malignancies. This organism is also
associated with cat bites, but the patient develops cellulitis and fulminant septicemia, especially in asplenic
patients. Pasteurella multocida is another pathogen that colonizes the nasopharynx and gastrointestinal tract of
cats and dogs. Cats have the highest rate of colonization (50-90%), followed by dogs (50%), swine (50%), and
rats (14%). P. multocida most commonly causes a localized soft tissue infection or cellulitis after an animal bite,
but systemic symptoms may be present in about 40% of the cases. These symptoms include osteomyelitis,
septic arthritis, or tenosynovitis.

Horses (choice D) and horse manure have been associated with a pulmonary opportunistic infection with
cavitation caused by Rhodococcus equi that resembles tuberculosis in immunocompromised patients.
Burkholderia mallei (the cause of glanders) is characterized by non-caseating granulomatous abscesses of
skin, lymphadenopathy, and pronounced involvement of the lungs.

Parrots (choice E) are associated with psittacosis caused by Chlamydia psittaci. Psittacosis is associated with a
dry, hacking cough productive of scant sputum, an interstitial infiltrate in the lungs, severe headache, and
myalgias. A pale macular rash is also seen.

A 73-year-old woman with a history of diabetes presents with left ear pain and drainage of pus from the ear
canal. She has swelling and tenderness over the left mastoid bone. Which of the following microorganisms is the
most likely causative agent?

A. Hemophilus influenzae

B. Klebsiella pneumoniae

C. Mucor sp.

D. Pseudomonas aeruginosa

E. Streptococcus pyogenes

Explanation:

The correct answer is D.Pseudomonas aeruginosa causes malignant otitis externa, which is a severe
necrotizing infection of the external ear canal. Infection tends to spread to the mastoid bone, temporal bone,
sigmoid sinus, base of the skull, meninges, and brain. Patients at increased risk include the elderly, those with
diabetes, and the immunocompromised. Pseudomonas is also associated with many other clinical syndromes,
including infection following traumatic wounds to the feet in persons wearing sneakers or rubber-soled shoes,
since the organism can be cultured from the shoes.

Hemophilus influenzae(choice A) produces a variety of clinical syndromes. H.Influenzae is the third-most


common cause of meningitis in children aged 1 month to 18 years. It is the most common cause of acute
epiglottitis, the most common cause of purulent bacterial conjunctivitis, and the second-most common cause of
otitis media. Patients at risk include those with COPD and cystic fibrosis, alcoholics, splenectomized patients,
and young patients.

Klebsiella pneumoniae(choice B) is a gram-negative organism that produces a necrotizing pneumonia in


diabetics and alcoholics. Patients typically present with an abrupt onset of fever, shaking chills, and purulent,
foul-smelling sputum.

Mucor (choice C) is a fungal infection that is particularly severe in the diabetic or the immunocompromised
patient. In the acidotic diabetic, the fungus produces a life-threatening, invasive rhinocerebral infection. The
infection begins in the nasal passages, extends into the paranasal sinuses, and spreads through the cribiform
plate to the frontal lobes of the brain. Patients typically complain of headache, facial pain, and orbital swelling.

Streptococcus pyogenes(choice E) causes bacterial pharyngitis, otitis media, and sinusitis. It is also associated
8
with toxin-related diseases and skin infections.

A 29-year-old female diagnosed with AIDS has been suffering from a progressive blurring of vision in her right
eye. On funduscopic examination, a small white opaque lesion is noted on the retina of her right eye. Which of
the following is the most appropriate therapy for this patient?

A. Acyclovir

B. Amantadine

C. Flucytosine

D. Ganciclovir

E. Zidovudine

Explanation:

The correct choice is D. This patient is most likely suffering from cytomegalovirus (CMV) retinitis. The best drug
treatment for this infection is ganciclovir.

Acyclovir (choice A) is not effective in CMV infections. It is used more for HSV type 1 and 2 infections.

Amantadine (choice B) is used either therapeutically or prophylactically for the influenza A virus.

Flucytosine (choice C) is an antifungal agent.

Zidovudine (choice E) is a first-line drug for the treatment of AIDS. The drug by itself is ineffective against CMV
retinitis.

A newborn is infected in utero with an enveloped virus containing double-stranded DNA. The child develops
petechiae, hepatosplenomegaly, and jaundice. Brain calcifications are detected on computed tomography (CT).
With which of the following viruses is the newborn most likely infected?

A. Cytomegalovirus

B. Hepatitis B

C. Hepatitis C

D. Hepatitis D

E. Herpes simplex

Explanation:

The correct answer is A. The most common agents causing congenital infection in the United States are
cytomegalovirus, herpes simplex virus, and Treponema pallidum. Congenital cytomegalovirus infection, which
occurs in 1 to 2% of all live births in the U.S., results from transplacental acquisition of a primary or recurrent
maternal infection. Most infections are asymptomatic, but about 5% will present with hepatosplenomegaly,
periventricular brain calcification, petechial hemorrhages, and hydrops. Sensorineural deafness is a common
manifestation in affected infants. Primary infection of the mother is strongly associated with more severe
symptoms in affected infants. The mortality rate is 30% and most of the survivors will have neurologic
impairment. CMV infections in children and adults are often asymptomatic; however, the virus is responsible for
9
10% of infectious mononucleosis cases (characterized by an absence of sheep RBC agglutinins). CMV also
causes serious disease in immunosuppressed patients, including bone marrow recipients and AIDS patients.
Ganciclovir is the drug of choice, although foscarnet has also been used with some success.

Infections with hepatitis viruses (choices B, C, and D), if symptomatic, are characterized by jaundice, lethargy,
failure to thrive, abdominal distention, clay- colored stools, and elevated transaminase levels. Hepatitis B(choice
B) is the only viral hepatitis agent that is recognized as an important cause of neonatal disease. The infection is
usually acquired during the birth process; therapy includes hyperimmune human anti-hepatitis B serum followed
a few weeks later with active immunization with rHBsAg injections.

Herpes simplex (choice E) can also infect the neonate. The hallmark of infection is painful skin vesicles (present
in 50% of affected infants); other manifestations include encephalitis, pneumonia, hepatitis, and disseminated
intravascular coagulopathy. Massive hepatic and adrenal necrosis is common. Therapy is with acyclovir.

A 60-year-old alcoholic smoker abruptly develops high fever, shakes, a severe headache, and muscle pain. He
initially has a dry, insignificant cough, but over the next few days he develops marked shortness of breath
requiring assisted ventilation. Chest x-ray demonstrates homogeneous radiographic shadowing that initially
involves the left lower lobe but continues to spread until both lungs are extensively involved. Culture of
bronchoalveolar lavage fluid on buffered charcoal yeast extract (BCYE) demonstrates a coccobacillary pathogen.
Which of the following is the most likely causative organism?

A. Legionella pneumophila

B. Listeria monocytogenes

C. Spirillium minus

D. Staphylococcus aureus

E. Streptococcus pneumoniae

Explanation:

The correct answer is A. The patient has a severe, potentially fatal, pneumonia with prominent systemic
symptoms. Culture on BCYE is the specific clue that the organism is Legionella pneumophila. The disease is
respiratory Legionellosis, also known as Legionnaire's disease, because the disease was first described when it
occurred in epidemic form following an American Legion convention at a Philadelphia hotel. Patients tend to be
older (40-70 years old) and may have risk factors including cigarette use, alcoholism, diabetes, chronic illness,
or immunosuppressive therapy.

Listeria monocytogenes(choice B) causes listeriosis and is not a notable cause of pneumonia.

Spirillium minus (choice C) is a cause of rat-bite fever and is not a notable cause of pneumonia.

Staphylococcus aureus(choice D) can cause pneumonia, but is easily cultured on routine media.

Streptococcus pneumoniae(choice E) can cause pneumonia, but is easily cultured on routine media.

A 33-year-old woman presents with fever, vomiting, severe irritative voiding symptoms, and pronounced
costovertebral angle tenderness. Laboratory evaluation reveals leukocytosis with a left shift; blood cultures
indicate bacteremia. Urinalysis shows pyuria, mild hematuria, and gram-negative bacteria. Which of the following
drugs would best treat this patient's infection?
10
A. Ampicillin and gentamicin

B. Erythromycin

C. Gentamicin and vancomycin

D. Phenazopyridine and nitrofurantoin

E. Tetracycline

Explanation:

The correct answer is A. Acute pyelonephritis is an infectious disease involving the kidney parenchyma and the
renal pelvis. Gram-negative bacteria, such as Escherichia coli, Proteus, Klebsiella, and Enterobacter, are the
most common causative organisms in acute pyelonephritis. Laboratory evaluation will often reveal leukocytosis
with a left shift, and urinalysis typically shows pyuria, varying degrees of hematuria, and white cell casts. Since
bacteremia is present, the patient should be hospitalized and empirically started on IV ampicillin and gentamicin.
This regimen may be need to be changed, however, once the sensitivity results are available.

Erythromycin (choice B) and tetracycline (choice E) are both bacteriostatic antibiotics and would not be
recommended in a patient with a severe infection, such as acute pyelonephritis with bacteremia.

Vancomycin (choice C) is primarily used in the treatment of severe gram-positive infections.

Phenazopyridine (choice D) is a urinary analgesic, and nitrofurantoin (choice D) is a urinary tract anti-infective.
Although nitrofurantoin is indicated for the treatment of "mild" cases of pyelonephritis, as well as cystitis, this
patient's condition is severe and should be treated with appropriate antibiotics.

A newborn infant has multiple, hemorrhagic, cutaneous lesions and does not respond to sound. Head CT scan
shows periventricular calcifications. Which of the following infectious agents is the most likely cause of this child's
presentation?

A. Cytomegalovirus

B. Herpes simplex

C. Rubella

D. Syphilis

E. Toxoplasmosis

Explanation:

The correct answer is A. The infectious agents listed are all important causes of congenital disease. The triad of
cutaneous hemorrhages ("blueberry muffin baby"), deafness, and periventricular CNS calcifications suggests
congenital CMV infection, the most common cause of intrauterine fetal viral infection. Other manifestations
include microcephaly and hepatosplenomegaly.

Neonatal herpes (choice B) may be congenital, but more commonly is acquired during vaginal delivery. The
infection is characterized by vesicles on the skin and mucous membranes, encephalitis, or disseminated
disease.

Congenital rubella (choice C) can cause mental retardation, heart abnormalities, blindness, encephalitis, and
motor abnormalities.
11
Congenital syphilis (choice D) can cause death in utero, or a variety of problems including abnormal teeth,
bones, and central nervous system.

Toxoplasmosis (choice E) can be either acquired during delivery (mild) or congenital (severe). Severe infections
can cause stillbirth, chorioretinitis, intracerebral calcifications, and hydro- or microcephaly.

A very ill neonate has widespread granulomas. In utero infection with which of the following organisms is
suggested by this finding?

A. Clostridium botulinum

B. Escherichia coli

C. Haemophilus influenzae

D. Listeria monocytogenes

E. Neisseria gonorrhoeae

Explanation:

The correct answer is D. All of the organisms listed can cause infection in infants, but Listeria monocytogenes is
the one to cause potentially fatal granulomas (granulomatis infantiseptica) following in utero infection.

Clostridium botulinum(choice A) can cause infant botulism, characterized by failure to thrive, and progressive
muscular weakness.

Escherichia coli(choice B) can cause diarrhea, pneumonia, and meningitis in infants.

Haemophilus influenzae(choice C) can cause acute epiglottitis, meningitis, pneumonia, and otitis media in
young children.

Neisseria gonorrhoeae(choice E) can be transmitted via the birth canal, and causes ophthalmia neonatorum.

A 37-year-old female presents to the emergency room with a fever. Chest x-ray shows multiple patchy infiltrates
in both lungs. Echocardiography and blood cultures suggest a diagnosis of acute bacterial endocarditis limited to
the tricuspid valve. Which of the following is the most probable etiology?

A. Congenital heart disease

B. Illicit drug use

C. Rheumatic fever

D. Rheumatoid arthritis

E. Systemic lupus erythematosus

Explanation:

The correct answer is B. The most probable etiology of bacterial endocarditis involving the tricuspid valve is
illicit intravenous drug use, which can introduce skin organisms into the venous system that then attack the
tricuspid valve. Staphylococcus aureus accounts for between 60% and 90% of cases of endocarditis in
intravenous drug users.
12

The endocarditis associated with congenital heart disease (choice A) typically involves either damaged valves
or atrial or ventricular septal defects. The tricuspid valve is not particularly vulnerable.

Rheumatic fever (choice C) most commonly damages the mitral and aortic valves, and tricuspid damage is
usually less severe and seen only when the mitral and aortic valves are heavily involved. Consequently,
secondary bacterial endocarditis involving only the tricuspid valve in a patient with a history of rheumatic fever
would be unusual.

Rheumatoid arthritis (choice D) is not associated with bacterial endocarditis.

Systemic lupus erythematosus (choice E) can produce small, aseptic vegetations on valves, but is not
associated with bacterial endocarditis.

A 4-year-old boy is brought to the emergency room in extreme respiratory distress, with a temperature of 103.8
degrees Fahrenheit. He is drooling and has difficulty swallowing, and on physical examination, inspiratory stridor
is noted. A lateral x-ray shows swelling of the epiglottis. He has had no previous vaccinations. Which of the
following agents is the most likely cause of these symptoms?

A. Haemophilus influenzae

B. Klebsiella pneumoniae

C. Legionella pneumophila

D. Mycoplasma pneumoniae

E. Streptococcus pyogenes

Explanation:

The correct answer is A. Epiglottitis is the most common disease of the upper respiratory tract produced by
Haemophilus influenzae type b, a gram-negative encapsulated rod. It is also a common cause of otitis media in
children and may cause bronchitis, bronchiolitis, and pneumonia in adults. The incidence of serious disease
caused by Haemophilus influenzae type b has decreased greatly with the introduction of an effective vaccine.
The vaccine is composed of the H. influenzae type b capsular polysaccharides coupled to a carrier molecule,
given to children between 2 and 15 months of age. The patient had not received the Hib conjugate vaccine and
therefore was susceptible to this organism.

Klebsiella pneumoniae(choice B) causes pneumonia and pulmonary abscesses, but is not considered to be a
pathogen in the upper respiratory tract.

Legionella pneumophila(choice C) causes pneumonia in man. The disease may be mild (an atypical
pneumonia) or a fulminating disease with a high mortality (30%).

Mycoplasma pneumoniae(choice D) causes community-acquired atypical pneumonia. It is the most common


cause of pneumonia in young adults.

Streptococcus pyogenes(choice E) is the most common cause of pharyngitis; however, this patient's
presentation strongly suggests epiglottitis.

A 15-year-old boy presents to his physician with several weeks of slowly worsening pruritus of both of his feet. He
is otherwise well and taking no medications. On examination, he has bilateral, erythematous, dry scaling lesions
that are most obvious in the interdigital web spaces and on the soles. There is no bleeding or exudate. What
13
would most likely be found in a potassium hydroxide (KOH) mount of a scraping of the affected skin?

A. Branching hyphae with rosettes of conidia

B. Branching, septate hyphae

C. Budding yeasts

D. Hyphae, arthroconidia, and blastoconidia

E. Pigmented, septate hyphal fragments

F. Short, curved hyphae and round yeasts

Explanation:

The correct answer is B. This is a case of tinea pedis, or athlete's foot, caused by a variety of dermatophytic
fungi, which are easily detected on alkali mounts of scraped skin as colorless, branching hyphae with
cross-walls.

Hyphae with rosettes of conidia (choice A) describes the environmental/transmission form for Sporothrix
schenckii, the agent of rose gardener's disease, which is a subcutaneous mycosis.

Budding yeasts (choice C) describes the form found in clinical specimens from patients with sporotrichosis.

Hyphae, arthroconidia, and blastoconidia (choice D) would be found in clinical specimens from patients infected
with Trichosporon beigelii (white piedra), which is a superficial mycosis of the hair of the head.

Pigmented, septate hyphal fragments (choice E) would be found in cases of phaeohyphomycosis, a diverse
group of cyst-forming subcutaneous, pigmented (dematiaceous) fungi, rare in the U.S.

Short, curved hyphae and round yeasts (choice F), or the "spaghetti and meatball" presentation in clinical
specimens, are characteristic of Malassezia furfur, the agent of pityriasis versicolor.

A 24-year-old AIDS patient develops chronic abdominal pain, low-grade fever, diarrhea, and malabsorption.
Oocysts are demonstrated in the stool. Which of the following organisms is most likely to be the cause of the
patient's diarrhea?

A. Diphyllobothrium latum

B. Entamoeba histolytica

C. Giardia lamblia

D. Isospora belli

E. Microsporidia

Explanation:

The correct answer is D. All of the organisms listed are protozoa. There are two intestinal protozoa specifically
associated with AIDS that can cause transient diarrhea in immunocompetent individuals but can cause
debilitating, and potentially life-threatening chronic diarrhea in AIDS patients. These organisms are Isospora
belli, treated with trimethoprim-sulfamethoxazole (or other folate antagonists) and Cryptosporidium parvum (no
treatment presently available).
14

Diphyllobothrium latum(choice A) is the fish tapeworm and occasionally causes diarrhea.

Entamoeba histolytica(choice B) and Giardia lamblia(choice C) are both causes of diarrhea, but they are not
specifically associated with AIDS.

Microsporidia (choice E) are a protozoan cause of diarrhea, but produce spores rather than oocysts.

An African child develops massive unilateral enlargement of his lower face in the vicinity of the mandible. Biopsy
demonstrates sheets of medium-sized blast cells with admixed larger macrophages. This type of tumor has been
associated with which of the following?

A. Epstein-Barr virus and t(8;14)

B. Hepatitis B and t(9;22)

C. Herpesvirus and CD5

D. Human immunodeficiency virus and CD4

E. Human papillomavirus and t(2;5)

Explanation:

The correct answer is A. The patient has Burkitt's lymphoma. This type of lymphoma is a high-grade B-cell
lymphoma that occurs in endemic form in Africa (it is the most common neoplasm in children in an equatorial
belt that includes Africa and New Guinea) and sporadically in the United States and Europe. The sporadic form
is often in an abdominal site and occurs in young adults. The African form of Burkitt's lymphoma has been
strongly associated with antibodies directed against Epstein-Barr virus; the association is weaker in sporadic
cases. A characteristic translocation, t(8;14) (q24.l3;q32.33) has been described.

Hepatitis B (choice B) is associated with hepatocellular carcinoma. t(9;22) is the Philadelphia chromosome,
which is seen in some cases of CML and AML.

Herpesvirus (choice C) does not have a strong tumor association, although a link to cervical cancer has
intermittently been proposed. CD5 is a marker seen in small lymphocytic and mantle cell lymphomas.

HIV (choice D) is linked to Kaposi's sarcoma (and AIDS). Some patients also develop primary lymphomas (not
usually Burkitt's). CD4 is a marker for helper T cells and some T cell lymphomas.

Human papillomavirus (choice E) is linked with common warts, genital condylomata, and genital cancers. t(2;5)
is linked to anaplastic large cell lymphoma.

A 49-year-old Vietnamese man is diagnosed with tuberculosis. On physical examination, large flocculent masses
are noted over the lateral lumbar back, and a similar mass is located in the ipsilateral groin. This pattern of
involvement strongly suggests an abscess tracking along the

A. adductor longus

B. gluteus maximus

C. gluteus minimus

D. piriformis
15

E. psoas major

Explanation:

The correct answer is E. This is the classic presentation of a psoas abscess. This clinical entity was formerly a
fairly common complication of vertebral tuberculosis, but is now rare in clinical practice in this country. The
psoas muscle is covered by a fibrous sheath known as the psoas fascia. This sheath is open superiorly,
permitting an infection involving the soft tissues around the spine to enter the sheath, then track down to the
groin.

The adductor longus (choice A) is a muscle of the anterior thigh, and is not related to the lumbar portion of the
back.

The gluteus maximus (choice B) gluteus minimus (choice C) and piriformis (choice D) are muscles of the
buttock with no relationship to the groin.

A 36-year-old man with AIDS develops right-sided weakness involving the lower, but not the upper, limb. MRI
scans reveal a ring-enhancing lesion within the white matter of the left frontal lobe. A biopsy shows coagulative
necrosis of brain parenchyma with macrophage-rich chronic inflammatory infiltration admixed with microscopic
cysts that contain characteristic bradyzoites. Which of the following is the most common source of this type of
infection?

A. Anopheles mosquitoes

B. Bird droppings

C. Cats

D. Cooling systems

E. Washbasins

Explanation:

The correct answer is C. Clues to the correct answer are the underlying disease (AIDS), which predisposes
to opportunistic infections, the typical MRI appearance of the lesion (ring-enhancing mass), and the
histopathologic features (presence of encysted bradyzoites). In short, this patient has cerebral toxoplasmosis,
which represents one of the most common opportunistic infections in AIDS. Toxoplasma gondii is a protozoon
that infects humans who ingest the oocysts from cat feces or incompletely cooked lamb or pork. Only
immunodepressed patients and fetuses are vulnerable to this infection. In the fetus, toxoplasmosis causes
extensive damage to brain parenchyma and retina. Toxoplasmosis associated with AIDS manifests with
necrotizing lesions surrounded by chronic inflammation. A ring-enhancing lesion is a mass that contains a rim of
contrast enhancement (bright signal on MRI) surrounding a dark core corresponding to central necrosis. In
AIDS, the most frequent causes of a ring-enhancing lesion are primary brain lymphoma and toxoplasmosis.

Anopheles mosquitoes (choice A) transmit malaria parasites. Cerebral malaria is caused by Plasmodium
falciparum, which is able to adhere to endothelial cells of small cerebral vessels. Vascular occlusion ensues,
resulting in numerous small infarcts.

Bird droppings (choice B) represent the vehicle of infection for two of the most common opportunistic fungal
infections affecting immunocompromised patients: cryptococcosis and histoplasmosis. Cryptococcus
neoformans causes meningoencephalitis, not intracerebral necrotic lesions. Histoplasma capsulatum rarely
affects the brain. These fungi can be visualized in tissue sections by silver stains.

Cooling systems (choice D) may harbor Legionella pneumophila, spreading the bacteria in aerosolized form. L.
16
pneumoniae is a gram-negative bacterium that causes Legionnaire's disease, a fatal form of pneumonia that
first struck participants at a meeting of the American Legion. It has been reported in immunocompromised
patients as well.

Washbasins (choice E) frequently contain Pseudomonas aeruginosa, which has also been isolated from
respirator devices, cribs, and antiseptic-containing bottles. P. aeruginosa tends to affect patients with cystic
fibrosis, severe burns, or immune impairment. It may cause bronchopneumonia, osteomyelitis, endocarditis,
external otitis, and keratitis, but not cerebral infection.

A 16-year-old girl presents with a painlessly enlarged lymph node in her right axilla. Peripheral blood counts are
within normal limits. The lymph node is biopsied, and numerous granulomas filled with neutrophils and necrotic
debris are observed. Which of the following organisms could produce this disease?

A. Bartonella henselae

B. Borrelia burgdorferi

C. Chlamydia psittaci

D. Coxiella burnetii

E. Rickettsia prowazekii

Explanation:

The correct answer is A.Bartonella henselae is the infective agent of cat scratch disease, which generally
presents as regional lymphadenopathy with or without low fevers and headaches. Bartonella is a gram-variable
pleomorphic rickettsial organism that is introduced to the skin in a cat bite or scratch. It produces a self-limited
granulomatous response in the draining lymph nodes.

Borrelia burgdorferi (choice B) is a spirochetal organism that is transmitted by a tick bite (Ixodes spp.),
producing Lyme disease. Lyme disease progresses from a skin rash to fevers, headache and pain over about
one month. It may produce lymphadenopathy, but is not associated with granuloma formation.

Chlamydia psittaci (choice C) infection occurs after contact with infected bird droppings and produces an
atypical pneumonia. The central nervous system may also be involved, but lymph nodes are spared. Chlamydia
trachomatis is the chlamydial species that typically produces suppurative nodal granulomas (lymphogranuloma
venereum).

Coxiella burnetii(choice D) infection is transmitted by inhaling dusts or drinking milk from infected mammals,
especially sheep and cows. The disease in humans, Q fever, is marked by mild nonspecific symptoms or
pneumonia, and may progress to myocarditis or hepatitis.

Rickettsia prowazekii(choice E) produces epidemic (louse-borne) typhus, which is transmitted by body lice and
produces a rash akin to Rocky Mountain spotted fever. Although the organism may reside in the lymph nodes in
dormancy, it does not elicit granuloma formation.

A 28-year-old male intravenous drug user presents with a febrile illness that has lasted 2 weeks. He also
complains of chills, weakness, dyspnea, cough, arthralgia, diarrhea, and abdominal pain. On examination, a heart
murmur is present, and small, tender nodules are found on the finger and toe pads, along with small
hemorrhages on the palms and soles. Three sets of blood cultures are obtained from different veins. While
awaiting laboratory confirmation, an empiric antibiotic regimen should primarily be directed at which of the
following organisms?

A. Enterococci
17

B. Pseudomonas aeruginosa

C. Staphylococcus aureus

D. Streptococcus pneumoniae

E. Streptococcus pyrogenes

Explanation:

The correct answer is C. This patient is presenting with signs and symptoms of acute infective endocarditis (IE).
The fact that the patient is an intravenous drug user places him at a very high risk for the development of IE.
The factors that determine the clinical presentation of IE include the nature of the organism, the valve infected,
and the route of the infection. Although the clinical characteristics of IE can vary, most patients present with a
febrile illness lasting several days to 2 weeks. This illness is often accompanied by a variety of nonspecific signs
and symptoms such as chills, weakness, dyspnea, cough, arthralgia, diarrhea, and abdominal pain. Heart
murmurs occur in approximately 90% of all patients, but may be absent in patients with right-sided infections.
Other clinical signs include Osler nodules (purplish or erythematous subcutaneous papules or nodules on the
pads of the fingers and toes), Janeway lesions (hemorrhagic painless plaques on the palms and soles),
petechiae (small erythematous painless hemorrhagic lesions that may appear anywhere), and splinter
hemorrhages (thin linear hemorrhages found under the nail beds of fingers and toes). The diagnosis of IE is
dependent on positive blood cultures and echocardiographic evidence of "valvular vegetation" and/or valvular
injury, with echocardiography the preferred method of diagnosis. A past medical history of intravenous drug
abuse contributes to the diagnosis. The initial treatment measures of IE should be aimed at the stabilization of
cardiac and/or respiratory symptoms, if applicable. The second priority is obtaining three blood cultures from
different veins. Once the blood cultures are obtained, empiric antibiotic therapy for Staphylococci, Streptococci,
and Enterococci should be instituted. Since Staphylococcus aureus accounts for most cases of acute IE,
empiric coverage should primarily be directed toward Staphylococcus aureus.

Enterococci(choice A) cause a minority of cases of native valve endocarditis, but are not usually involved in
endocarditis associated with intravenous drug abuse.

Gram-negative organisms such as Pseudomonas aeruginosa(choice B) are rarely the cause of IE.

Streptococcus pneumoniae(choice D) and Streptococcus pyogenes(choice E) are not common causes of IE.

A 20-year-old female presents with a two day history of dysuria and increased urinary frequency. She states that
she was recently married and was not sexually active prior to the marriage. Physical exam reveals a temperature
of 100.7 °F with normal vital signs. Gynecological exam reveals no evidence of discharge, vaginitis, or cervicitis.
Urinalysis reveals 14 white blood cells per high-powered field with many gram-negative rods. The most
appropriate therapy would be

A. ampicillin

B. ceftriaxone

C. fluconazole

D. gentamicin

E. metronidazole

Explanation:

The correct answer is A. The patient's presentation is consistent with a simple urinary tract infection; there is a
18
short history of dysuria, increased urinary frequency and the appearance of white blood cells and
gram-negative rods in the urine. Urinary tract infections are common in women after they become sexually
active. The infection is likely caused by urethral trauma during intercourse, which leads to bacterial
contamination of the bladder. Since the majority of these infections are caused by Escherichia coli (a
gram-negative rod), the most appropriate therapy would be ampicillin for around 10 days.

Ceftriaxone (choice B) is the treatment of choice for uncomplicated infections with N. gonorrhoeae, now that
most strains are resistant to penicillin. Intravenous ceftriaxone is a regimen reserved for the treatment of
life-threatening infections.

Fluconazole (choice C) is indicated for the treatment of vaginal candidiasis. Since there is no vaginal discharge
and the patient has gram-negative rods in the urine, a diagnosis of vaginal candidiasis can be excluded.

Gentamicin (choice D) would be an inappropriate choice since the majority of urinary tract infections caused by
gram negative rods are sensitive to ampicillin and the potential for toxicity secondary to gentamicin is great.

Metronidazole (choice E) is an antibiotic typically used in the treatment of Trichomoniasis, Giardiasis, and
Gardnerella, as well as serious infections believed to be caused by anaerobic bacteria. Since there is no
vaginal discharge and the patient has gram negative rods in the urine, one can conclude that these infections
are not present and the patient instead has a urinary tract infection.

A 36-year-old mother of two children presents with a 4-day history of swollen, painful hands. Her wrists and
metacarpophalangeal joints are boggy and inflamed bilaterally. Her 5-year-old son had been sent home from
school approximately 3 weeks previously with red cheeks and a blotchy rash on his torso. What is the most likely
diagnosis?

A. Listeriosis

B. Lyme disease

C. Mumps

D. Parvovirus

E. Reiter's syndrome

Explanation:

The correct answer is D. Parvovirus B19 causes erythema infectiosum, or Fifth disease. The 5-year-old boy has
the classic "slapped cheek" appearance. Adults typically do not get the facial rash, but have arthralgias and
arthritis. The symmetrical distribution of involved joints is similar to that in rheumatoid arthritis. The onset in
adults is typically 3 to 4 weeks after exposure. Parvovirus infections may persist in immunosuppressed patients,
resulting in red blood cell aplasia.

Listeriosis (choice A) is caused by the gram-positive rod Listeria monocytogenes. Meningitis and bacteremia
are common clinical manifestations. Elderly, neonates, pregnant women, and those taking steroids have the
highest risk for infection.

Lyme disease (choice B) is caused by Borrelia burgdorferi. It is the most common vector-borne disease (Ixodes
ticks) in the U.S. The incidence is highest in the summer and fall. The white-footed mouse and the white-tailed
deer are zoonotic reservoirs. From 3 to 32 days following a tick bite, the patient develops fever,
lymphadenopathy, meningismus, and the characteristic rash (erythema migrans). The rash enlarges and
resolves over 3 to 4 weeks. Sequelae include arthritis, carditis, and neurologic abnormalities.

Mumps (choice C) is caused by a Paramyxovirus. The virus most commonly affects glandular tissue. Parotitis,
pancreatitis, and orchitis are characteristic. Mumps meningoencephalitis is one of the most common viral
19
meningitides. Mumps polyarthritis is most common in men between the ages of 20 and 30 years. Joint symptoms
begin 1 to 2 weeks after the parotitis subsides and large joints are involved.

Reiter's syndrome (choice E) is a seronegative, asymmetric arthropathy predominantly affecting the lower
extremities. It may be triggered by a C. trachomatis infection. In addition to the arthritis, patients may have
urethritis (which is usually due to chlamydia), conjunctivitis, mucocutaneous disease such as balanitis, oral
ulcerations, or keratoderma. Approximately 80% of patients are HLA-B27 positive.

A 28-year-old pregnant woman develops a flu-like illness with fever, headache, myalgia, and back pain. As a
complication of the illness, she has a spontaneous abortion. Examination of the abortus demonstrates severe
amnionitis. Which of the following organisms would most likely be isolated from the placental membranes?

A. Borrelia burgdorferi

B. Leptospira interrogans

C. Listeria monocytogenes

D. Spirillium minus

E. Streptobacillus moniliformis

Explanation:

The correct answer is C. Spontaneous abortion is a problem associated with Listeriosis, caused by Listeria
monocytogenes. The pattern of abortions was first recognized in herd animals, notably sheep and cattle, and
then listeriosis was later implicated as a cause of spontaneous abortion in pregnant women. Listeriosis can
occur in either epidemic (food-borne or hospital-acquired) forms or may be sporadic (noticed in animal or animal
product handlers). Soft cheeses like Brie are a particularly common source of food-borne listeriosis. The disease
may range in severity from asymptomatic carrier cases, to flu-like illness, to spontaneous abortion or neonatal
death, to fatal illness in children or adults secondary to septicemia or meningoencephalitis. Other localized
infections can also occur, primarily in the immunosuppressed. The treatment of choice is intravenous
administration of ampicillin or penicillin, often in combination with an aminoglycoside.
Trimethoprim-sulfamethoxazole has been used successfully in patients with penicillin allergy.

Borrelia burgdorferi(choice A) causes Lyme disease.

Leptospira interrogans(choice B) causes leptospirosis.

Spirillium minus(choice D) is a cause of rat-bite fever.

Streptobacillus moniliformis(choice E) is a cause of rat-bite fever.

A 14-year-old boy has just moved with his family from Brazil to the U.S. He starts complaining of shortness of
breath and palpitations. Chest x-ray films demonstrate pulmonary congestion, and ECG shows alterations in
heart rhythm. Echocardiography reveals biventricular dilatation with massive cardiac enlargement. An
endomyocardial biopsy shows diffuse interstitial fibrosis, myocyte necrosis, chronic inflammation, and the
presence of intracellular protozoan parasites. The patient may also develop which of the following complications?

A. Achalasia

B. Chronic arthritis

C. Cysts in the brain


20

D. Pleuritis

E. Splenomegaly

Explanation:

The correct answer is A. The patient has myocarditis due to Trypanosoma cruzi. This infectious condition,
known as Chagas disease, is endemic in vast areas of South America and is transmitted from person to person
by triatomids known as "kissing bugs." Experts assess the number of persons with Chagas disease at about 7
million with about 35 million at risk in South America. T. cruzi is an intracellular protozoon that localizes mainly in
the heart and nerve cells of the myenteric plexus, leading to myocarditis and dysmotility of hollow organs, such
the esophagus, colon, and ureter. Cardiac involvement manifests with ventricular dilatation and congestive
heart failure secondary to myocyte necrosis and fibrosis. Intracellular parasites can be visualized in tissue
sections. Chagas disease is a cause of acquired achalasia, in which the distal third of the esophagus dilates
because of loss of its intrinsic innervation. A similar pathologic mechanism accounts for megacolon and
megaureter in Chagas disease.

The remaining choices refer to different infectious conditions that may also involve the myocardium:

Chronic arthritis (choice B) is a manifestation of the chronic stage of Lyme disease, which is caused by Borrelia
burgdorferi and is transmitted to humans by deer ticks. Skin, CNS, and heart are the main targets of this
infection.

Cysts in the brain (cysticerci) (choice C) may develop as a consequence of infestation by the tapeworm Taenia
solium. Humans acquire this parasite by ingesting the eggs from undercooked pork. Cysticercosis may also
affect the heart, skeletal muscle, and skin.

Group B coxsackievirus infections cause pleuritis (choice D) and myocarditis, manifesting with fever, chest pain,
and, if myocarditis is severe, congestive heart failure. As in any form of viral myocarditis, the myocardium is
infiltrated by lymphocytes, but there are no morphologic markers specific for Coxsackievirus infection.

Splenomegaly (choice E), often of massive proportions, is seen in patients with malaria. Plasmodium organisms
can also invade the myocardium, leading to myocarditis.

A debilitated 72-year-old woman develops dry cough, fever, headache, and muscular pains. She treats herself
with aspirin and ampicillin without any improvement. Her children take her to a local hospital, where chest x-ray
films reveal scattered opacities, suggestive of interstitial infiltration. Laboratory investigations demonstrate the
presence of cold agglutinins. She is treated with erythromycin, and her symptoms rapidly improve. Which of the
following is the most likely etiologic agent of this patient's condition?

A. Influenza virus

B. Mycoplasma pneumoniae

C. Pneumocystis carinii

D. Respiratory syncytial virus

E. Streptococcus pneumoniae

Explanation:

The correct answer is B. The patient's clinical presentation is typical of primary atypical pneumonia. In contrast
to bacterial pneumonia, primary atypical pneumonia presents with the following features:
21
- Caused by M. pneumoniae; less frequently by viruses (influenza, respiratory syncytial virus, adenovirus,
rhinoviruses, rubeola and varicella virus), Chlamydia, or Coxiella burnetii

- Characterized pathologically by interstitial, rather than intra-alveolar, inflammation

- Characterized clinically by nonspecific symptomatology and few "localizing" symptoms

Why is M. pneumoniae, and not influenza virus (choice A) or respiratory syncytial virus (choice D), the cause of
this patient's pneumonia? First, M. pneumoniae infections are often associated with the appearance of cold
agglutinins in the serum, detection of which is diagnostically important. Second, the patient responded quickly to
treatment with erythromycin, an antibiotic effective against M. pneumoniae, but obviously not effective in treating
viral infections.

Pneumocystis carinii(choice C) is a fungal organism causing pneumonia in severely immunocompromised hosts,


especially AIDS patients. P. carinii pneumonia (PCP) is characterized by accumulation of a frothy exudate
containing numerous organisms within alveolar spaces. Also, P. carinii is not sensitive to erythromycin. The drug
of choice for treatment of PCP is trimethoprim-sulfamethoxazole.

Streptococcus pneumoniae(choice E) is the usual causative agent of lobar pneumonia, characterized by


consolidation of a single lobe due to intra-alveolar acute inflammatory exudation. Lobar pneumonia is more
prevalent in young, healthy individuals, whereas primary atypical pneumonia favors old, debilitated patients. S.
pneumoniae is highly sensitive to penicillin.

A 25-year-old female presents with a confluent maculopapular rash that began on her face, then spread
downward over her trunk. She states that 3 days ago she started having a fever and headache, with bilateral
pain associated with the front and back of her neck. She also complains of joint pain. Which of the following
diseases does she most likely have?

A. Infectious mononucleosis

B. Lyme disease

C. Roseola

D. Rubella

E. Rubeola

Explanation:

The correct answer is D. Rubella, also called German measles or 3-day measles, is a disease caused by a
Togavirus, which are small, enveloped, single-stranded, (+) linear RNA viruses. Approximately 40% of patients
are asymptomatic or have mild symptoms. In symptomatic patients, the clinical presentation typically consists of
an erythematous rash beginning on the head, which spreads downward to involve the trunk, lasting for
approximately 3 days. In addition to a transient rash, symptoms include fever, posterior cervical
lymphadenopathy, and arthralgias. The greatest danger from rubella is to the fetus. If clinical rubella develops
or seroconversion is demonstrated, there is a high risk of congenital abnormalities or spontaneous abortion.
The risk varies from 40%–60% if infection occurs during the first 2 months of gestation to 10% by the 4th
month. Females of childbearing age should be warned not to become pregnant within 2-3 months from the time
of immunization. Mild arthralgias and other symptoms may develop in 25% of immunized women. Enteroviral
rashes may mimic rubella and rubeola.

Infectious mononucleosis (choice A) is caused by the Epstein-Barr virus, a herpesvirus. Classic findings include
fever, exudative pharyngitis, generalized lymphadenopathy, severe malaise (most common complaint), and
hepatosplenomegaly. A rash is not a characteristic feature unless the patient has been treated with ampicillin.

Lyme disease (choice B) is caused by the spirochete Borrelia burgdorferi. The disease is transmitted by the bite
22
of the tick, Ixodes dammini. Reservoirs in nature include the white-tailed deer and the white-footed mouse. The
initial lesion is an annular rash with central clearing and a raised red border (erythema chronicum migrans) at
the bite site. The rash is warm, but not painful or itchy. Patients also have fever, malaise, myalgias, arthralgias,
headache, generalized lymphadenopathy, and, occasionally, neurologic findings.

Roseola (choice C) is caused by human herpesvirus 6. Other names include exanthem subitum or sixth
disease. Children have a febrile period of 3–5 days with rapid defervescence followed by an
erythematous maculopapular rash lasting 1–3 days.

Rubeola (choice E), or regular measles, is a disease caused by a paramyxovirus. Patients present with an
upper respiratory prodrome and characteristic oral lesions (Koplik's spots) that precede the rash. The
non-pruritic maculopapular rash begins on the face and spreads to the trunk and extremities, including palms
and soles. The incubation period is 10–14 days. Patients also have a posterior cervical
lymphadenopathy. The virus is not associated with risk to a fetus.

A Michigan fisherman presents with complaints of chronic diarrhea and fatigue. His physician orders a complete
blood count with differential, which reveals a megaloblastic anemia. Which of the following organisms is the most
likely cause of this patient's problems?

A. Clonorchis sinensis

B. Diphyllobothrium latum

C. Echinococcus granulosus

D. Taenia saginata

E. Taenia solium

Explanation:

The correct answer is B. Vitamin B12 deficiency with resulting megaloblastic anemia is specifically associated
with infection with the fish tapeworm, Diphyllobothrium latum, found in Scandinavia and the Great Lakes. The
adult worm attaches to the small intestinal mucosa and releases eggs into the feces. Crustaceans living in
contaminated water ingest free-swimming embryos that hatch from the eggs. Certain species of fish ingest the
crustaceans, then humans acquire the infection by ingesting the undercooked or raw fish. Vitamin B12
deficiency can develop because the tapeworm absorbs vitamin B12 from the intestinal lumen and also interferes
with ileal B12 absorption.

Clonorchis sinensis(choice A) infections produce obstruction of the biliary tract or pancreatic duct and are
associated with an increased risk of cholangiocarcinoma.

Echinococcus granulosus(choice C) larvae infect humans and produce unilocular cystic lesions in the liver.
Echinococcus is prevalent in areas in which dogs are used to help raise livestock.

Taenia saginata is the beef tapeworm (choice D); it causes diarrhea but not megaloblastic anemia.

Taenia solium(choice E) is the pork tapeworm. It occasionally causes diarrhea, but is better known as the cause
of cysticercosis. It does not produce megaloblastic anemia.

A child with sickle cell anemia is seen in a hematology clinic. Her mother states that she has been feeling very
tired lately, and may have "come down with a virus." On physical examination, the girl is very pale, and a
complete blood count shows severe anemia. A bone marrow aspirate contains no erythroid precursor cells. The
girl was probably infected with which of the following viruses?
23

A. Coxsackie virus

B. Echovirus

C. Hepadnavirus

D. Herpes virus

E. Parvovirus

Explanation:

The correct answer is E. Parvoviruses are small single-stranded DNA viruses, of which only serotype B19 is
pathogenic for humans. This virus causes three distinct syndromes: a childhood febrile rash known as
erythema infectiosum ("Fifth disease"); aplastic crisis in individuals with chronic hemolytic diseases (sickle cell
anemia, thalassemia, etc); and congenital infections that can present as stillbirth, hydrops fetalis (analogous to
severe Rh incompatibility), or severe anemia.

Coxsackie viruses (choice A) usually cause cold-like illness, but can cause herpangina, myocarditis, and
meningitis.

Echoviruses (choice B) can infect a variety of organ systems (GI, CNS, eyes, heart, respiratory, skin), but are
not a cause of aplastic crises.

Hepadnavirus (choice C) is the causative agent of hepatitis B.

Herpes viruses (choice D) cause a variety of acute to chronic infections including herpes simplex types I and II,
chicken pox, chronic herpes zoster, CMV infection, and Epstein Barr virus infections.

A 43-year-old executive presents to a physician with chronic, symmetric polyarthritis involving the knees. The
man gives a history of having developed an extensive rash after a deer hunting trip in Connecticut several years
earlier. He recalls that he felt "sick" at the time, and developed knee pain that prevented him from climbing stairs
for several months, but then partially resolved. Which of the following organisms is most likely etiologically related
to the patient's arthritis?

A. Fungus

B. Gram-negative cocci

C. Gram-negative rod

D. Gram-positive cocci

E. Spirochete

Explanation:

The correct answer is E. The history is characteristic for Lyme disease, caused by the spirochete Borrelia
burgdorferi. The clues are deer, Connecticut, rash, knees, and acute arthritis followed by chronic arthritis.

Fungal arthritis (choice A) can be caused by coccidioidomycosis, blastomycosis, sporotrichosis, and


candidiasis.

N. gonorrhoeae are gram-negative cocci (choice B) that can cause septic arthritis.
24
Haemophilus (young children) and Salmonella (sickle cell patients) are gram-negative rods (choice C) that can
cause septic arthritis.

Staphylococcus are gram-positive cocci (choice E) that can cause septic arthritis.

A 39-year-old HIV-positive man is brought into the emergency room after experiencing a seizure witnessed by
several friends. The observers relate that the patient suddenly lost consciousness and experienced both leg and
arm jerking. The man's tongue has been severely bitten, and loss of bowel and bladder function is evident upon
admission. On physical examination, the patient is lethargic, unable to answer simple questions, and has an
obvious left-sided hemiparesis. An MRI of the head shows multiple ring-enhancing lesions. Infection with which of
the following agents is most likely responsible for this presentation?

A. Cryptococcus neoformans

B. Herpes simplex

C. Mycobacteria tuberculosis

D. JC virus

E. Toxoplasma gondii

Explanation:

The correct answer is E. While all five pathogens can cause central nervous system (CNS) manifestations,
toxoplasmosis (caused by Toxoplasma gondii) presents with seizures in 15-25% of cases. The disease is
spread by ingestion of cysts from undercooked meat or from cat feces. Tachyzoites develop from cysts
phagocytized by macrophages, then spread to the brain, muscle, and other tissues, where they encyst and
multiply. Primary CNS lymphoma in AIDS patients can present clinically in nearly the same manner as
toxoplasmosis, but the course is usually much more indolent.

Cryptococcal meningitis (choice A) usually presents as a subacute meningitis with headache, nausea, vomiting,
and confusion. Cranial nerve abnormalities are common with cryptococcal infections.

Herpes simplex encephalitis (choice B) typically has a subacute onset with headache, meningismus, and
personality changes.

Mycobacterium tuberculosis(choice C) is a cause of a basilar meningitis, which can present insidiously with
headache and mental changes over a week or two, or can present acutely as confusion, lethargy, altered
sensorium, and a stiff neck. Cranial nerve palsies, focal cerebral ischemia, and hydrocephalus are
characteristic.

Progressive multifocal leukoencephalopathy (PML) is a demyelinating disease caused by JC virus (choice D), a
human polyomavirus. PML occurs in about 4% of patients with AIDS. The disorder has a slow, insidious onset,
in which altered mental status is not a prominent feature.

Which of the following organisms would most likely cause infection after a sterilization procedure that killed
vegetative cells but did not kill spores?

A. Chlamydia

B. Clostridium

C. Escherichia
25

D. Pseudomonas

E. Streptococcus

Explanation:

The correct answer is B. Although bacterial spores are extensively discussed in microbiology courses, you
should be aware that only Bacillus (aerobic) and Clostridium (anaerobic) species produce spores. This means
that the list of diseases related to bacterial spore formation is also small: anthrax (Bacillus anthracis), some
forms of food poisoning (Bacillus cereus), botulism (Clostridium botulinum), tetanus (Clostridium tetani), gas
gangrene (Clostridium perfringens and others), and pseudomembranous colitis (Clostridium difficile).

A newborn in the neonatal intensive care unit becomes tachypneic and irritable. Blood cultures grow a
gram-positive coccus in chains that is presumed to be a streptococcus. Which of the following characteristics
would help to differentiate Streptococcus agalactiae from Streptococcus pneumoniae?

A. Alpha-hemolysis

B. Carbohydrate capsule

C. Cytochrome enzyme system

D. Growth in bile

E. Oxacillin sensitivity

Explanation:

The correct answer is A. Streptococci are usually initially speciated by their hemolytic capacity on sheep blood
agar. Beta-hemolytic streptococci include groups A, B, and D. S. agalactiae is the classic group B
streptococcus. The non beta-hemolytic streptococci consist principally of the pneumococci and the viridans
group.

Both S. agalactiae and pneumococcus have a carbohydrate capsule (choice B), an important virulence factor
and means of subtyping streptococcal species.

None of the streptococci utilize cytochrome enzymes (choice C). They derive all of their energy from the
fermentation of sugars to lactic acid.

Neither pneumococcus nor S. agalactiae can grow in bile (choice D). This ability is specific for the enterococcus
group (group D) of streptococci.

Both pneumococci and S. agalactiae are usually treated with penicillin-type antibiotics, although group B
streptococci require a penicillinase-resistant type such as oxacillin (choice E).

A 35-year-old man presents to the emergency room with intense back pain. He is hydrated and given pain
medication. After several hours he passes a kidney stone. Laboratory analysis of the stone reveals that it is
composed of struvite (magnesium ammonium phosphate). Infection with which of the following organisms promotes
the production of such stones?

A. Escherichia coli
26
B. Proteus mirabilis

C. Pseudomonas aeruginosa

D. Staphylococcus saprophyticus

E. Ureaplasma urealyticum

Explanation:

The correct answer is B. Proteus species produce urease, which raises the urinary pH and promotes the
production of struvite stones.

E. coli(choice A) are by far the most common cause of urinary tract infections, but are not responsible for the
development of struvite stones.

Pseudomonas aeruginosa(choice C) can also cause urinary tract infections, most commonly in hospitalized or
immunocompromised patients (especially burn patients, patients on immunosuppressives/antimicrobials, and
cystic fibrosis patients). It does not predispose to struvite stones.

Staphylococcus saprophyticus(choice D) is a common cause of urinary tract infections in sexually active young
women.

Ureaplasma urealyticum(choice E) can produce urease (like Proteus), but they are responsible for urethritis, not
stones.

A 54-year-old truck driver in Oklahoma collides with a deer on the highway. He takes the deer home and
butchers it. Five days later, he develops fever, non-productive cough, shortness of breath, and generalized
lymphadenopathy. No skin lesions are noted. What is the most likely diagnosis?

A. Anthrax

B. Leptospirosis

C. Pasteurella multocida infection

D. Plague

E. Tularemia

Explanation:

The correct answer is E. Tularemia is caused by Francisella tularensis. Fifty percent of the cases have been
reported in Missouri, Arkansas, and Oklahoma. Infection can be transmitted by tick bites or by handling animal
carcasses such as rabbits, squirrels, beavers, muskrats, and deer. The diagnosis is suggested by the history of
exposure, clinical progression, and generalized lymphadenopathy. The ulceroglandular form of tularemia is the
most common clinical presentation.

Anthrax (choice A) is caused by Bacillus anthracis. Typically, infection with B. anthracis causes a localized skin
lesion at the site of inoculation that develops into a black eschar surrounded by draining lymphadenitis.
Woolsorter's disease is the inhalation form of anthrax.

Leptospirosis (choice B) is an infection caused by contact with urine from infected animals such as rats and
dogs. L. interrogans is the most common isolate. The disease is biphasic, with the leptospiremic phase
characterized by abrupt-onset headache (98%), fever, chills, conjunctivitis, severe muscle aches,
gastrointestinal symptoms, changes in sensorium, rash, and hypotension. This phase lasts from 3 to 7 days.
27
The immune stage occurs after a relatively asymptomatic period of 1 to 3 days and is characterized by
recurrence of fever and generalized symptoms. Meningeal symptoms often develop during the immune period.
In more serious cases, hepatic dysfunction and renal failure may develop.

Pasteurella multocida(choice C) is associated with dog and cat bites. It causes a rapidly progressing cellulitis,
bacteremia, and, occasionally, infective endocarditis.

Plague (choice D) is caused by Yersinia pestis. It is enzootic in the southwestern United States. Rats and fleas
are the vectors. Clinical presentations include lymphadenopathy with septicemia or pneumonia (which has the
highest case-fatality rate).

A 67-year-old black man with a history of glucose-6-phosphate dehydrogenase (G-6-PD) deficiency presents with
fever, irritative voiding symptoms, and perineal pain. Rectal examination is remarkable for a boggy, exquisitely
tender prostate. A urine Gram's stain is positive for gram-negative rods. The risk for development of hemolytic
anemia is highest if he receives high-dose, 21-day therapy with

A. ampicillin

B. cefaclor

C. ciprofloxacin

D. sulfamethoxazole/trimethoprim

E. tetracycline

Explanation:

The correct answer is D. The patient has acute prostatitis, which is characterized by fever, chills, and dysuria,
with a swollen, extremely tender prostate on rectal exam. The urine Gram's stain and culture will generally be
positive. The treatment regimen for this bacterial infection is typically a 21-day course of ampicillin, a
fluoroquinolone, or sulfamethoxazole/trimethoprim (SMX-TMP). G-6-PD deficiency is an X-linked recessive
disorder affecting 10% to 15% of American black males. The medications most commonly associated with the
induction of hemolytic anemia in deficient patients are sulfonamides, nitrofurantoin, dapsone, primaquine, and
quinine. The sulfamethoxazole in the SMX-TMP combination is a sulfonamide, and can produce hemolytic anemia
in patients with G-6-PD deficiency.

Ampicillin (choice A) is a broad-spectrum penicillin antibiotic commonly used in the treatment of infections in the
genitourinary, respiratory, or GI tracts, as well as in the skin and soft tissues.

Cefaclor (choice B) is a second-generation cephalosporin indicated for a variety of bacterial infections, including
those of the respiratory and GI tracts.

Ciprofloxacin (choice C) is a fluoroquinolone commonly used in the treatment of serious infections caused by
gram-negative organisms.

Tetracycline (choice E) is most commonly used in the treatment of acne vulgaris and gonococcal infections.

A 25-year-old man presents with a high fever and generalized malaise. His condition deteriorates so rapidly that
his friends decide to take him to the emergency department 24 hours after the onset of symptoms. He has a
history of intravenous drug abuse. A test for anti-HIV antibodies is negative. Physical examination reveals a
systolic murmur, and echocardiography shows bulky vegetations attached to the tricuspid valve leaflets. Which of
the following microorganisms will be most likely be isolated from this patient's blood cultures?

A. Candida albicans
28
B. Hemophilus influenzae

C. Staphylococcus aureus

D. Staphylococcus epidermidis

E. Viridans (α-hemolytic) streptococci

Explanation:

The correct answer is C. The patient has a fever and is extremely ill. The most important clue to the diagnosis is
the presence of bulky vegetations on the tricuspid valve, indicating that he has infective endocarditis. On the
basis of the rapid clinical course, this is likely a case of acute infective endocarditis. The diagnosis of this
condition must be confirmed by blood cultures, which are also necessary to determine bacterial antibiotic
sensitivity. S. aureus (commonly present on the skin) is the most frequent etiologic agent of infective
endocarditis in intravenous drug abusers. It commonly affects the tricuspid valve. Because of its high virulence,
S. aureus-related endocarditis follows an acute course and may lead to death within a few days.

The causative agents of infective endocarditis differ depending on host factors. Fungal organisms, such as
Candida albicans(choice A), may cause infective endocarditis in severely immunosuppressed patients, such as
those with AIDS.

A minority of cases of infective endocarditis are caused by a number of normal commensals in the oral cavity,
i.e., the "HACEK" group: Hemophilus(choice B), Actinobacillus, Cardiobacterium, Eikenella, and Kingella.

S. epidermidis(choice D) and other coagulase-negative staphylococci tend to produce endocarditis in recipients


of prosthetic valves.

Viridans streptococci (choice E) are the most frequent agents causing endocarditis in previously abnormal
valves, such as those damaged by rheumatic disease, or congenitally abnormal valves. Coagulase-negative
staphylococci and viridans (α-hemolytic) streptococci are less virulent than S. aureus and are thus
associated with a subacute (more prolonged) clinical course and a better prognosis.

A newborn infant develops respiratory distress shortly after delivery and is taken to the neonatal intensive care
unit for observation. The next day she becomes febrile with persistent dyspnea and coughing. Sputum recovered
by the respiratory therapist reveals numerous neutrophils and gram-negative rods that grow as pink colonies on
MacConkey's agar. The infective organism most likely belongs to which of the following genera?

A. Escherichia

B. Proteus

C. Pseudomonas

D. Salmonella

E. Shigella

Explanation:

The correct answer is A. Essentially, this question tests your understanding of MacConkey's agar and your
knowledge of the biochemical characteristics of the Enterobacteriaceae. MacConkey's agar is used to identify
organisms that ferment lactose, and consequently grow as pink colonies. Escherichia coli is one of several
lactose-fermenting Enterobacteria, along with Serratia, Klebsiella, and Citrobacter species. Neonatal pneumonia
due to aspirated E. coli during delivery is not uncommon, and it should be treated with a third-generation
29
cephalosporin and/or aminoglycoside.

Proteus species (choice B) are motile, non-lactose fermenting bacteria that characteristically swarm on an agar
plate. Proteus infections generally affect the genitourinary tract, but Proteus is an important nosocomial
pathogen.

Pseudomonas species (choice C) are normally widespread in nature, but may form a small portion of the bowel
flora. Pseudomonas utilizes cytochrome oxidase for energy production; it does not ferment any sugars.

Salmonella species (choice D) are non-lactose fermenters that may produce inflammatory diarrheas. They are
normally distinguished from the other Enterobacteriaceae by their ability to produce acid and gas from
dextrose-containing media.

Shigella species (choice E) are non-motile, non-lactose fermenting bacteria that live exclusively in the
gastrointestinal tract of primates. They typically produce dysentery.

Five days after returning to his military base in South Carolina after survival training in the nearby countryside, an
18-year-old recruit reports to the infirmary complaining of a headache. Physical examination reveals a fever, but
no other abnormalities are noted. A few days later he returns to the infirmary with a maculopapular rash involving
the hands and feet. The rash then spreads centripetally to involve the trunk. Which of the following diseases
should be suspected?

A. Chickenpox

B. German measles

C. Measles

D. Mumps

E. Rocky Mountain spotted fever

Explanation:

The correct answer is E. Take rashes involving the palms and soles (otherwise unusual sites) very seriously:
only a small number of infections can cause this pattern, including Rocky Mountain spotted fever,
meningococcemia, and secondary syphilis. Rocky Mountain spotted fever is caused by the rickettsia R.
rickettsii, and is found throughout the United States, particularly in south central and eastern portions (not the
Rocky Mountains). 3-12 days after a tick bite, patients develop malaise, frontal headache, and fever. Several
days later, the rash described in the question stem develops. Other manifestations can include
hepatosplenomegaly, thrombocytopenia, and (potentially fatal) disseminated intravascular coagulation.

Chickenpox (choice A), or varicella, is characterized by maculopapules that evolve into vesicles over hours to
days, then eventually form crusts. Typical lesions first appear on the trunk and face and rapidly spread to
involve other areas.

The maculopapular rash of German measles (choice B), or rubella, usually begins on the face, then spreads
down the body.

Although the maculopapular rash of measles (choice C), or rubeola, can include the palms and soles, it typically
begins along the hairline in frontal and temporal regions, then spreads down the trunk to the limbs.

Mumps (choice D) is characterized by fever, malaise, parotitis, and orchitis, but not a rash.
30
A 47-year-old grocer complains of diarrhea and painful arthritis. Physical examination is remarkable for
lymphadenopathy and weight loss. Biopsy of his small bowel reveals PAS-positive macrophages within the lamina
propria. Electron microscopic examination of the macrophages reveals small rod-shaped structures. These
structures are most likely

A. Clostridium

B. Enterotoxigenic E. coli

C. Isospora

D. Salmonella

E. Tropheryma

Explanation:

The correct answer is E. For many years, Whipple's disease was suspected of having a bacterial etiology
because bacterial forms could be seen on electron microscopy. However, the identity of the causative agent
remained elusive. It has recently been found to be a bacterium which is now named Tropheryma whippelii.

Clostridium difficile(choice A) causes pseudomembranous colitis, generally after antibiotic administration.

Enterotoxigenic E. coli(choice B) is associated with tropical sprue.

Isospora bella(choice C) is a cause of diarrhea in AIDS patients.

Salmonella sp.(choice D) can cause diarrheal illness after ingestion of contaminated poultry or beef.

A 33-year-old G1P0 female at 6 months gestation returns from a visit to her parents house in Arizona.
Approximately 7 days following her return, she develops axillary lymphadenopathy and a low-grade fever. Her
physician notices a small papule and healing scratch on her arm on the affected side. The patient states that she
has pet birds at home, and there was a new kitten at her mother's house. She does not remember receiving the
scratch. Which of the following organisms is most likely responsible for this illness?

A. Bartonella henselae

B. Coccidioides immitis

C. Cryptococcus neoformans

D. Histoplasma capsulatum

E. Pasteurella multocida

Explanation:

The correct answer is A.Bartonella henselae is a gram-negative bacillus that is the causative agent of cat
scratch disease. Cats, especially kittens, harbor the organism. It is usually inoculated through an easily
forgotten scratch. A pustule forms at the site within days to weeks, accompanied by tender regional
adenopathy. Microscopically, granulomas with central necrosis and organisms are observed. Normally,
treatment is not required because this disease is benign and self-limited, but immunosuppressed and pregnant
patients are more susceptible to dissemination. Treatment is with ciprofloxacin or doxycycline. Aminoglycosides
or erythromycin have also been employed.
31
Coccidioidomycosis is a disease caused by the fungus C. immitis(choice B). Spores of the organism are
inhaled, usually from the soil. These spores enlarge to sporangia, which burst and spread. The disease is
characterized by pulmonary symptoms, but is usually self-limited. This organism is common in arid desert states
like Arizona, and can affect people after brief visits.

Cryptococcus neoformans(choice C) is a fungus found worldwide. The main reservoir is pigeon droppings. The
small cryptococci are inhaled into the lungs, but do not usually cause disease in immunocompetent individuals.
In the immunosuppressed patient, Cryptococcus can cause pulmonary and CNS disease, especially meningitis.

Histoplasma capsulatum(choice D) is an infection acquired by breathing dust that contains bird/bat droppings
that contain spores of the organism. The primary infection is in the lungs, and may be latent, or brief and
self-limited. Symptoms include fever, cough and sweats. In the United States, it is common in the
Ohio-Mississippi valley.

Pasteurella multocida(choice E) is a gram-negative rod spread via dog and cat bites. The infection is
characterized by rapidly evolving cellulitis starting at the site of the bite. There was no indication in this patient
of an animal bite.

A 33-year-old woman begins seeing a new boyfriend. Several weeks later, a week after her menstrual period, the
woman develops a painful, swollen right knee. The most likely causative organism has which of the following
characteristics?

A. Both hyphae and spores

B. Can live within neutrophils

C. Cannot make ATP

D. No true cell wall

E. Stains with silver stains

Explanation:

The correct answer is B. The causative organism is Neisseria gonorrhoeae, which is a sexually transmitted
gram-negative coccus that can live in phagocytic vacuoles in neutrophils and macrophages. The organism is
the most common cause of septic arthritis in otherwise healthy, sexually active adults.

Hyphae and spores (choice A) would be features of Candida, which usually causes arthritis as a rare
complication of systemic candidiasis in immunocompromised patients.

The Chlamydia are intracellular organisms that cannot make ATP (choice C) and do not usually cause septic
arthritis.

Mycoplasma do not have true cell walls (choice D) and do not usually cause septic arthritis.

Syphilis is caused by a spirochete that stains with silver stains (choice E). Secondary syphilis can (uncommonly)
cause an acute arthritis, but the interval between acquiring the infection and developing the arthritis is usually
months to years.

A 32-year-old man presents to the emergency room with a severe headache. Nuchal rigidity is found on physical
examination. Lumbar puncture demonstrates cerebrospinal fluid with markedly increased lymphocytes. Other cell
populations are not increased. Which of the following agents is the most likely cause of his symptoms?
32
A. Escherichia coli

B. Haemophilus influenzae

C. Herpes virus

D. Mycobacterium tuberculosis

E. Treponema pallidum

Explanation:

The correct answer is C. The clinically suspected diagnosis is meningitis, which is confirmed by the abnormal
cerebrospinal fluid. The markedly increased lymphocytes suggests acute lymphocytic meningitis, which is
distinguished from acute pyogenic meningitis (increased neutrophils as well as lymphocytes). Acute lymphocytic
meningitis is usually viral in origin. Among the many viruses that have been implicated, mumps, herpes,
Epstein-Barr, echovirus, and Coxsackie virus are the most common.

Escherichia coli(choice A) and Haemophilus influenzae(choice B) cause acute pyogenic meningitis.

Mycobacterium tuberculosis(choice D) and Treponema pallidum(choice E) cause chronic meningitis.

A febrile 12-year-old child presents with severe right lower quadrant pain that is interpreted by the attending
physician as acute appendicitis. The patient has also been complaining of joint pain. At laparotomy, the surgeon
notes that the appendix is normal; however, the mesenteric lymph nodes are markedly enlarged and contain focal
areas of microabscess formation on cut section. This patient is most likely

A. an asthmatic

B. deficient in C1 esterase inhibitor activity

C. HLA-B27 positive

D. leukopenic

E. serologically positive for toxoplasmosis

Explanation:

The correct answer is C.Yersinia enterocolitica is the pathogen producing this clinical syndrome. Yersinia is
transmitted to patients via the oral route, via contaminated blood products, or by cutaneous inoculation.
Patients developing iron overload because of multiple transfusions (i.e., thalassemia patients) are at increased
risk of Yersinia infections because some strains are unable to synthesize bacterial iron chelators called
siderophores. They can, however, use host-chelated iron stores or the drug deferoxamine (a siderophore
produced by Streptomyces pilosus). Yersinia is associated with reactive arthritis following an infection by an
enteropathogenic organism. Most patients who develop arthritis express HLA-B27.

A history of asthma (choice A) is not associated with Yersinia infections.

Deficiency in C1 esterase inhibitor activity (choice B) produces the syndrome of angioedema. This is an
autosomal dominant trait associated with a deficiency of the serum inhibitor of the activated first complement
component. The patients have multiple episodes of edema, affecting skin and mucosal surfaces such as the
larynx and the GI tract.

Leukopenia (choice D) is not associated with the clinical scenario in the question. Normal-to-elevated leukocyte
counts are the rule in Yersinia infection.
33
Serological positivity for toxoplasmosis (choice E) is not suggested because toxoplasmosis produces a different
clinical picture. Toxoplasmosis can be acquired transplacentally with infected infants exhibiting a wide array of
symptoms depending on when the mother was infected during gestation. Toxoplasmosis is also seen in
immunocompromised patients such as those with AIDS. In this group of patients, the main organ system
involved is the central nervous system.

A 16-year-old girl presents to her physician complaining of redness and a yellowish discharge from her left eye
for the past 2 days. She reports minimal crusting upon awakening and denies eye pain or previous trauma. Upon
examination, there is diffuse conjunctival hyperemia associated with a mucoid discharge. The pupils dilate
normally, although there is mild photophobia in the affected eye. Gram's stain of the exudate shows pleomorphic,
gram-negative bacilli. To culture this organism with whole blood agar, which of the following must be used?

A. Antibiotics to inhibit normal flora

B. Cold temperature

C. Egg yolk cholesterol

D. Reduced oxygen tension

E. Staphylococcus aureus

Explanation:

The correct answer is E. The patient is suffering from conjunctivitis caused by Hemophilus, a fastidious
organism requiring factors V (nicotinamide adenine dinucleotide) and X (hematin), which must be released from
whole blood preparations to be accessible to the organism. This can be done by gentle heating, which lyses the
red blood cells, or by co-culture with Staphylococcus aureus, which is beta hemolytic and lyses the red blood
cells to provide the factors Hemophilus needs for growth. This is called the satellite phenomenon.

Antibiotics (choice A) would not be necessary since, with proper culture techniques, no normal flora should be
available to confuse the diagnosis from this site.

Cold temperature (choice B) would not be necessary because Hemophilus grows at normal body temperature.
Listeria is the genus commonly mentioned for which a cold temperature is necessary.

Cholesterol (choice C) is necessary for the growth of Mycobacteria, but not Hemophilus.

Low oxygen (choice D) tension is necessary for the growth of the microaerophiles (such as Campylobacter,
Helicobacter, and Borrelia), but is not a condition for growth of Hemophilus.

A baby born at 32 weeks gestation with Apgar scores of 2 and 7 was placed in the neonatal intensive care unit.
She developed a fever and leukocytosis; lumbar puncture revealed pleocytosis with increased protein, decreased
glucose, and gram-positive rods. Which one of the following organisms was most likely isolated from the CSF?

A. Escherichia coli

B. Listeria monocytogenes

C. Neisseria meningitidis

D. Streptococcus agalactiae

E. Streptococcus pneumoniae
34

Explanation:

The correct answer is B. The three major causes of neonatal meningitis are group B streptococci
(Streptococcus agalactiae; choice D), Escherichia coli(choice A), and Listeria monocytogenes. All can be found
in the vaginal tract of normal women and may contaminate the infant during passage through the birth canal.
They colonize the upper respiratory tract and can cause pneumonia, septicemia, and/or meningitis in the
neonate. They are readily distinguished on morphologic grounds; the streptococci are gram-positive cocci in
chains, E. coli is a gram-negative rod, and L. monocytogenes is a gram-positive pleomorphic rod. There are
other gram-positive rods that resemble Listeria (e.g., the diphtheroid bacilli found in the upper respiratory tract
and on the skin), hence a motility test is done to confirm the identification. L. monocytogenes is motile at room
temperature but not when grown at 37 degrees C. L. monocytogenes is also associated with drinking
unpasteurized milk.

Neisseria meningitidis(choice C) is the most common cause of meningitis in school-age children and young
adults. It is a fastidious, non-motile, gram-negative diplococcus that would be a very rare cause of meningeal
disease in very young patients, such as this one.

Streptococcus pneumoniae(choice E) is a gram-positive coccus that grows in pairs and short chains. It is the
number one cause of pneumonia, septicemia, and meningitis in the elderly. There is a vaccine composed of the
capsular carbohydrate of 23 serotypes of this organism that is routinely given to individuals over the age of 60,
as well as to individuals with splenic abnormalities (e.g., sickle cell disease) who are at increased risk for the
development of pneumococcal sepsis.

A 27-year-old intravenous drug user presents with difficulty swallowing. Examination of the oropharynx reveals
white plaques along the tongue and the oral mucosa. Which of the following best describes the microscopic
appearance of the microorganism responsible for this patient's illness?

A. Budding yeast and pseudohyphae

B. Encapsulated yeast

C. Mold with nonseptate hyphae

D. Mold with septate hyphae

Explanation:

The correct answer is A. This patient has Candida esophagitis. Any time a patient presents with dysphagia or
odontophagia and you note white plaques in the oropharnyx (thrush), you can assume that the Candida is
affecting the esophagus as well. The fact that the patient is an IV drug user makes an opportunistic infection
such as Candida more likely. Now comes the hard part–remembering the morphology of Candida.
Candida appears as budding yeast with pseudohyphae in vivo.

The other answer choices represent the morphology of other important opportunistic fungi:

Cryptococcus are encapsulated yeast (choice B). You should think about Cryptococcus neoformans when
you're presented with an immunocompromised patient with neurological symptoms. The classic clue is the
presence of encapsulated organisms observable in an India ink preparation.

Mucor and Rhizopus are molds with nonseptate hyphae (choice C). You should think about Mucor when you are
presented with a diabetic (especially ketoacidosis) or a leukemic patient with a severe sinus infection.

Aspergillus is a mold with septate hyphae (choice D). In immunocompromised patients, aspergillosis can present
with acute pneumonia, often with cavitation (aspergillomas = fungus balls in the lungs).
35

A 24-year-old woman in her third trimester of pregnancy presents with urinary frequency and burning for the past
few days. She denies fever, nausea, vomiting, or chills. She takes no medications besides prenatal vitamins and
is generally in good health. Physical examination is remarkable for mild suprapubic tenderness, and a urine
dipstick is positive for white blood cells, protein, and a small amount of blood. Culture produces greater than
100,000 colonies of gram-negative bacilli. Which of the following attributes of this uropathogenic organism is
most strongly associated with its virulence?

A. Bundle-forming pili

B. GVVPQ fimbriae

C. Heat labile toxins

D. Heat stable toxins

E. P pili

F. Type 1 pili

Explanation:

The correct answer is E. Urinary tract infections are the most common bacterial infections encountered during
pregnancy, and Escherichia coli is the most commonly isolated organism. 70% of cases in the U.S. are caused
by P pili-positive strains.

Bundle-forming pili (choice A) are found in enteroaggregative E. coli (EAEC).

GVVPQ fimbriae (choice B) are found in EAEC.

Heat labile toxins (choice C) are pathogenic factors in enterotoxic strains (ETEC).

Heat stable toxins (choice D) are pathogenic factors in ETEC or EAEC.

Type 1 pili (choice F) are a major pathogenic factor in ETEC.

A febrile 23-year-old college coed presents with fatigue and difficulty swallowing. Physical exam reveals exudative
tonsillitis, palatal petechiae, cervical lymphadenopathy, and tender hepatosplenomegaly. A complete blood count
reveals mild anemia, lymphocytosis with about 30% of the lymphocytes exhibiting atypical features, and a mild
thrombocytopenia. Coombs' test is positive. Which of the following is the most likely complication of this
syndrome?

A. Acute cholecystitis

B. Ascending cholangitis

C. Diarrhea

D. Immune complex vasculitis

E. Splenic rupture

Explanation:

The correct answer is E. The syndrome represented by the clinical vignette is infectious mononucleosis.
Epstein-Barr virus (EBV) is the usual cause of heterophile-positive infectious mononucleosis; cytomegalovirus is
36
responsible for a minority of cases. Rarely, splenic rupture requiring splenectomy can result from splenomegaly
and capsular swelling, usually occurring during the 2nd and 3rd weeks of the illness.

Acute cholecystitis (choice A) is not associated with infectious mononucleosis. The most frequently isolated
pathogens are E. coli, Klebsiella spp., group D Streptococcus, Staphylococcus spp., and Clostridium spp.

Ascending cholangitis (choice B) is not associated with infectious mononucleosis. Cholangitis usually presents
with biliary colic, jaundice, and spiking fever with chills (Charcot's triad). Blood cultures are usually positive (E.
coli is a common isolate), with an accompanying leukocytosis. AIDS-related cholangitis has been reported,
presenting with abdominal pain and obstructive liver symptoms. Potential etiologic agents include
Cytomegalovirus, Cryptosporidium parvum, and Microsporidia, including Enterocytozoon cuniculi.

Diarrhea (choice C) is not usually produced by infectious mononucleosis.

Immune complex vasculitis (choice D) is not associated with infectious mononucleosis.

A 37-year-old newly married man presents with multiple blister-like lesions on the glans of his penis, appearing
over the past 2 days. On questioning, he recalls similar episodes over the past 2 years. Examination is
remarkable for tender, 3-4 mm vesicular lesions on the shaft of his penis with no apparent crusting, drainage, or
bleeding. There is also slight bilateral inguinal adenopathy. During the asymptomatic period between outbreaks,
where would the causative agent likely have been found?

A. Fibroblasts

B. Lymphocytes

C. Macrophages

D. Mucoepithelial cells

E. Neurons of the sacral ganglia

F. Neurons of the trigeminal ganglia

Explanation:

The correct answer is E. This is a classic example of an infection with herpes simplex virus (probably type 2).
This agent causes lytic infections in mucoepithelial cells. It is then retrogradely transported into neurons of the
sacral ganglia, where it lays dormant during the asymptomatic phase of the disease.

Fibroblasts (choice A) may be infected by cytomegalovirus, another herpesvirus, but this produces a distinctive
mononucleosis-like syndrome in most normal individuals.

Lymphocytes (choice B) and macrophages (choice C) may be infected by herpes simplex type 2 but are not the
site of residence of the virus during quiescent periods.

Infection of mucoepithelial cells (choice D) by herpes simplex produces vesicular-like lesions on the mucous
membranes during symptomatic periods.

Neurons of the trigeminal ganglia (choice F) may be latently infected with herpes simplex type 1. This agent is a
possible cause of genital ulcers, but is usually associated with perioral lesions (cold sores).

A 67-year-old woman in a skilled nursing facility complains of flu-like symptoms. After several days, she develops
high fever, dyspnea, and a productive cough. The nurses also notice mental status changes, and she is
transported to the nearby community hospital. A chest x-ray shows a cavitary lesion in her left lung. Which of the
37
following organisms would most likely be identified from examination of her sputum?

A. Candida albicans

B. Klebsiella pneumoniae

C. Pneumocystis carinii

D. Staphylococcus aureus

E. Streptococcus pneumoniae

Explanation:

The correct answer is D. The woman has developed a pulmonary abscess, as evidenced by the chest
radiograph. Of the organisms listed, Staph. aureus is the most likely cause of bacterial pneumonia complicated
by abscess formation. Bacteria commonly producing pneumonia developing in the context of influenza include
Staphylococcus aureus, Haemophilus influenzae, and Streptococcus pneumoniae, but Streptococcus
pneumoniae is not a frequent cause of lung abscesses.

Candida albicans(choice A) would not be a likely cause of this woman's abscess, or of the pneumonia that
preceded it.

Klebsiella pneumoniae(choice B) is more likely in men who are alcoholic or diabetic, or suffering from chronic
obstructive lung disease.

Pneumocystis carinii(choice C) is associated with pneumonia in immunocompromised hosts, but there is no


indication of immune compromise in this patient.

Streptococcus pneumoniae(choice E) is only rarely associated with lung abscess.

A man presents to a dermatologist because of a severe mucocutaneous rash that involves most of his body,
including his palms and soles. Questioning reveals that he is a merchant marine who several months previously
had an encounter with a prostitute in Southeast Asia. Which of the following is the most likely causative agent of
this rash?

A. Herpes simplex I

B. Herpes simplex II

C. HIV

D. Neisseria gonorrhoeae

E. Treponema pallidum

Explanation:

The correct answer is E. The rash described is that of secondary syphilis, caused by Treponema pallidum.
Involvement of palms and soles by a rash is unusual, and secondary syphilis should come to mind. Not all
patients with secondary syphilis have a severe form of the rash, and consequentially some cases are missed.
Primary syphilis takes the form of a painless, button-like mass called chancres. Tertiary syphilis, which is now
rare, has a propensity for involving the aorta and central nervous system and can also cause "gummas"
(granulomatous-like lesions) in many sites, notably including liver and bone.
38
Herpes simplex I (choice A) usually causes perioral vesicular lesions.

Herpes simplex II (choice B) usually causes genital vesicular lesions.

HIV (choice C) does not itself cause a rash, although co-infection with other organisms can result in a rash.

Neisseria gonorrhoeae(choice D) does not typically cause a rash.

A child is brought to the pediatrician because of perianal itching, which has been disturbing the child's sleep.
Physical examination reveals scaly skin in the perianal region. The physician instructs the parents to place
transparent tape on the perianal region the next morning, and then return to the office. In the office, the tape is
transferred to a microscope slide, and low-power examination of the tape reveals oval eggs that are flattened
along one side. The microorganism most likely responsible for the child's symptoms is

A. Ancylostoma duodenale

B. Ascaris lumbricoides

C. Enterobius vermicularis

D. Necator americanus

E. Trichuris trichiura

Explanation:

The correct answer is C.Enterobius vermicularis (pinworm) inhabits the large intestine, and the gravid females
migrate at night into the perianal region to lay eggs, which can be identified by placing cellophane tape on the
perianal skin and then looking at the tape with a microscope. Enterobiasis is characterized by extreme pruritus
and is very contagious, transmitted by ingestion of the eggs of the organism. The infection is easily treated with
a single dose of mebendazole.

Ancylostoma duodenale(choice A) and Necator americanus(choice D) are hookworms, and typically cause
anemia.

Ascaris lumbricoides(choice B) is a large, intestinal roundworm that is very common, especially in the tropics.
Infection may be asymptomatic or can cause abdominal symptoms.

Trichuris trichiura(choice E) is the whipworm, which can cause rectal prolapse with heavy infestations.

One week following a visit to the woods along an Eastern seaboard beach, a 50-year-old woman develops fever,
headache, chills, and fatigue. A blood smear demonstrates protozoa within erythrocytes. Which of the following is
the most likely pathogen?

A. Babesia microti

B. Leishmania donovania

C. Plasmodium falciparum

D. Plasmodium vivax

E. Trypanosoma cruzi
39

Explanation:

The correct answer is A. The combination of a one week incubation period, the Eastern seaboard clue, and the
intra-erythrocyte parasites strongly suggest Babesia microti as the infecting organism. The clinical disease is
called babesiosis. The infecting protozoan is related to Plasmodium and is transmitted by the bite of the Ixodes
dammini tick. Occasional cases have been transmitted by blood transfusions. Most patients are asymptomatic;
symptomatic cases in reasonably healthy individuals have the features listed in the question stem. Rare severe
cases, which may be fatal, may develop in severely debilitated or asplenic individuals and can be accompanied
by severe hemolysis (up to 30% of RBCs may have the parasites) with subsequent hemoglobinuria, hemolysis,
and renal failure.

Leishmania donovania(choice B) is an intracellular tissue protozoan, not a blood protozoan.

Plasmodium falciparum(choice C) and Plasmodium vivax(choice D) are malarial parasites not encountered on
the Eastern seaboard.

Trypanosoma cruzi(choice E) is an intracellular tissue protozoan, and does not infect blood cells.

A patient is referred to a neurologist because of ataxia. The neurologist diagnoses degeneration of the dorsal
columns and dorsal roots of the spinal cord, which has caused impaired proprioception and locomotor ataxia.
Which of the following organisms most likely caused this pattern of damage?

A. Haemophilus influenzae

B. Herpes simplex I

C. Neisseria gonorrhoeae

D. Neisseria meningitidis

E. Treponema pallidum

Explanation:

The correct answer is E. The findings described are those of tabes dorsalis, a form of tertiary syphilis caused
by Treponema pallidum. Tabes dorsalis and other forms of tertiary syphilis are now uncommon in this country,
possibly because the common use of antibiotics may "treat" many unsuspected cases of syphilis.

Haemophilus influenzae(choice A) and Neisseria meningitidis(choice D) can cause meningitis.

Herpes simplex I (choice B) can cause an encephalitis that typically involves the frontal and temporal lobes.

Neisseria gonorrhoeae(choice C) causes gonorrhea, which usually does not involve the CNS.

Which of the following organisms would be most likely to cause an outbreak of enteritis in a day care center in the
United States?

A. Helicobacter jejuni

B. Salmonella typhi

C. Shigella species

D. Vibrio cholerae
40
E. Yersinia enterocolitica

Explanation:

The correct answer is E.Yersinia enterocolitica is an important cause of “mini-epidemics” of


pediatric diarrhea. Adults can also be affected, but less commonly than children. Some diarrheal cases are
severe (and occasionally fatal) and may be complicated by severe dysentery, appendicitis, or chronic relapsing
ileocolitis that may require antibiotics to shorten the course.

Helicobacter jejuni(choice A) is an important cause of hospital-acquired diarrhea.

Salmonella typhi(choice B) causes typhoid fever.

Shigella species (choice C) causes epidemics of dysentery in military camps.

Vibrio cholerae(choice D) causes cholera.

A 60-year-old woman has been hospitalized for three weeks with widely metastatic ovarian adenocarcinoma, and
she becomes septic with vancomycin-resistant enterococcus. What is the mechanism of vancomycin resistance in
this organism?

A. Acetylation of antibiotic

B. Altered drug-binding protein

C. β-lactamase production

D. Decreased ribosomal binding of antibiotic

E. Formation of novel cell wall peptide bridges

Explanation:

The correct answer is E. The antibiotic property of vancomycin depends upon its ability to bind D-ala-D-ala,
which is vital in the synthesis of peptidoglycan peptide bridges. Vancomycin-resistant enterococci instead utilize
D-lactic acid in their peptide bonds, and thus are resistant to the action of vancomycin.

Enzymatic deactivation of antibiotics is a common mode of resistance to aminoglycosides, chloramphenicol, and


sulfonamides. The most common chemical modifications are acetylation (choice A), adenylation, and
phosphorylation.

Penicillin that has penetrated the cell membrane is kept in place by binding to penicillin binding proteins (PBPs).
Penicillin resistance may be conferred through altered PBP affinity or number (choice B). Vancomycin
resistance does not depend on PBPs.

β-lactamase (choice C) is a bacterial enzyme that inactivates some of the penicillins and cephalosporins,
but has no activity on vancomycin.

Modified ribosomes with decreased antibiotic affinity (choice D) confer resistance to protein synthesis-inhibiting
antibiotics, including tetracycline, minocycline, doxycycline and the macrolide antibiotics. Vancomycin is not a
protein synthesis-inhibiting antibiotic.
41

A 2-year-old boy has surgery to correct a urinary tract obstruction. Post-operatively, with an indwelling urinary
catheter in place, he develops a urinary tract infection. Urine culture grows out a lactose-negative,
oxidase-positive, gram-negative rod. Which of the following agents is the most likely cause of this boy's urinary
tract infection?

A. Candida albicans

B. Enterococcus faecalis

C. Escherichia coli

D. Klebsiella pneumoniae

E. Proteus mirabilis

F. Pseudomonas aeruginosa

G. Staphylococcus saprophyticus

Explanation:

The correct answer is F.Pseudomonas aeruginosa is a gram-negative rod. It can easily be distinguished from
the family Enterobacteriaceae because Pseudomonas is oxidase positive. It is an opportunistic pathogen that
has an increased chance of causing urinary tract infections in patients with indwelling catheters, or who are on
antibiotics.

Candida albicans(choice A) is a yeast that can cause urinary tract infections in poorly controlled diabetics,
because glucose in the urine enhances its growth.

Enterococcus faecalis(choice B) is a gram-positive coccus that commonly causes urinary tract infections in
elderly men with prostate problems.

Escherichia coli(choice C) is a lactose-positive, oxidase-negative, gram-negative rod that is the most common
cause of community acquired urinary tract infections.

Klebsiella pneumoniae(choice D) is a lactose-positive, oxidase-negative, gram-negative rod that can cause


urinary tract infections in poorly controlled diabetics because glucose in the urine enhances its growth.

Proteus mirabilis(choice E) is a gram-negative rod that is a member of family Enterobacteriaceae. It is


lactose-negative, but contains urease, which splits urea to yield ammonia, which in turn, raises the pH of the
urine, creating a more hospitable environment for the organism. Patients with stones are at increased risk for
this organism because it is able to hide in the stones (conversely, Proteus predisposes for the development of
stones).

Staphylococcus saprophyticus(choice G) is a catalase-positive, coagulase-negative, gram-positive coccus that


causes urinary tract infections in young women.

A 65-year-old Laotian immigrant has developed thick, erythematous nodules on her ears and nose with
significant associated sensory loss. The nodules have grown slowly over the course of many years. Biopsy of the
lesions shows dermal granulomas with giant cells but no acid-fast bacteria. Culture on blood agar and
Lowenstein-Jensen medium shows no growth. These findings are most consistent with which of the following
diagnoses?

A. Cutaneous leishmaniasis
42
B. Onchocerciasis

C. Rhinoscleroma

D. Smallpox

E. Tuberculoid leprosy

Explanation:

The correct answer is E.Mycobacterium leprae is endemic to parts of Africa, Asia, and South America.
Tuberculoid leprosy is an indolent disease, typically affecting cooler parts of the body, such as the nose and
ears, producing dermal granulomas with very rare, acid-fast bacilli and damage to peripheral nerves. This is in
marked contrast to lepromatous leprosy, which is progressive and invasive and generally characterized by the
presence of numerous acid-fast bacteria in a histiocytic, but non-granulomatous tissue response. M. leprae has
not been grown in any culture medium.

Cutaneous leishmaniasis (choice A) is due to Leishmania species that show a worldwide distribution. The
infection is transmitted through a sandfly bite, and the skin lesion is typically ulcerated. Histology shows
intracellular parasites within the dermis and epidermis. Granulomas are not formed.

Onchocerciasis (choice B), or river blindness, is a roundworm infection transmitted by black flies of Africa and
South America. The microfilaria grow at the site of inoculation, and cause an inflamed subcutaneous nodule.
The organism is seen on tissue sections.

Rhinoscleroma (choice C) is a destructive granulomatous infection of the nasopharynx caused by Klebsiella


rhinoscleromatis. Gram-negative rods can be cultured from the lesions.

Smallpox (choice D), the infection caused by the variola virus (a DNA poxvirus), produces malaise, headaches,
and a macular/pustular rash involving the face and distal extremities. Smallpox has been eradicated through
worldwide vaccination.

Which of the following organisms would most likely be isolated from the vagina of a normal 5 year-old girl?

A. Candida

B. Lactobacillus

C. Neisseria

D. Staphylococcus

E. Streptococcus

Explanation:

The correct answer is D. The vagina of prepubertal girls and post-menopausal women is colonized by colonic
and skin bacteria, including Staphylococcus epidermidis, which is normally found on the skin.

The vagina of women of child-bearing age tends to be colonized by Lactobacillus (choice B) species, yeasts
such as Candida (choice A), and Streptococcus species (choice E).

The presence of Neisseria (choice C), such as N. gonorrhoeae (the cause of gonorrhea), in the vagina of a 5
year-old strongly suggests sexual abuse.
43
A 58-year-old homeless man is brought in to the emergency room by police. His temperature is 101.8°F rectally,
and he smells of cheap wine. Physical examination is remarkable for dullness to percussion, increased vocal
fremitus, and decreased breath sounds and wet crackles on the right. Chest x-ray reveals consolidation of the
right upper lobe. One of the nurses notes the production of thick, bloody sputum when the man coughs. The
most likely causative organism is

A. Haemophilus influenzae

B. Klebsiella pneumoniae

C. Pseudomonas aeruginosa

D. Staphylococcus aureus

E. Streptococcus pneumoniae

Explanation:

The correct answer is B. Although it is far better to rely on microbiology lab results than physical exam and
history, some classic clues for an infection with Klebsiella pneumoniae are a patient who is an elderly alcoholic
or diabetic, "currant-jelly" sputum (containing blood clots), and lobar pneumonia.

Hemophilus influenzae(choice A) is a frequent cause of community-acquired pneumonia, but does not


classically produce lobar pneumonia or currant-jelly sputum.

Pseudomonas aeruginosa(choice C) classically produces greenish sputum, and is associated with cystic fibrosis
rather than alcoholism.

Staphylococcus aureus(choice D) often produces pulmonary abscess, but may also produce an ordinary
bronchopneumonia.

Streptococcus pneumoniae(choice E) is a classic cause of lobar pneumonia, but does not produce currant-jelly
sputum or show a particular predilection for alcoholics.

An archaeologist who has been excavating a very old Middle Eastern site develops a pustule on his hand. The
pustule then ruptures to form a black eschar surrounded by expanding brawny edema. Which of the following
organisms is the most likely cause of this condition?

A. Bacillus anthracis

B. Borrelia burgdorferi

C. Francisella tularensis

D. Spirillium minus

E. Yersinia pestis

Explanation:

The correct answer is A. Anthrax forms extremely stable spores and has, in fact, been encountered in very old
(i.e., deep) dirt in Israeli and Arabian excavation sites. The causative organism, Bacillus anthracis, is found in
many animal species, and humans can acquire the organism either through contact with the animals or from
locally contaminated soil. The pustule described in the question stem is called a "malignant pustule" and may
be accompanied by lymphadenopathy. Most cases remain localized, but death can occur as the result of
complications such as bacteremia, meningitis, and pneumonia.
44

Borrelia burgdorferi(choice B) causes Lyme disease. In the first stage of this disease, there is a localized
expanding erythematous rash rather than an eschar. Furthermore, Lyme disease occurs in the U.S., Europe,
and Asia, corresponding to the distribution of Ixodid ticks, which spread the infection.

Francisella tularensis(choice C) causes tularemia. The organism can persist for weeks to months but does not
form spores that could survive for hundreds or thousands of years. Also, most human cases occur in the
endemic areas of the U.S.

Spirillium minus(choice D) is one of the causes of rat-bite fever. There is no indication the man was bitten by a
rat.

Yersinia pestis(choice E) causes bubonic plague. This organism does not form stable spores that could survive
for extended periods of time.

A 33-year-old woman who has recently returned from several years in the Peace Corps in Guatemala presents
with severe, acute, right upper quadrant abdominal pain. She recalls that before coming back to the United
States, she had several months of bloody diarrhea. CT scan of the liver demonstrates lesions that are interpreted
to be abscesses. Which of the following organisms is the most likely cause of her illness?

A. Ascaris lumbricoides

B. Entamoeba histolytica

C. Enterobius vermicularis

D. Salmonella typhi

E. Shigella species

Explanation:

The correct answer is B. The patient probably has hepatic amebiasis, which is a life-threatening complication of
intestinal infection with Entamoeba histolytica.E. histolytica is transmitted via the fecal-oral route. Intestinal
colonization (which may be asymptomatic) always precedes infection of the liver. The abscesses generally
contain necrotic debris, with amoebae located along the edges of the abscess.

Ascaris lumbricoides(choice A) can cause intestinal obstruction, but does not usually cause hepatic abscesses.

Enterobius vermicularis(choice C) is the pinworm, which inhabits the rectum.

Bloody diarrhea can also be seen with some strains of Salmonella (choice D) and Shigella (choice E), but these
organisms do not usually cause hepatic abscesses. Bacterial causes of hepatic abscesses include E. coli,
Klebsiella, Streptococcus, Staphylococcus, Bacteroides, and Pseudomonas.

A previously healthy 11-year-old girl develops a gastrointestinal infection with cramping and watery stools. After
several days, she begins to pass blood per rectum, and is hospitalized for dehydration. In the hospital, she is
noted to have decreasing urine output with rising blood urea nitrogen (BUN). Total blood count reveals anemia
and thrombocytopenia, and the peripheral smear is remarkable for fragmented red cells (schistocytes). Infection
with which of the following bacterial genera is most likely responsible for this syndrome?

A. Campylobacter
45
B. Clostridium

C. Salmonella

D. Shigella

E. Vibrio

Explanation:

The correct answer is D. This patient has developed hemolytic-uremic syndrome (HUS), a complication of the
Shiga toxin or Shiga-like toxin: exotoxins released by Shigella species and the enterohemorrhagic E.coli. HUS in
children usually develops after a gastrointestinal or flu-like illness, and is characterized by bleeding, oliguria,
hematuria and microangiopathic hemolytic anemia. Presumably the Shiga toxin is toxic to the microvasculature,
producing microthrombi that consume platelets and RBCs, and may fragment the red cell membrane.

The incorrect choices are all bacteria which may produce an enterocolitis, but do not elicit HUS.

A long-term consequence of Campylobacter (choice A) infection is a reactive arthritis or full-blown Reiter's


syndrome.

Clostridial enterocolitis is produced by Clostridium difficile(choice B), a normal inhabitant of the gut that
produces pseudomembranous colitis when other gut flora are suppressed by treatment with antibiotics.

In the United States, Salmonella infections (choice C) are almost all non-typhoid inflammatory diarrhea,
producing a simple enterocolitis that may proceed to sepsis in some cases. Typhoid fever (produced by
Salmonella typhi and S. paratyphi) produces a protracted illness that progresses over several weeks and
includes rash and very high fevers, but not HUS.

Vibrio (choice E) infections produce copious amounts of watery diarrhea, and the major risk of cholera and
other Vibrio enteritides is shock due to hypovolemia or electrolyte loss.

A 20-year-old man visiting Europe dives into a secluded natural pool. He later develops meningoencephalitis
localized to the base of his brain. The causative organism is found to be the free-living amoeba, Naegleria
fowleri. Which of the following structures did he probably damage during the dive?

A. Cribriform plate

B. Lacrimal bone

C. Mandible

D. Nasal bone

E. Zygomatic bone

Explanation:

The correct answer is A. The rare, but life-threatening, meningoencephalitis caused by Naegleria fowleri
apparently ensues when the organism enters the body through the nasal mucosa and crosses the cribriform
plate to enter the brain via the olfactory nerves. It has been speculated that trauma to the nose during diving
predisposes to this infection.

The lacrimal bone (choice B) of the medial aspect of the orbit does not appear to be involved in amoebic
infection.
46
The mouth and its associated bones, including the mandible (choice C), do not appear to be a route of infection
for this organism.

The nasal bones (choice D), located on the superior surface of the nose, do not appear to be a route of
infection for this organism.

The zygomatic bone (choice E), which forms the cheekbone, does not appear to be a route of infection for this
organism.

A 1-year-old girl presents with a 2-day history of fever, vomiting, and watery, nonbloody diarrhea. On physical
exam, she appears dehydrated. Which of the following best describes the most likely infecting organism?

A. It has a complex double-stranded DNA genome

B. It has a partially double-stranded circular DNA genome

C. It has a segmented, double-stranded RNA genome

D. It has a single-stranded circular RNA genome

E. It has a single-stranded RNA genome

Explanation:

The correct answer is C. Rotavirus is the most common cause of gastroenteritis in children between 3 months
and 2 years of age. It is most prevalent in the winter. Rotavirus, one of the reoviruses, looks like a wheel (which
ROTAtes) and possesses a double-shelled icosahedral capsid with no envelope. Its genome consists of 11
segments of double-stranded RNA.

A complex double-stranded DNA genome (choice A) is found in enteric adenoviruses, the third most common
cause of gastroenteritis in infants and children. This organism possesses an icosahedral nucleocapsid.

A partially double-stranded circular DNA genome (choice B) is characteristic of hepatitis B. Its envelope contains
surface antigen (HBsAg). Its capsid is icosahedral and contains the genome along with DNA-dependent DNA
polymerase, which also has reverse transcriptase activity.

A single-stranded circular RNA genome (choice D) is characteristic of hepatitis D virus. Its envelope consists of
HBsAg. The virus is defective and is able to replicate only in cells infected with hepatitis B.

A single-stranded RNA genome (choice E) is characteristic of several viruses that cause gastroenteritis in
children, including astrovirus and Norwalk virus. Astrovirus is the second most common cause of viral
gastroenteritis in young children. Three structural proteins form its capsid. Norwalk virus is the most common
cause of gastroenteritis outbreaks in older children and adults. Its capsid consists of one structural protein.

A 16-year-old male with sickle cell disease is hospitalized for a severe infection. His spleen has
autosplenectomized and he has suffered from other minor infections in the past. His symptoms include fever,
chills, cough, and chest pain. Bacteria from the patient's sputum yield optochin-sensitive organisms with a
positive Quellung reaction. The organism is

A. Escherichia coli

B. Haemophilus influenzae
47
C. Klebsiella pneumoniae

D. Neisseria gonorrhoeae

E. Streptococcus pneumoniae

Explanation:

The correct answer is E. The combination of optochin sensitivity and positive Quellung reaction are properties
of a single organism, Streptococcus pneumoniae. The other encapsulated organisms that have
Quellung-positive reactions are Haemophilus influenzae(choice B), Neisseria meningitidis, and Klebsiella
pneumoniae(choice C). However, none of these organisms are optochin sensitive.

The other choices, Escherichia coli(choice A) and Neisseria gonorrhoeae(choice D), are not encapsulated.

Which of the following is specifically associated with infection with Schistosoma haematobium?

A. Adenocarcinoma of the bladder

B. Adenocarcinoma of the renal pelvis

C. Squamous cell carcinoma of the bladder

D. Transitional cell carcinoma of the bladder

E. Transitional cell carcinoma of the renal pelvis

Explanation:

The correct answer is C. Carcinomas of the bladder and renal pelvis are usually transitional cell (choices D and
E) carcinomas. However, Schistosoma haematobium infection (where Schistosomes lay eggs in the veins near
the bladder, thereby inducing a marked inflammatory response) is associated with squamous metaplasia and
squamous cell carcinoma of the bladder. Some authors have suggested that medications used to kill the worms
may contribute to the etiology. Adenocarcinomas of the renal pelvis and bladder (choices A and B) are rare.

An active intravenous drug abuser presents to the emergency department with fever of 5 days' duration, a cough
occasionally productive of blood, and pleuritic chest pain. Petechiae are present in his mouth and conjunctivae,
and splinter hemorrhages are visible under the fingernails. Which of the following test results would most likely
confirm the identity of the causative agent?

A. Antibodies to p24 capsid antigen

B. Antibodies to Trichinella spiralis antigen

C. Blood culture of a catalase-positive, novobiocin-sensitive, gram-positive coccus

D. Blood culture of a coagulase-positive, catalase positive, gram-positive coccus

E. Blood culture of a gamma-hemolytic, gram-positive coccus on bile-esculin agar

F. Blood culture of an alpha-hemolytic, optochin-resistant, gram-positive coccus


48

G. Blood culture of an alpha-hemolytic, optochin-sensitive, gram-positive coccus

Explanation:

The correct answer is D. The patient is exhibiting the classic signs of acute bacterial endocarditis. In intravenous
drug abusers, Staphylococcus aureus is the most common causative agent, because it is the most common
normal flora organism on the skin of these patients, and choice D is the classic description of this species.
Catalase positivity distinguishes the genus Staphylococcus from Streptococcus, and Staphylococcus aureus is
the only coagulase positive member of its genus.

Antibodies to p24 capsid antigen (choice A) would be likely in an HIV-positive drug abuser, rather than a patient
with endocarditis.

Trichinella spiralis(choice B) can cause splinter hemorrhages during the larval migration period, but would not
be the most likely agent given the other symptoms.

Staphylococcus epidermidis is a catalase-positive, novobiocin-sensitive, gram-positive coccus (choice C). It is


sometimes implicated in subacute bacterial endocarditis, but is less common as a causative agent in drug
abusers than in normal individuals.

Enterococci are gamma-hemolytic, gram-positive cocci that grow on bile-esculin agar (choice E). Enterococci
can cause subacute bacterial endocarditis, chiefly after urologic instrumentation in men, rather than in
association with intravenous drug abuse.

Streptococcus viridans is an alpha-hemolytic, optochin-resistant, gram-positive coccus (choice F). It is implicated


in subacute bacterial endocarditis after oral or dental treatments but would not be the most likely agent, given
the acute nature of this patient's presentation.

Streptococcus pneumoniae is an alpha-hemolytic, optochin-sensitive, gram-positive coccus (choice G) that


produces cough and chest pain, but would be an unlikely cause of bacterial endocarditis.

A 22-year-old woman presents with a 1-week history of mild lower abdominal pain and a yellowish vaginal
discharge. She describes the pain as dull in nature, relieved slightly by acetaminophen and worsened by
intercourse. Pelvic examination reveals a red, swollen cervix without motion tenderness. The mucosa is friable.
Potassium hydroxide (KOH) mount is negative, and wet mount does not reveal clue cells. Gram's stain of the
exudate reveals gram-negative cocci. Which of the following procedures would most likely lead to the correct
diagnosis?

A. Culture the blood on Thayer-Martin agar

B. Order DNA probe assays of endocervical exudates

C. Order serologic tests to identify specific capsular antigens

D. Order the germ tube test

E. Order the rapid plasma reagin (RPR) test

Explanation:

The correct answer is B. The symptoms suggest infection with Neisseria gonorrhoeae; however, with Gram's
stain results alone, it is not possible to distinguish the gonococcus from normal flora organisms such as
Acinetobacter, unless gram-negative diplococci are found within polymorphonuclear leukocytes. The fastest
and most reliable assay specific for diagnosis of gonorrhea is the use of commercial DNA probes, with results
available in 2-4 hours.
49

Culture on Thayer-Martin medium (choice A) is indeed the choice for culture of N. gonorrhoeae, but would yield
slower results, and therefore be a secondary choice.

Serology (choice C) is not a good choice because N. gonorrhoeae does not have a significant capsule (the
meningococcus does), and furthermore, serological tests for the gonococcus have proven insensitive and
nonspecific.

The germ tube test (choice D) is one of the diagnostic tests for Candida albicans, but this case presentation is
not characteristic of candidiasis.

The RPR (choice E) is a test for reaginic (heterophilic) antibodies formed early in infection with Treponema
pallidum, but this case presentation is not suggestive of syphilis.

A patient develops fever, shortness of breath, and appears to be quite ill. X-ray demonstrates bilateral interstitial
lung infiltrates. Bronchial washings demonstrate small "hat-shape" organisms visible on silver stain within alveoli.
Which predisposing condition is most likely to be present in this patient?

A. AIDS

B. Congestive heart failure

C. Pulmonary embolus

D. Rheumatoid arthritis

E. Systemic lupus erythematosus

Explanation:

The correct answer is A. The disease is Pneumocystis pneumonia, which is caused by an agent now believed to
be a fungus rather than a true bacteria. Pneumocystis carinii pneumonia is seen in immunocompromised
patients, particularly in those with AIDS, cancer, and in malnourished children. It can be the AIDS-defining
illness.

Congestive heart failure (choice B) predisposes the patient to pulmonary edema.

Pulmonary embolus (choice C) can cause pulmonary infarction or sudden death.

Rheumatoid arthritis (choice D), particularly in miners, can cause formation of lung nodules similar to
subcutaneous rheumatoid nodules.

Systemic lupus erythematosus (choice E) can cause pleuritis, but is not associated with a significantly increased
incidence of pneumonia.

An Hispanic male is referred to the dermatology clinic of a major medical center. On physical examination, the
man has several disfiguring lesions on his face and there is loss of cutaneous sensation to fine touch, pain, and
temperature. An acid-fast organism is observed in scrapings from a skin lesion. Which of the following organisms
is the most likely cause of this patient's disease?

A. Bartonella henselae

B. Listeria monocytogenes

C. Mycobacterium avium-intracellulare
50

D. Mycobacterium leprae

E. Nocardia asteroides

Explanation:

The correct answer is D. The disease in question is leprosy, or Hansen's disease. A key feature in the
description is the fact that the organism is acid-fast. Both of the mycobacteria, M. avium-intracellulare and M.
leprae are strongly acid-fast, that is they retain the carbol fuchsin dye in the face of acid-alcohol decolorization.
M. leprae has a predilection for the skin and cutaneous nerves, thereby producing the symptoms of
depigmentation and anesthetic cutaneous lesions. This loss of peripheral nerve function leads to many of the
disfiguring features of the disease; because the patients do not have normal pain sensation, they sustain
repeated injuries. In addition, the organism attacks cartilage and causes granuloma formation in the skin,
leading to some of the facial disfigurement.

Bartonella henselae(choice A) is a very small, gram-negative bacterium that is closely related to the rickettsia,
although it is able to be cultured on lifeless media. It is the cause of cat-scratch disease, a local, chronic
lymphadenitis most commonly seen in children, and bacillary angiomatosis, a disease seen particularly in AIDS
patients.

Listeria monocytogenes(choice B) is a ubiquitous microbe that causes disease in over 100 animal species.
Although it is best known as an agent of meningitis in the newborn, it is a cause of multiple other diseases. A
characteristic feature of these infections is the development of granulomas at the site of the infection. The
organism is not acid-fast and has no particular predilection for skin or nervous tissues.

M. avium-intracellulare(choice C) causes tuberculosis-like pulmonary disease in the immunosuppressed.

Nocardia asteroides(choice E) primarily produces pulmonary infections in humans. The organism is consider to
be "weakly" acid-fast, meaning that if the amount of HCl used in the decolorization step is reduced, the
organisms will retain the carbolfuchsin primary stain.

A 23-year-old man develops explosive watery diarrhea with blood, fecal leukocytes, and mucus approximately 3
days after eating chicken that was improperly cooked. Comma-shaped organisms were found in the fecal smear
along with red blood cells and leukocytes. Which of the following pathogens is the most likely cause of these
symptoms?

A. Campylobacter jejuni

B. Enterotoxigenic E. coli

C. Shigella sonnei

D. Staphylococcus aureus

E. Vibrio cholera

Explanation:

The correct answer is A.Campylobacter jejuni is a pathogen causing an invasive enteric infection associated
with ingestion of raw or undercooked food products, or through direct contact with infected animals. In the U.S.,
ingestion of contaminated poultry that has not been sufficiently cooked is the most common means of acquiring
the infection. The patients typically have bloody diarrhea, abdominal pain, and fever. The presence of fecal
leukocytes indicates an invasive infection. The organism is a gram negative rod with a "comma-shape."
51
Enterotoxigenic E. coli(choice B) causes the classic traveler's diarrhea. The infection is non-invasive and is
acquired via the fecal-oral route through consumption of unbottled water or uncooked vegetables. The major
manifestation is a copious outpouring of fluid from the GI tract presenting as explosive diarrhea. This is due to
the action of one of two types of enterotoxins on the GI tract mucosa.

Shigella sonnei(choice C) produces a syndrome very similar to C. jejuni. However, the organism appears as a
gram-negative rod on Gram's stain. It does not have a comma shape. Transmission is from person to person
via the fecal-oral route. Infection requires a low infective dose since the organism is fairly resistant to gastric
acidity.

Staphylococcus aureus(choice D) produces food poisoning due to the ingestion of a pre-formed enterotoxin.
The organism is present in food that is high in salt content such as potato salad, custard, milk shakes, and
mayonnaise. The patient presents with nausea, vomiting, and abdominal pain, followed by diarrhea beginning
1-6 hours after ingestion of the enterotoxin.

Vibrio cholerae(choice E) produces a secretory diarrhea due to increases in cAMP in the intestinal cells. The
organism is not invasive. The patient presents with the sudden onset of painless, watery diarrhea that becomes
voluminous, followed by vomiting. The stool appears nonbilious, gray, and slightly cloudy with flecks of mucus,
no blood, and a sweet odor.

A 37-year-old, intravenous drug-abusing male presents with fever and chills. Blood cultures are positive for
Staphylococcus aureus. He develops central nervous system symptoms, and a cerebral abscess is suspected.
Which part of the brain is most often affected by septic emboli in patients with infective endocarditis?

A. Brainstem

B. Cerebellum

C. Frontal lobe

D. Occipital lobe

E. Parietal lobe

Explanation:

The correct answer is E. Embolization from infective endocarditis typically causes multiple, small parietal lobe
abscesses. This "factoid" is worth knowing because some patients with infective endocarditis present with what
clinically looks like multiple small "strokes", and their treatable cardiac disease may be completely unsuspected.

A 33-year-old male with AIDS and a history of shingles develops a severe, multifocal encephalitis. Therapy is
instituted with acyclovir, but the man dies on the fourth day of his hospital admission. Which of the following
viruses is the most likely cause of his encephalitis?

A. Cytomegalovirus

B. Herpes simplex type I

C. Herpes simplex type II

D. Herpes zoster-varicella

E. Measles virus
52

Explanation:

The correct answer is D. The specific clue to the cause of the severe encephalitis in this AIDS patient is the
history of shingles, due to reactivation of the herpes zoster-varicella virus. In otherwise healthy adults, the virus
(which is usually introduced to the body as a childhood case of chickenpox) remains dormant in a dorsal root
ganglion, only to reactivate in later life, causing a painful vesicular eruption that characteristically conforms to
the distribution of a single dermatome. In AIDS patients, the virus can cause a severe, multifocal encephalitis
that may be resistant to acyclovir therapy.

Cytomegalovirus (choice A) can cause disseminated disease (including brain infection) in AIDS patients, but is
less likely in this patient, given the past history of shingles.

Herpes simplex type I (choice B) and herpes simplex type II (choice C) can cause disseminated disease
(including brain infection) in AIDS patients, but is less likely in this patient, given the past history of shingles,
and the lack of response to acyclovir.

Measles virus (choice E) appears to be related to subacute sclerosing panencephalitis, but this condition is not
specifically increased in AIDS patients.

A 35-year-old male undergoes an appendectomy. Several days later, an abscess has formed at the surgical site. It
does not improve with administration of a cephalosporin, but does respond to nafcillin. The infecting organism most
likely produced an enzyme that would hydrolyze which bond in the above molecule?

A. A

B. B

C. C

D. D

Explanation:

The correct answer is D. Abscesses are often caused by Staphylococcus aureus, an organism that may produce
penicillinase, an enzyme that cleaves the amide bond of beta-lactam antibiotics (the molecule shown is penicillin).
The enzyme thus confers resistance to the beta-lactam antibiotics (penicillins and cephalosporins). Nafcillin (a
semisynthetic penicillin) is very effective against penicillinase-producing S. aureus.

You should be familiar with other important enzymes and toxins produced by this organism, including:
heat-resistant enterotoxin, toxic shock syndrome toxin, exfoliatin (causes scalded-skin syndrome in children),
alpha toxin (kills leukocytes), and coagulase.
53

A 10-year-old boy is attending summer camp in Texas. After 2 weeks of camp, he complains of a sore throat,
headache, cough, and malaise. On physical examination, he also has a low-grade fever and keratoconjunctivitis.
Within hours, several other campers and counselors visit the infirmary with similar symptoms. All of the patients
had been swimming in the camp swimming pool. Eventually, more than 50% of the camp complain of symptoms
similar to the initial case that last 5 to 7 days. Which of the following is the most likely causative organism?

A. Adenovirus

B. Chlamydia

C. Gram-negative diplococcus

D. Gram-positive enterococcus

E. Herpesvirus

Explanation:

The correct answer is A. Adenoviruses are non-enveloped (naked), icosahedral DNA viruses causing a variety
of clinical syndromes. Adenoviruses cause a pharyngoconjunctivitis that affects children and sometimes adults
who are living in the same household. Contaminated swimming pools have been implicated as sources for the
virus. The virus is latent in the lymphoepithelial tissue of the nasopharynx and other sites. Adenoviruses also
cause watery, non-bloody diarrhea.

Chlamydia spp. (choice B) produce a variety of clinical syndromes, including a sexually transmitted urethritis,
pelvic inflammatory disease, neonatal pneumonia and inclusion conjunctivitis, lymphogranuloma venereum,
adult interstitial pneumonia, and a zoonotic pneumonitis.

A gram-negative diplococcus (choice C) would be a Neisseria spp. or Moraxella. Neisseria meningitidis would be
the logical choice, since it is associated with outbreaks among children. However, the clinical syndrome caused
by infection with the meningococcus is one of a fulminating, progressive septicemia and/or meningitis with fever,
vascular collapse, and disseminated intravascular coagulation. N. gonorrhoeae does not cause a clinical
syndrome as described in the question. Moraxella catarrhalis causes pneumonia in patients with chronic
obstructive pulmonary disease.

Gram-positive enterococcus (choice D) would refer to Group D streptococcus. These organisms are associated
with endocarditis and genitourinary infections.

Herpesviruses (choice E) are large, enveloped DNA viruses with an icosahedral shape. Possible diagnoses for
this patient include Epstein-Barr virus, producing infectious mononucleosis, or cytomegalovirus, producing a
mononucleosis-like syndrome. The patients in the question did not have hepatosplenomegaly (characteristic of
mononucleosis), but did have keratoconjunctivitis. The organism causing the outbreak in the question also has
a higher level of infectivity and a short incubation time.

A 35-year-old sexually active male presents to his internist with a painless penile vesicle. Physical exam reveals
inguinal lymphadenopathy. The infecting organism is definitively diagnosed and is known to exist in distinct
extracellular and intracellular forms. Which of the following is the most likely pathogen?

A. Calymmatobacterium granulomatis

B. Chlamydia trachomatis

C. Haemophilus ducreyi
54

D. Neisseria gonorrhoeae

E. Treponema pallidum

Explanation:

The correct answer is B. This patient has lymphogranuloma venereum caused by Chlamydia trachomatis (type
L1, 2, or 3). Chlamydia exhibit distinct infectious and reproductive forms. The extracellular infectious form is
known as the elementary body (EB), which is incapable of reproduction. It attaches to the host cell and enters
through endocytosis. Once inside the cell, the EB is transformed into the reticulate body (RB) within the
endosome. The RB is capable of binary fission and divides within the endosome; fusion with other endosomes
occurs to form a single large inclusion. Eventually, the RBs undergo DNA condensation and disulfide bond
bridgings of the major outer membrane protein, forming EBs. The EBs are then released. Note that C.
trachomatis is responsible for several sexually or perinatally transmitted diseases, including ocular trachoma
(types A, B, and C), neonatal conjunctivitis, nongonococcal urethritis, cervicitis, and pelvic inflammatory disease
(types D-K).

Calymmatobacterium granulomatis(choice A) is a gram-negative rod that causes superficially ulcerated genital


or inguinal papules that coalesce to form substantial lesions. The appearance of Donovan bodies in histiocytes
is diagnostic of this infection.

Haemophilus ducreyi(choice C) is a gram-negative rod that causes a soft, painful penile chancre, unlike that of
a chlamydial or syphilitic lesion. This infection is common in the tropics.

Neisseria gonorrhoeae(choice D) is a gram-negative diplococcus responsible for gonorrhea. Patients typically


present with purulent penile discharge, not genital lesions.

Treponema pallidum(choice E) is the spirochete responsible for syphilis. It may cause a firm, painless ulcer as a
manifestation of primary syphilis, but the organism does not exist in distinct extracellular and intracellular forms
as does Chlamydia. Secondary syphilis is associated with the appearance of condyloma lata–flat, gray,
wart-like lesions.

A 45-year-old white male with a history of alcohol abuse and periodontal disease is brought to the emergency
room for a spiking fever and chills. Physical examination is significant for signs of lung consolidation. A chest
x-ray shows a cavity in the right lower lobe that has an air/fluid level. A transtracheal aspiration is performed and
the specimen is submitted to the laboratory for routine cultures and Gram's stain. Based upon the clinical
presentation, which of the following would be the most likely finding?

A. Anaerobic bacteria

B. Aspergillus fumigatus

C. Entamoeba histolytica

D. Staphylococcus aureus

E. Streptococcus pyogenes

Explanation:

The correct answer is A. Anaerobic bacteria derived from the oral flora in the clinical setting of periodontal
disease are the most common isolates from lung abscesses. Single lung abscesses are the most common
pattern, with the superior segment of a lower lobe or the posterior segment of an upper lobe being affected
most often.
55
Aspergillus fumigatus(choice B) presents in the lung as hemorrhagic infarctions, aspergillomas (fungus balls) in
cavitary tuberculosis cavities, or as allergic bronchopulmonary aspergillosis.

Entamoeba histolytica(choice C) is associated with pulmonary abscesses as an extension of a liver abscess


across the diaphragm.

Staphylococcus aureus(choice D) usually presents as multiple lung lesions in non-contiguous sites since the
spread is embolic. The source of the infection is usually tricuspid endocarditis in IV drug abusers.

Streptococcus pyogenes(choice E) typically produces a bronchopneumonia pattern following an upper


respiratory infection.

A traveler in Bogota, Colombia drinks a glass of fruit juice with ice cubes made from tap water. E. coli
contaminating the water supply grow in the traveler's intestine and synthesize a protein that causes his intestinal
epithelium to overproduce cyclic AMP, resulting in a watery diarrhea. This syndrome is typical of which of the
pathogenic strains of E. coli?

A. Enteroaggregative

B. Enterohemorrhagic

C. Enteroinvasive

D. Enteropathogenic

E. Enterotoxigenic

Explanation:

The correct answer is E. Enterotoxigenic E. coli (ETEC), an important cause of traveler's diarrhea, produces a
toxin that activates intestinal adenylate or guanylate cyclase. Consequently, the intestinal mucosa
overexpresses cAMP, resulting in a mild and self-limited secretory diarrhea.

Enteroaggregative E. coli (EAEC; choice A) does not express a toxin, but is seen to cluster over the colonic
mucosa in some individuals. Although occasionally found in patients with chronic diarrhea, no clear mechanism
for mucosal pathophysiology has been determined for EAEC.

Enterohemorrhagic E. coli (EHEC; choice B), classically associated with strain O157:H7 and present in
undercooked hamburgers, expresses a Shiga-like toxin that causes bloody diarrhea and hemolytic-uremic
syndrome.

Enteroinvasive E. coli (EIEC; choice C) does not express a toxin, but rather is pathogenic through its capacity to
invade the colonic mucosa and evoke an inflammatory response. The resulting dysentery-like inflammatory
diarrhea generally necessitates vigilant hydration, but no antibiotics are indicated.

Enteropathogenic E. coli (EPEC; choice D) does not produce any known toxins, but adheres tightly to the
glycocalyx of the colonic mucosa and disrupts the microvilli. Villous atrophy, mucosal thinning, and inflammation
in the lamina propria, are produced, resulting in impaired absorption and diarrhea.

Zygomycosis, a destructive fungal infection of the sinuses, is likely to reach the brain by which of the following
routes?

A. Cavernous sinus
56
B. External carotid artery

C. Internal carotid artery

D. Superior sagittal sinus

E. Superior vena cava

Explanation:

The correct answer is A. The cavernous sinuses are located on either side of the body of the sphenoid bone,
and become a potential route of infection because they receive blood both from the face (via the ophthalmic
veins and sphenoparietal sinus) and some of the cerebral veins. The spread of infection, especially by Mucor
sp., into the cavernous sinus, can produce either CNS infection or cavernous sinus thrombosis, both of which
are potentially fatal.

The route from the face to the brain is not arterial (choices B and C).

The superior sagittal sinus (choice D) is located in the falx cerebri, and drains venous blood from the brain to
other dural sinuses, from which it eventually drains into the jugular vein. Zygomycosis does not reach the brain
by way of the superior sagittal sinus.

The superior vena cava (choice E) drains blood from the upper part of the body into the heart.

A farmer's wife develops abdominal pain and diarrhea, followed several days later by fever, periorbital edema,
eosinophilia, and myalgia. She does not remember eating anything unusual recently, but notes that she does
make her own pork sausage. Which of the following techniques would be most helpful for the diagnosis of this
patient?

A. Gastric biopsy

B. Muscle biopsy

C. Scotch tape test

D. Stool for ova

E. Stool for protozoal parasites

Explanation:

The correct answer is B. The patient probably has trichinosis; the diagnosis is confirmed by demonstration of
cysts in a muscle biopsy. Trichinosis is uncommon in this country because the organisms are usually found in
pork and can be killed by adequate cooking. Cases in the United States are usually due to tasting of raw pork
sausage (for seasoning purposes) or ingestion of poorly cooked bear meat.

The organism is not usually demonstrated in stool (choices D and E) or gastric biopsies (choice A).

The scotch tape test (choice C) is for Enterobius (pinworms).

A 70-year-old man with a history of prostate cancer presents with a chief complaint of pain on the right side of his
chest for the past several days. He has been receiving external beam radiation to spinal metastases of his
prostate cancer for the past several weeks. On examination, there is marked tenderness along the right side of
the chest wall in a 4-6 cm stripe from the midline to the flank. Multiple small vesicular lesions are visible in this
57
area on an erythematous base. Some of the lesions are fluid-filled, and some are crusted. How do members of
the virus family responsible for his condition produce messenger RNA?

A. By direct translation from the genome

B. By producing a double-stranded DNA intermediate

C. By producing a negative sense intermediate

D. By producing a positive sense intermediate

E. By transcribing the genomic DNA

F. By transcription from proviral DNA

G. The genomic RNA is used directly on the ribosome

Explanation:

The correct answer is E. This is a classic case of shingles, a stress-activated secondary disease caused by
reactivation herpes zoster. Herpesviruses are DNA viruses that use the mechanisms used by our own cells to
transcribe an RNA strand from their genomic DNA and use the transcribed RNA as messenger RNA.

Positive RNA viruses use direct translation from the genome (choice A) to make protein, not messenger RNA.

Retroviruses produce a double-stranded DNA intermediate (choice B) to effect genomic duplication.

Positive sense RNA viruses produce a negative sense intermediate (choice C) to produce a genomic duplicate.

Negative sense RNA viruses produce a positive sense intermediate (choice D) to produce messenger RNA and
a genomic duplicate.

Transcription from proviral DNA (choice F) is used by the retroviruses to make messenger RNA.

Genomic RNA used directly on the ribosomes (choice G) characterizes the production of messenger RNA by
positive sense RNA viruses.

A 66-year-old man with urinary retention secondary to prostatic hyperplasia develops a spiking fever and
tachypnea. Physical exam reveals intercostal muscle retractions and bilateral inspiratory crackles. A chest x-ray
exhibits bilateral interstitial and alveolar infiltrates. Arterial blood gases demonstrate severe hypoxemia. Blood
cultures would most likely reveal

A. gram-negative diplococci

B. gram-negative rods

C. gram-positive cocci

D. gram-positive diplococci

E. gram-positive rods

Explanation:

The correct answer is B.Escherichia coli is a very common pathogen associated with urinary tract infections and
is a common cause of cystitis. It is part of the normal flora of the GI tract. Patients with cystitis can develop
58
bacteremia and subsequent septic shock and adult respiratory distress syndrome (ARDS). The patient
described in the history has an obstructive lesion of the urinary tract that predisposes him to the overgrowth of
microorganisms such as E. coli. Pulmonary symptoms are consistent with ARDS. E. coli is a gram-negative rod.

Gram-negative diplococci (choice A) might be Neisseria spp. or Moraxella catarrhalis. Neisseria gonorrhea is
the cause of gonorrhea, a sexually transmitted disease that presents with urethritis, or may be asymptomatic.
Patients are usually younger, sexually active males. Moraxella spp. and Kingella kingae are gram-negative
cocci that can cause a wide variety of infections. Moraxella catarrhalis is usually implicated as a cause of otitis
media and sinusitis in children, or as a cause of purulent tracheobronchitis and pneumonia in a population of
patients who are over 50 and have underlying obstructive lung disease.

Gram-positive cocci (choice C), such as Staphylococcus aureus and S. epidermidis and Streptococci, rarely
cause cystitis. Staphylococcus saprophyticus causes urinary tract infections, but the patients are typically
young, sexually active women.

Gram-positive diplococci (choice D) would be a description of Streptococcus pneumoniae, which is the most
common cause of community-acquired pneumonia.

Gram-positive rods (choice E) would include members of the following genera: Clostridium, Bacillus, Listeria,
and the coryneform bacteria. The only significant member of this group to produce urinary tract infections is
Corynebacterium urealyticum. (C. jeikeium). The organism creates an alkaline urine environment with the
potential for stone formation. Patients are usually immunocompromised or have had recurrent urinary tract
infections.

A sexually active 25-year-old man develops epididymitis and orchitis. Needle biopsy demonstrates a prominent
leukocytic infiltrate with numerous neutrophils. Which of the following organisms is the most likely cause of this
man's infection?

A. Escherichia coli

B. Mycobacterium tuberculosis

C. Neisseria gonorrhoeae

D. Pseudomonas sp.

E. Treponema pallidum

Explanation:

The correct answer is C. Acute epididymitis and orchitis with prominent neutrophils in a sexually active male are
most likely due to infection with Neisseria gonorrhoeae or Chlamydia trachomatis. N. gonorrhoeae can produce
a nonspecific pattern of acute inflammation (nonspecific epididymitis and orchitis) or can be sufficiently severe
as to cause frank abscesses within the epididymis.

Escherichia coli(choice A) is an important cause of nonspecific epididymitis and orchitis in children with
congenital genitourinary abnormalities and in older men.

Mycobacterium tuberculosis(choice B) can cause tuberculosis of the epididymis and testes, characterized by
granuloma formation.

Pseudomonas sp.(choice D) has been implicated as an important cause of nonspecific epididymitis and orchitis
in older men.

Treponema pallidum(choice E), the causative agent of syphilis, can cause testicular involvement with gumma
formation, endarteritis, and/or a prominent plasma cell infiltrate.
59

Which of the following organisms is the most common cause of community-acquired pneumonia?

A. Chlamydia pneumoniae

B. Haemophilus influenzae

C. Mycoplasma pneumoniae

D. Staphylococcus aureus

E. Streptococcus pneumoniae

Explanation:

The correct answer is E. The most common bacteria implicated in community-acquired pneumonia is the
pneumococcus, Streptococcus pneumoniae. Other organisms frequently implicated in patients less than age 60
without comorbidity include Mycoplasma pneumoniae, respiratory viruses, Chlamydia pneumoniae, and
Haemophilus influenzae. When community-acquired pneumonia occurs in elderly patients or patients with
comorbidity, aerobic gram-negative bacilli and Staphylococcus aureus are added to the list.

The organisms listed in choices A, B, and C are important causes of community-acquired pneumonia, but are
not the most frequent causes.

Staphylococcus aureus(choice D) is an important cause of community-acquired pneumonia (particularly in the


elderly and in patients with comorbidity), but is not the most frequent cause.

A patient with a history of chronic diarrhea and bloody stools presents to the emergency department with right
upper quadrant pain and fever. Physical examination demonstrates a large, tender liver. Which of the following
would be most likely to have caused the patient's problems?

A. Cryptosporidium parvum

B. Entamoeba histolytica

C. Giardia lamblia

D. Isospora belli

E. Trichomonas vaginalis

Explanation:

The correct answer is B.Entamoeba histolytica is an intestinal amoebic parasite that either can cause relatively
mild diarrhea, or can behave more aggressively, causing dysentery (bloody diarrhea with abdominal pain and
dehydration), peritonitis, or liver abscess formation (such as this patient has). Treatment is with metronidazole.

Cryptosporidium parvum(choice A), Giardia lamblia(choice C), and Isospora belli(choice D) can cause chronic
diarrhea, but would not be expected to cause a liver abscess.

Trichomonas vaginalis(choice E) is not a cause of diarrhea, but instead causes vaginitis.


60

A neonate is born in very poor condition, with a severe, generalized encephalitis. Which of the following viruses is
the most likely pathogen in this setting?

A. Eastern equine encephalitis virus

B. Herpes simplex type II

C. Herpes zoster-varicella virus

D. Poliomyelitis virus

E. St. Louis encephalitis virus

Explanation:

The correct answer is B. Viral causes of neonatal encephalitis include three members of the herpes family of
viruses: herpes simplex I, herpes simplex II, and cytomegalovirus. All three types can have devastating effects
on the neonate, with extensive CNS damage leading to mental retardation, seizures, and focal neurologic
problems. Acyclovir may be of some help in modifying these infections, but both treatment and prognosis
remain very problematic.

Eastern equine encephalitis virus (choice A) and St. Louis encephalitis virus (choice E) are causes of epidemic
encephalitis but are not the most likely cause of neonatal encephalitis.

Herpes zoster-varicella virus (choice C), unlike herpes simplex, is not usually a cause of neonatal encephalitis.

Poliomyelitis virus (choice D) is a gastrointestinally transmitted virus that is not usually encountered in neonates
(or anyone else in the U.S. currently).

A 15-year-old high school student and several of her friends ate lunch at a local Chinese restaurant. They all
were served the daily luncheon special, which consisted of sweet and sour pork with vegetables and fried rice. All
of the girls developed nausea, vomiting, abdominal pain, and diarrhea within 6 hours of eating lunch. Which of the
following is the most likely cause of these symptoms?

A. Bacillus cereus

B. Clostridium botulinum

C. Clostridium perfringens

D. EHEC (Enterohemorrhagic E. coli)

E. Staphylococcus aureus

F. Vibrio cholerae

Explanation:

The correct answer is A.Bacillus cereus produces a self-limited diarrhea due to ingestion of the preformed
enterotoxin in contaminated fried rice and seafood. The incubation period is typically around 4 hours. The
degree of vomiting is greater than the diarrhea. B. cereus is also associated with keratitis, producing a corneal
ring abscess.

Clostridium botulinum(choice B) produces a neurotoxin that blocks the release of acetylcholine, resulting in a
61
symmetric descending paralysis that may lead to respiratory complications causing death. Symptoms include
blurred vision, photophobia, dysphagia, nausea, vomiting, and dysphonia. Most cases are associated with the
ingestion of contaminated home-canned food.

Clostridium perfringens(choice C) produces a severe diarrhea with abdominal pain and cramping (sometimes
called "church picnic" diarrhea). The incubation period is 8-24 hours after ingesting contaminated meat, meat
products, or poultry. The meats have usually been cooked, allowed to cool, and then warmed, which causes
germination of the clostridial spores.

EHEC-Enterohemorrhagic E. coli(choice D), produces a bloody, non-invasive diarrhea due to the ingestion of
verotoxin found in undercooked hamburger at fast food restaurants. The 0157:H7 serotype typically produces
this syndrome. Some patients develop a life-threatening complication called hemolytic-uremic syndrome.

Staphylococcus aureus(choice E) produces a self-limited food poisoning syndrome with nausea, vomiting, and
abdominal pain followed by diarrhea beginning 1-6 hours after ingestion of the enterotoxin. The organism is
found in foods such as potato salad, custard, milk shakes, and mayonnaise.

Vibrio cholerae(choice F) typically produces a watery, non-bloody diarrhea with flecks of mucus (rice-water
stools). Abdominal pain is not a feature. Massive fluid loss and electrolyte imbalance are complications. In the
U.S., cases of cholera (El Tor 01 strain) are associated with the Gulf coast and ingestion of poorly cooked or
poorly stored crabs, shrimp, or oysters. A strain of V. cholerae called non-01 is also found along the Gulf coast.
Patients who ingest contaminated shellfish experience fever, copious watery diarrhea, and abdominal cramps
within 48 hours after eating.

A 29-year-old news correspondent returns from covering an earthquake and its aftermath in a third world
country. The man feels tired and has sore muscles, so he consults a physician, who examines him, but decides
not to admit him to the hospital. The man subsequently develops a disabling illness complicated by severe
anemia, pulmonary edema, renal failure, and shock. Which of the following organisms is the most likely
pathogen?

A. Babesia microti

B. Plasmodium falciparum

C. Plasmodium malariae

D. Plasmodium ovale

E. Plasmodium vivax

Explanation:

The correct answer is B. Most forms of malaria cause chronic disease with significant morbidity but very little
mortality. The exception is malaria caused by Plasmodium falciparum (sometimes still called malignant tertian
malaria), which has a propensity for being severe and having life-threatening complications. Falciparum malaria
may have very severe anemia with hemoglobin less than 5 g/dL and more than half of the erythrocytes bearing
parasites. Complications of this form of malaria include pulmonary edema, renal failure with or without
hemoglobinuria, shock, hypoglycemia, and cerebral malaria. Episodes occur with an irregular periodicity with
fever spikes at 48-hour intervals during symptomatic periods.

Babesia microti(choice A) is an intraerythrocytic parasite that causes relatively mild, self-limited disease called
babesiosis.

Plasmodium malariae(choice C) causes relatively mild malaria characterized by fever spikes with a 72-hour
periodicity.

Plasmodium ovale(choice D) and Plasmodium vivax(choice E) cause relatively mild malaria characterized by
fever spikes with a 48-hour periodicity.
62

A 9-month-old infant is brought to the Health Department to receive the second dose of OPV (oral polio vaccine)
2 weeks after the first vaccination. The child has mild diarrhea, so the decision is made to defer further
immunizations. Bacteriologic examination of a stool culture is unremarkable; however, a small, single-stranded,
positive RNA virus is isolated from the specimen. This same agent was isolated from sewage effluent the
preceding week. The viral isolate was not inactivated by ether. Which of the following viruses was most likely
isolated?

A. Adenovirus

B. Hepatitis C

C. Parvovirus B19

D. Poliovirus

E. Rotavirus

Explanation:

The correct answer is D. Poliovirus, which is a single-stranded +RNA virus, is naked (i.e., non-enveloped) and
hence will not be inactivated by lipid solvents such as ether. The live virus vaccine had colonized the intestinal
tract of the infant and was still being shed 2 weeks after the earlier oral dose. This same virus, the vaccine
strain, is likely to be found in sewage, as all vaccinated infants will shed virus for a period of time after
immunization with OPV.

Adenoviruses (choice A) and parvovirus B19 (choice C) also may cause diarrheal disease and both are
non-enveloped; however, they both have a DNA genome.

Hepatitis C (choice B) is an enveloped, single-stranded +RNA virus; its major target organ is the liver, not the
intestinal tract. It is a fragile agent that does not survive well outside the body and would not be isolated from
raw sewage effluent.

Rotavirus (choice E) is the major cause of diarrheal disease in infants under the age of 2 years. It is a member
of the reovirus family and, as such, is double-stranded. This virus causes hospitalization of 30% to 40% of the
infected infants and kills hundreds of thousands of infants in developing nations where access to hospitals is
not readily available. Therapy for the watery diarrhea produced by this agent is fluid and electrolyte
replacement.

A 24-year-old male Asian immigrant presents with an ulcerative genital lesion. The lesion first appeared 1 month
ago as a papule with an erythematous base, which eventually became ulcerated and painful. On physical
examination, the man is afebrile. A tender ulcerative lesion is present on his prepuce, and inguinal adenopathy is
evident.

Which of the following would be the most likely microscopic finding in a scraping from the rash?

A. Epithelial cells with intranuclear inclusion bodies

B. Iodine-staining intracellular inclusion bodies

C. Koilocytotic squamous epithelial cells

D. Neutrophils containing gram-negative diplococci

E. Pleomorphic gram-negative rods in a "school of fish" pattern

F. Spirochetes visible by darkfield microscopy


63

Explanation:

The correct answer is E. This is a typical case description of chancroid, caused by Haemophilus ducreyi, a
pleomorphic gram-negative rod that displays a characteristic pattern on Gram's stained slides.

Epithelial cells with intranuclear inclusion bodies (choice A) would be found with herpes simplex infections, but
these lesions would not have the appearance described in this case history.

Iodine-staining intracellular inclusion bodies (choice B) would be found with genital lesions of Chlamydia
trachomatis, the causative agent of lymphogranuloma venereum, but this lesion is generally nonpainful.

Koilocytotic squamous epithelial cells (choice C) would be found in infections with human papilloma virus (HPV),
which is associated with venereal warts.

Neutrophils containing gram-negative diplococci (choice D) would be found if this were gonorrhea, but the case
symptoms are not consistent with this disease.

Spirochetes (choice F) would be found on darkfield microscopy if this were a case of syphilitic chancre, but that
chancre would be hard and nontender.

A burn patient at the university hospital has been progressively deteriorating. He was catheterized for several
days and developed a severe pneumonia, for which he was intubated and is now ventilator-dependent. A
gram-negative, non-fermenting rod is isolated from his sputum. It produces a blue-green pigment on growth
media and has a grape-like fruity odor. The organism most likely isolated is

A. Escherichia coli

B. Klebsiella pneumoniae

C. Legionella pneumophila

D. Pseudomonas aeruginosa

E. Serratia marcescens

Explanation:

The correct answer is D. Pseudomonas aeruginosa is a very common opportunist in burn patients, in whom it
classically causes secondary wound infections and septicemia. It may also cause cystitis in patients with urinary
catheters and pneumonia in patients with cystic fibrosis. The organism is found in water and usually gains
access to the body as a contaminant in the water used in respirators or in water baths used to cleanse wounds.
This organism is a non-fermenter, that is, it does not metabolize sugars by classic pathways. It produces a
blue-green, water-soluble pigment (pyocyanin), and has a fruity odor when grown on laboratory media.

Escherichia coli(choice A) is a lactose-fermenting, gram-negative rod commonly seen as normal flora of the
intestine of man. It is the most common cause of urinary bladder infections, pyelonephritis, and sepsis in
patients with indwelling urinary catheters. It is also the major cause of traveler's diarrhea and is a very important
pathogen in neonates who become infected during passage through the birth canal.

Klebsiella pneumoniae(choice B) is a gram-negative, highly encapsulated rod that is a significant pulmonary


pathogen in individuals with a respiratory compromise. It is a common cause of aspiration pneumonia and
pulmonary abscesses in alcoholics and patients with chronic obstructive pulmonary disease. The organism is
readily grown on standard laboratory media such as blood agar or MacConkey's enteric agar.
64
Legionella pneumophila(choice C) is a fastidious, gram- negative respiratory pathogen that may cause either a
fulminating disease or a mild "walking pneumonia-like" condition (i.e., an atypical pneumonia). The organism
can be cultured on a charcoal yeast extract medium, but identification is usually accomplished by
immunofluorescent examination of the pulmonary specimen.

Serratia marcescens(choice E) is a gram-negative organism that is found in water, soil, and as an occasional
normal flora of humans. It is an opportunistic pathogen that causes respiratory disease in hospitalized patients.
Many strains produce a pigment, but the colonies are usually red, pink, or orange. Interestingly, the pathogenic
varieties are most often non-pigmented.

A 57-year-old fisherman with a history of alcoholism is hospitalized in Gulfport, Mississippi with a 1-day history of
severe, watery diarrhea after eating several raw oysters. He is badly dehydrated on admission, and within 12
hours, he becomes severely hypotensive and dies. Which of the following pathogens is the most likely cause of
this man's death?

A. Citrobacter diversus

B. Enterotoxigenic E. coli

C. Providencia stuartii

D. Vibrio cholerae

E. Vibrio vulnificus

Explanation:

The correct answer is E.Vibrio vulnificus is an extremely invasive organism, producing a septicemia in patients
after eating raw shellfish, or causing wound infections, cellulitis, fasciitis, and myositis after exposure to
seawater or after cleaning shellfish. Patients at high risk for septicemia include those with liver disease,
congestive heart failure, diabetes mellitus, renal failure, hemochromatosis, and immunosuppression.

Citrobacter diversus(choice A) produces neonatal meningitis and can be frequently cultured from the umbilicus.

Enterotoxigenic E. coli(choice B) produces the classic traveler's diarrhea. The toxin is ingested in water and
salads. The incubation period is approximately 12 hours. The diarrhea is non-inflammatory and treatment is
supportive.

Providencia stuartii(choice C) is a gram-negative rod related to Proteus. It is a common cause of nosocomial


bacteremia in nursing home patients with chronic catheterization.

Vibrio cholerae(choice D) produces a non-invasive, non-inflammatory, high-volume secretory diarrhea that is


toxin-mediated.

A 38-year-old AIDS patient presents to his physician's office in Kansas City, Missouri, complaining of fever for the
past week and an increasing headache. He also states that sunlight hurts his eyes and that he has been feeling
nauseated and weak. His past medical history is significant for Pneumocystis pneumonia and a total CD4 count of
89. Current medications are trimethoprim/sulfa and indinavir. Cerebrospinal fluid (CSF) reveals 4 WBC, and
budding encapsulated yeast forms grow on Sabouraud's agar. Which of the following is an accurate description
of the morphology of the infectious form of the organism responsible for the man's illness?

A. Broad-based, budding yeasts

B. Budding yeasts in a "pilot's wheel" arrangement


65
C. Cylindrical arthroconidia

D. Encapsulated budding yeasts

E. Filamentous molds

F. Septate hyphae with microconidia and macroconidia

Explanation:

The correct answer is D. This is a classic case of Cryptococcus neoformans meningitis. Clues included the
patient population (HIV positive), geographic area (Mississippi and Missouri river beds), and diagnostic form
(encapsulated yeast). Cryptococcus is a monomorphic fungus, unlike many of the classic pathogens within the
fungal group, so the encapsulated yeast form would be found both in clinical specimens and in the environment
as the infectious form.

Broad-based, budding yeasts (choice A) would be the forms expected to be found in clinical specimens (not
environmental forms) from patients infected with Blastomyces dermatitidis, which is far more likely to present
with skin and bone lesions than with meningitis.

Budding yeasts in a "pilot's wheel" arrangement (choice B) would be the form expected to be found in clinical
specimens (not environmental forms) from patients infected with Paracoccidioides brasiliensis, which is limited in
geographic region to Central and South America and typically presents as a primary pulmonary disease.

Cylindrical arthroconidia (choice C) would be the transmission forms characteristic of Coccidioides immitis,
which may be a cause of fungal meningitis in immunologically compromised individuals, but is geographically
restricted to the sub-Sonoran desert zone of the U.S. (San Joaquin valley fever).

Filamentous molds (choice E) are the transmission forms of several of the fungal agents (eg, Coccidioides,
Blastomyces, Paracoccidioides) but Cryptococcus does not have a filamentous form.

Septate hyphae with microconidia and macroconidia (choice F) are the transmission (environmental) forms of
Histoplasma capsulatum, which is not an encapsulated yeast in spite of its name. It is primarily a pulmonary
infection acquired by exposure to the droppings of birds or bats.

An 8-year-old boy is taken to a pediatrician because of behavioral changes, mild intellectual deterioration, and
"laziness." Over the next several months the boy develops increasing clumsiness and periodic, involuntary, jerky
movements every 3 to 6 seconds. Visual deterioration is apparent upon visual field testing, and optic atrophy is
evident on funduscopic examination. Cerebrospinal fluid studies do not show significant pleocytosis, but
oligoclonal bands of IgG are present on CSF electrophoresis. The electroencephalogram shows periodic
discharges that are synchronous with the periods of myoclonus. Computed tomography (CT) of the head shows
low-density white matter lesions and cerebral atrophy. At the age of 10, the boy dies. Prior infection with which of
the following agents was probably related to the patient's condition?

A. Measles virus

B. Mumps virus

C. Papilloma virus

D. Poliovirus

E. Varicella

Explanation:

The correct answer is A. The rare disease illustrated is subacute sclerosing panencephalitis (SSPE), which
66
typically presents as in the question stem. SSPE appears to be due to a combination of persistent, possibly
abnormal measles virus and to autoimmune damage caused by antibodies directed against the virus. Many
patients developing SSPE have had measles at 2 years of age or younger; there is typically a six-year interval
between measles infection and symptom development. A small proportion of cases have followed vaccination
with live measles virus. Unfortunately, no effective therapy has been developed, although some experimental
work with drugs such as isoprinosine shows some promise.

Post-infectious encephalomyelitis, rather than SSPE, can follow mumps (choice B) or varicella (choice E). Latent
infection with the varicella virus (choice E) causes shingles (herpes zoster).

Papilloma viruses (choice C) are associated with warts. They do not usually infect the brain.

Poliovirus (choice D) causes gastrointestinal disturbances, viremia, and paralysis.

A 57-year-old man presents with an episode of shaking chills the previous night. He has now developed
right-sided pleuritic chest pain, fever, sweats, malaise, purulent sputum, and mild hemoptysis. On examination,
the patient is diaphoretic but alert, with right basilar rales. Chest X-ray films show a right lower lobe infiltrate with
blunting of the right costophrenic angle. Why is this patient's sputum filled with pus?

A. Teichoic acids and peptidoglycan are chemotactic for neutrophils

B. The capsule of the causative agent is chemotactic for neutrophils

C. The causative agent is an intracellular organism

D. The causative agent is beta hemolytic

E. The organism produces an IgA protease

Explanation:

The correct answer is A. The answer to this question requires that the student realize that pus consists of
bacteria and dead and dying neutrophils. This fact, taken along with the highly characteristic case history,
reflects that the patient has a typical pneumonia. In the United States, the most common agent of this would be
Streptococcus pneumoniae, a gram-positive extracellular pathogen rich in teichoic acids and peptidoglycan,
which elicit the neutrophilic exudate.

The capsule of this organism is a polysaccharide and primarily elicits an antibody response rather than
attracting neutrophils (choice B).

Streptococcus pneumoniae is an extracellular, not an intracellular (choice C) organism.

Streptococcus pneumoniae is alpha hemolytic, not beta hemolytic (choice D).

Streptococcus pneumoniae does produce an IgA protease (choice E) that enhances the ability of the organism
to infect the respiratory mucosa, but this does not contribute to pus formation.

A 48-year-old waitress presents to a physician with malaise, loss of appetite, nausea, moderate fever, and
jaundice. Laboratory tests indicate a marked increase in serum transaminases. Serology for hepatitis viruses is
performed and indicates positive results for the presence of HBsAg, HBc IgM antibody, and HCV antibody.
Antibody tests for HBsAb and HAV are negative. The results indicate:

A. A dual infection of HBV and HAV

B. Chronic hepatitis A infection


67

C. Chronic hepatitis B infection

D. Hepatitis C infection

E. The presence of an acute HBV infection.

Explanation:

The correct answer is E. The presence of hepatitis B surface antigen (HBsAg) along with hepatitis B core IgM
antibody (HBc IgM Ab), and the absence of hepatitis B surface antibody (HBsAb) indicates the presence of the
early stages of an acute infection with Hepatitis B. The presence of antibody to Hepatitis C (HCV) only indicates
exposure, but not a specific time of exposure; however, 85% of patients who are infected with HCV develop
chronic infections, indicating that this patient has an 85% chance of having a dual infection with HBV and HCV.
The acute or chronic HCV infection can be confirmed by PCR.

A dual infection of HBV and HAV (choice A) is not plausible since the IgM anti-HAV serology is negative.

Hepatitis A does not cause chronic disease (choice B).

Chronic HBV infection (choice C) is unlikely because the patient has HBc IgM Ab, which is characteristic of an
acute infection, rather than a chronic infection.

Hepatitis C infection (choice D) is not confirmed by these data because the presence of HCV Ab only indicates
exposure to the virus, and not the state of infection. This could be caused by exposure at some earlier time
(the elevated serum transaminases might be due to HBV infection). An active or chronic HCV infection can only
be confirmed by PCR.

An 8-year-old girl is brought to the pediatrician for a severe sore throat. The pediatrician prescribes penicillin and
sends the girl home. Later that day, she develops a diffuse maculopapular rash, shortness of breath, and
wheezing. Her parents take her to the emergency department, where she is diagnosed with anaphylaxis and
treated successfully with epinephrine. Three months later, the girl has dysuria and urinary frequency. Urine
Gram's stain reveals numerous gram-negative rods. Which of the following antibiotics could be safely given to
this patient?

A. Ampicillin

B. Aztreonam

C. Cefoperazone

D. Cephalexin

E. Methicillin

F. Oxacillin

G. Ticarcillin

Explanation:

The correct answer is B. Aztreonam is a monobactam. Since the basic ring structure is different from penicillins,
there is no cross-allergenicity, and it can be safely given to those who have had severe reactions to penicillins.
Aztreonam is highly active against gram-negative bacteria, but has no activity against gram-positive bacteria or
obligate anaerobes. It is not β-lactamase resistant.
68
In general, there is cross-allergenicity between all the penicillins, since most people react to breakdown
products of the β-lactam ring structure common to all penicillins. Patients who have experienced an allergic
reaction to a penicillin may also be sensitive to cephalosporins.

Ampicillin (choice A) is a third-generation penicillin. It is a broad-spectrum penicillin that is active against certain
gram-negative and gram-positive bacteria. It is also one of the main antibiotics (along with clindamycin) that can
lead to antibiotic-induced pseudomembranous colitis.

Cefoperazone (choice C) is a third-generation cephalosporin. It is a broad-spectrum cephalosporin that is active


against many gram-negative and gram-positive bacteria. It contains a methylthiotetrazole side chain that can
cause a vitamin K deficiency and disulfiram-like reaction to alcohol. Many people also experience diarrhea as a
side effect. It is a drug of choice in people with impaired renal function because 60% is eliminated by the biliary
route.

Cephalexin (choice D) is a first-generation cephalosporin. It is mainly active against gram-positive bacteria.

Methicillin (choice E) is a second-generation penicillin. It is active against many gram-positive organisms. When
given in high doses for more than 2 weeks, it can cause interstitial nephritis, hepatitis, and neutropenia.

Oxacillin (choice F) is a second-generation penicillin. It is used against many gram-positive organisms. When
used in high dose for more than 2 weeks, it can cause hepatitis and neutropenia.

Ticarcillin (choice G) is a fourth-generation penicillin. Fourth-generation penicillins are extended-spectrum


agents that are active against many gram-positive and gram-negative bacteria, including Pseudomonas and
many Enterobacteriaceae. Ticarcillin is given with clavulanic acid, a β-lactamase inhibitor. Since ticarcillin
is a disodium salt, it causes a large salt load, which can lead to salt retention and hypokalemia.

A 38-year-old woman with a history of multiple sexual partners is most at risk for which of the following?

A. Bladder carcinoma

B. Cervical carcinoma

C. Endometrial carcinoma

D. Ovarian carcinoma

E. Rectal carcinoma

Explanation:

The correct answer is B. In a sense, cervical carcinoma can be considered a "sexually transmitted disease,"
since human papilloma virus (typically strains HPV-16 or HPV- 18) is transmitted venereally. In our culture,
HPV-related dysplasia of the cervix is common, even in the 18-28 year old group, and carcinomas of the cervix
(more rare now than in the past due to screening and aggressive therapy of dysplasia) can occur in the 35-45
age group. The other cancers listed typically occur in older age groups and are not as clearly related to
venereal agents.

A Brazilian immigrant is hired at a meat-packing plant, and undergoes an employment physical. Chest x-ray
demonstrates a patchy, bilateral pneumonia and a lung mass, and he is referred to a specialist. Biopsy of the
mass demonstrates fungal organisms with a few very distinctive "pilot's wheel" yeast forms. Which of the following
is the most likely diagnosis?

A. Blastomycosis
69
B. Coccidioidomycosis

C. Histoplasmosis

D. Paracoccidioidomycosis

E. Sporotrichosis

Explanation:

The correct answer is D. Latin America and "pilot's wheel" budding yeast are clues for paracoccidioidomycosis.
This disease is caused by Paracoccidioides brasiliensis, a dimorphic fungus that is found as a multiply-budding
yeast in tissues.

Clues for questions about blastomycosis (choice A) would include spending time in states east of the Mississippi
River and fairly large yeast.

Clues for questions about coccidioidomycosis (choice B) would be spending time in the southwestern deserts of
the United States and spherules filled with endospores.

Clues for questions about histoplasmosis (choice C) would be spending time in the Ohio, Mississippi, and
Missouri River Valleys and finding tiny yeast forms in macrophages.

Clues for questions about sporotrichosis (choice E) would be skin lesions in rose gardeners.

A psychotic, indigent man with a history of multisubstance abuse has been involuntarily hospitalized for 1 week.
Because of persistent diarrhea, stools are sent for ova and parasites, revealing numerous granular, spherical,
thin-walled cysts measuring 10-20 µm in diameter. Trichrome stains show up to four nuclei in most of the cysts.
These finding are consistent with an infection by which of the following organisms?

A. Cryptosporidium parvum

B. Dientamoeba fragilis

C. Entamoeba histolytica

D. Giardia lamblia

E. Isospora belli

Explanation:

The correct answer is C. Entamoeba are relatively common enteric pathogens that can produce asymptomatic
infection or more severe disease characterized by mucosal ulcerations and submucosal spread causing
abdominal distress and liquid stools. Stools may show either trophozoite forms or the typical spherical cysts.
Several species of Entamoeba are seen, including Entamoeba coli and E. hartmanni. E. histolytica cysts
characteristically are spherical in shape, 10-20 µm in diameter, and have granular cytoplasm containing 1, 2, or
4 nuclei.

Cryptosporidium parvum(choice A) infections occur in the immunocompromised population and may cause
severe diarrhea. The organism presents as minute (2-5 µm) intracellular spheres or arc-shaped merozoites
under normal mucosa, and can be difficult to appreciate by light microscopy. Cysts in the stool are too small
(4-5 µm) to be confused with Entamoeba.

Dientamoeba fragilis(choice B) is an intestinal amoeba that also may produce an infectious diarrhea. It does not
have a cyst form, and only the trophozoite forms are seen in stools.
70

Giardia lamblia(choice D) is a flagellate protozoan that infects the stomach and small intestine when
contaminated water is ingested. Diagnosis is usually made by examining duodenal contents; however, the stools
may contain the oval or elliptical cysts, which are thick-walled and measure 8-14 µm in diameter. Spherical cysts
are not seen in Giardia infection.

Isospora belli(choice E) produces self-limited intestinal infections mostly in the tropics, where fever and diarrhea
may last weeks to months. The stool-borne cysts are much larger than Entamoeba (30 x 15 µm), are
asymetrical, and are typically almond-shaped.

An autopsy is performed on a man who suddenly began vomiting voluminous quantities of blood and exsanguinated.
The man's stomach is presented in the accompanying photograph. Which of the following organisms is most likely
implicated in the pathogenesis of this disease?

A. Cryptosporidium parvum

B. Entamoeba histolytica

C. Escherichia coli

D. Helicobacter pylori

E. Mycobacterium tuberculosis

Explanation:

The correct answer is D. The gross photograph shows a stomach with a perforated peptic ulcer. Perforation of a
peptic ulcer is potentially fatal, because of either peritonitis with sepsis or sudden exsanguination (if the perforation
damages one of the many arteries of the stomach). Peptic ulcer disease, gastritis, and possibly gastric carcinoma
and gastric lymphoma have been strongly associated with Helicobacter pylori colonization of the mucus layer
covering the gastric mucosa. Colonization is associated with destruction of the mucus layer, thereby destroying its
71
protective function.

Cryptosporidium parvum(choice A) causes diarrhea that is severe in immunocompromised patients.

Entamoeba histolytica(choice B) produces dysentery-like symptoms or can cause liver abscess.

Escherichia coli(choice C) causes a variety of diarrheal diseases and can infect the bladder and soft tissues.

Mycobacterium tuberculosis(choice E) causes tuberculosis, characterized by granuloma formation, especially in the


lungs.

A 34-year-old HIV-positive man without previous opportunistic infections presents complaining of dyspnea with
daily activity. He states that he has had a mild cough and fever but denies having had chills, sputum production,
or chest discomfort. Physical examination is remarkable for oral thrush and a few small, nontender cervical lymph
nodes. A chest x-ray film reveals bilateral interstitial infiltrates, and bronchoalveolar lavage reveals small
silver-staining cysts. In which other patient population in the U.S. is this organism a frequent cause of a
life-threatening pneumonia?

A. Bone marrow transplant patients

B. Hospitalized adults on antibiotic therapy

C. Late-term pregnant women

D. Normal adults in periods of stress

E. Premature infants

F. Preschool children

G. Sexually active adolescents

H. Third-trimester fetuses

Explanation:

The correct answer is E. This patient has Pneumocystis carinii pneumonia, the most common atypical
pneumonia in AIDS patients. The next most common pool of at-risk individuals consists of premature infants.

Bone marrow transplant patients (choice A) would be immunologically compromised, and might be a second
at-risk group (behind premature infants), but would not be the first choice on this list.

Hospitalized adults on antibiotic therapy (choice B) might be slightly immunologically compromised or stressed,
but are not the major patient pool at risk for this infection in the U.S.

Late-term pregnant women (choice C) are not immunologically compromised and would not be at risk.

Normal adults in periods of stress (choice D) are not particularly susceptible to opportunistic pathogens.

Preschool children (choice F) have normal rates of exposure to P. carinii, as would any adult, but are not
susceptible to serious pneumonia with this agent unless they are immunologically suppressed.

Sexually active adolescents (choice G) would have the same infection rates as normal infants and adults, but
again would not be susceptible to life-threatening pneumonia. P. carinii is transmitted via aerosols, not sexual
activity.
72

Third-trimester fetuses (choice H) are not susceptible to infection with this fungus, although they may become
infected in utero with HIV.

A 16-year-old high school cheerleader presents with low grade fever, pleuritic pain and a non-productive cough.
Her serum agglutinates Streptococcus salivarius strain MG. Therapy should include which of the following?

A. Ampicillin

B. Erythromycin

C. Oxygen and external cooling

D. Penicillin G

E. Ribavirin

Explanation:

The correct answer is B. The patient has primary atypical pneumonia caused by Mycoplasma pneumoniae.
These organisms are fastidious and difficult to culture in the laboratory, however serodiagnosis can be most
helpful. Patients produce one or two heterophile antibodies during the course of the infection: one agglutinates
human O+ RBCs in the cold (the cold hemagglutinins) while the other causes the agglutination of a strain of
Streptococcus salivarius termed strain MG (the Strep MG agglutinins). Mycoplasma are susceptible to the
macrolide family of antibiotics (erythromycin, clarithromycin, etc.). These organisms lack a cell wall, hence they
are indifferent to antibiotics that interfere with peptidoglycan synthesis, such as penicillin (choice D) and
ampicillin (choice A).

Oxygen and external cooling (choice C) are therapeutic measures that are used in the treatment of severe
respiratory diseases such as pneumococcal pneumonia.

Ribavirin (choice E) is used in the treatment of respiratory syncytial virus infection in infants. This is the most
common cause of hospitalization for respiratory disease in the very young, probably because aerosol
administration of the antiviral compound is best accomplished in a hospital setting.

A 24-year-old summer camp counselor complains of a severe headache and weakness. His condition rapidly
deteriorates over a period of hours, and he is airlifted to a nearby hospital. A lumbar puncture is performed and a
Gram's stain of spinal fluid reveals gram-negative diplococci. Infection with this organism is also associated with
which of the following?

A. Dysentery

B. Erythema chronicum migrans

C. Myocarditis

D. Ophthalmia neonatorum

E. Waterhouse-Friderichsen syndrome

Explanation:

The correct answer is E. The Neisseria are gram-negative diplococci, and have two clinically important species,
N. meningitidis and N. gonorrhoeae.N. meningitidis can either cause a sudden, fulminant, life-threatening
73
meningitis or meningococcemia with a vasculitic purpura and disseminated intravascular coagulation. The latter
presentation may be complicated by adrenal involvement, precipitating the usually fatal
Waterhouse-Friderichsen syndrome with coagulopathy, hypotension, adrenal cortical necrosis, and sepsis. N.
gonorrhoeae causes gonorrhea and ophthalmia neonatorum (choice D), a neonatal eye infection.

Classical dysentery (multiple, small-volume stools with blood, mucus, and pus associated with abdominal cramps
and tenesmus); (choice A) is caused by Shigella dysenteriae. A similar syndrome can be caused by other
bacteria or amoebae as well.

Erythema chronicum migrans (choice B) is the pathognomonic dermatologic manifestation of Lyme disease. This
skin lesion consists of an expanding erythematous lesion, with central clearing.

Bacterial myocarditis (choice C) is uncommon, and when it does occur, it usually involves Staphylococcus aureus
or Corynebacterium diphtheriae.

A British dairy farmer develops fever with chills, myalgias, headache, skin rash, and vomiting. He is quite ill and is
hospitalized. Blood cultures demonstrate tightly coiled, thin, flexible spirochetes shaped like a Shepherd's crook.
The spirochetes are easily cultured in serum-enriched nutrient agar. Which of the following organisms should be
suspected?

A. Brucella abortus

B. Brucella melitensis

C. Leptospira interrogans

D. Pseudomonas mallei

E. Pseudomonas pseudomallei

Explanation:

The correct answer is C. The only spirochete among the choices is Leptospira interrogans, so even if you didn't
know the diseases these organisms produce, you may have been able to answer the question. Leptospirosis,
which this patient has, is caused by a spirochete; if you were presented with a list of spirochetes in the choices,
the phrase "Shepherd's crook" should tip you off to Leptospira. Clinically, leptospirosis may range from nearly
asymptomatic, or at least indistinguishable from other minor flu-like illnesses, to a potentially fatal form (Wal's
disease) with jaundice, bleeding, renal failure, and skeletal muscle necrosis. Spread is via contact with blood or
urine from infected animals, notably rats. Leptospirosis is found worldwide, but its more severe forms are most
likely to occur in the tropics.

Brucella abortus (choice A) is a gram-negative coccus and is one of the causes of brucellosis.

Brucella melitensis(choice B) is a gram-negative coccus and is one of the causes of brucellosis.

Pseudomonas mallei (choice D) and Pseudomonas pseudomallei(choice E) are small gram-negative bacilli that
cause melioidosis.

A patient is referred to a neurologist because of ataxia. Neurological examination reveals a loss of proprioception
and a wide-based, slapping gate. Magnetic resonance imaging reveals degeneration of the dorsal columns and
dorsal roots of the spinal cord. Which of the following organisms is most likely to have caused this pattern of
damage?

A. Haemophilus influenzae
74

B. Herpes simplex I

C. Neisseria gonorrhoeae

D. Neisseria meningitidis

E. Treponema pallidum

Explanation:

The correct answer is E. The findings described are those of tabes dorsalis, a form of tertiary syphilis caused
by Treponema pallidum. Tabes dorsalis, and other forms of tertiary syphilis, are now uncommon in this country,
possibly because the common use of antibiotics may "treat" many unsuspected cases of syphilis.

Haemophilus influenzae (choice A) and Neisseria meningitidis(choice D) can cause meningitis.

Neisseria gonorrhoeae(choice C) causes gonorrhea, which usually does not involve the CNS.

Herpes simplex I (choice B) can cause an encephalitis that typically involves the frontal and temporal lobes.

A patient presents to a physician with jaundice. Physical examination reveals a nodular, enlarged liver. CT of the
abdomen shows a cirrhotic liver with a large mass. CT-guided biopsy of the mass demonstrates a malignant
tumor derived from hepatic parenchymal cells. Infection with which of the following viruses would most likely be
directly related to the development of this tumor?

A. Epstein-Barr virus (EBV)

B. Hepatitis B virus (HBV)

C. Human herpesvirus type 8 (HHV 8)

D. Human papillomavirus (HPV)

E. Human T-lymphocyte virus (HTLV-1)

Explanation:

The correct answer is B. The tumor is hepatocellular carcinoma, which usually develops in the setting of
cirrhosis due to a variety of damaging agents, including hepatitis B virus (HBV) infection, alcohol use, and
hemochromatosis.

EBV (choice A) is associated with Burkitt's lymphoma and nasopharyngeal carcinoma.

HHV 8 (a member of the herpes family, choice C) is associated with Kaposi's sarcoma.

HPV (human papillomavirus, choice D) is associated with cervical, penile, and anal carcinoma.

HTLV-1 (human T-lymphocyte virus, choice E) is associated with adult T-cell leukemia.

A 58-year-old alcoholic man with multiple dental caries develops a pulmonary abscess and is treated with
antibiotics. Several days later, he develops nausea, vomiting, abdominal pain, and voluminous green diarrhea.
Which of the following antibiotics is most likely responsible for this patient's symptoms?
75

A. Chloramphenicol

B. Clindamycin

C. Gentamicin

D. Metronidazole

E. Vancomycin

Explanation:

The correct answer is B. Any time you see the development of diarrhea in the same question stem as the words
"treated with antibiotics," you should immediately think of pseudomembranous colitis. Pseudomembranous
colitis is caused by Clostridium difficile and typically occurs as a result of treatment with clindamycin or
ampicillin. You would confirm your suspicion by sending a stool culture to be tested for the presence of the C.
difficile toxin.

Chloramphenicol's (choice A) most test-worthy side effect is aplastic anemia, not diarrhea. In addition, you
might have been able to eliminate this choice simply because of the extremely low probability that this patient
would receive this antibiotic in the USA.

Gentamicin's (choice C) key side effects include ototoxicity and nephrotoxicity.

Metronidazole (choice D) and vancomycin (choice E) do not cause pseudomembranous colitis; they are used to
treat it.

A 38-year-old AIDS patient presents to the clinic complaining of nausea, occasional vomiting and "bumps" on his
groin. On physical examination, multiple, nontender, pedunculated reddish purple nodules in the inguinal and
perirectal areas are observed. The patient's liver is palpable 8 cm below the right costal margin. Routine
laboratory tests are unremarkable except for an alanine aminotransferase level of 58 and alkaline phosphatase
of 90. He denies any foreign travel, but has two pet cats. Which of the following is the most likely cause of this
patient's infection?

A. Bartonella henselae

B. Human papillomavirus

C. Molluscum contagiosum virus

D. Rickettsia prowazekii

E. Treponema pallidum

Explanation:

The correct answer is A. Bacillary angiomatosis is a disease that occurs primarily in AIDS patients, and is
indicative of a defect in cell-mediated immunity. It is caused by either Bartonella henselae or Bartonella
quintana. The domestic cat is the reservoir for these organisms and they are usually transmitted to humans via
a cat scratch or cat bite. Patients with this illness usually have multiple skin lesions and extracutaneous
manifestations involving liver and bone. Diagnosis is usually based on characteristic histopathologic findings
including plump "epithelioid" endothelial cells and mitotic figures. A macrolide, such as erythromycin or
azithromycin, is the drug of choice for the infection.

Human papillomavirus (choice B) causes warts. Infection can present as a sessile wart or as condyloma
76
acuminatum, which are fleshy soft growths that coalesce into large masses. When cellular immunity is
depressed, as in AIDS, the condylomata acuminatum proliferate.

Molluscum contagiosum virus (choice C) is a pox virus that is spread by close person-to-person contact.
Infection produces a firm nodule that often becomes umbilicated, and may resolve by discharging its contents.
In AIDS, the lesions do not resolve, but enlarge and spread.

Rickettsia prowazekii(choice D) is the cause of epidemic typhus. It is spread by the human body louse,
Pediculus humanis. Its reservoirs are humans and flying squirrels.

Treponema pallidum(choice E) is the spirochete that causes syphilis. The characteristic primary lesion is a
chancre (a painless, indurated ulcer) at the site of inoculation.

A 7-year-old girl develops a fever, conjunctivitis, photophobia, and a cough. Her pediatrician notes white spots on
a bright red background on the girl's buccal mucosa. Within days, a rash begins around the hairline, then
spreads to the trunk and extremities. One week later, the child suddenly begins to convulse, and loses
consciousness. She is taken to the emergency room, where involuntary movements and pupillary abnormalities
are noted. Which of the following would most likely be seen on CNS biopsy?

A. Demyelination of white matter of cerebral hemispheres with abnormal giant oligodendrocytes

B. Perivenous microglial encephalitis with demyelination

C. Phagocytosis of motor neurons in the spinal cord

D. Severe hemorrhagic and necrotizing encephalitis of the temporal lobe with eosinophilic Cowdry type A
inclusion in neurons and glia

E. Small granulomas with central caseation in the meninges

Explanation:

The correct answer is B. The initial history given is classic for measles, with the appearance of Koplik's spots
(white spots on the buccal mucosa) followed by a rash beginning along the neck and hairline and spreading to
the trunk and extremities. The sequela this child is experiencing is post-infectious encephalomyelitis, which can
follow either infection with measles, varicella, rubella, mumps, or influenza, or vaccination with vaccinia vaccine
or rabies vaccine derived from nervous tissue. Treatment is supportive, with a mortality of 15 to 40%; survivors
frequently have significant permanent neurologic deficits. The pathologic finding is perivenous microglial
involvement with demyelination.

Choice A describes the findings of progressive multifocal leukoencephalopathy, a demyelinating disease caused
by infection with JC virus, especially in immunocompromised individuals.

Choice C describes the findings of poliomyelitis, a paralytic disease affecting the ventral horn of the spinal cord
and motor cortex, caused by an enterovirus (poliovirus).

Choice D describes the findings in herpes encephalitis, which typically affects the inferomedial temporal lobes
and orbitofrontal gyri.

Choice E describes the findings in tuberculous meningitis, caused by M. tuberculosis.

A poor African community is experiencing an epidemic of severe hepatitis. The mortality among pregnant women
is particularly high. Which of the following viruses is the most likely cause of the epidemic?

A. Cytomegalovirus (CMV)
77

B. Hepatitis A virus (HAV)

C. Hepatitis C virus (HCV)

D. Herpes simplex I

E. Hepatitis E virus (HEV)

Explanation:

The correct answer is E. Hepatitis E is an important, and until recently, unrecognized cause of epidemics of
enterically transmitted acute hepatitis. Hepatitis E is caused by an enterically transmitted virus that occurs
primarily in India, Asia, Africa, and Central America. Infection with the virus is associated with a very high
mortality among pregnant women.

CMV (choice A) can cause acute hepatitis, but the disease is usually mild and often goes unrecognized, except
in profoundly immunosuppressed patients.

Hepatitis A virus (choice B) is the major cause of epidemics of enterically transmitted viral hepatitis, but is not a
significant cause of mortality in pregnant women.

Hepatitis C virus (choice C) is usually transmitted parenterally, rather than enterically, and is not a significant
cause of mortality in pregnant women.

Herpes simplex (choice D) usually causes significant hepatitis only in profoundly immunosuppressed patients.

A medical worker has a needle-stick accident involving an empty syringe that had been previously used on a
patient with a known hepatitis B infection. Which of the following is the most probable outcome for the medical
worker?

A. Acute hepatitis followed by recovery

B. "Healthy" carrier

C. Persistent infection followed by recovery

D. Persistent infection progressing to chronic hepatitis

E. Subclinical disease followed by recovery

Explanation:

The correct answer is E. Hepatitis B infection can produce a wide variety of clinical outcomes. The most
common outcome (60% to 65%), however, turns out to be subclinical disease followed by complete recovery.
The other choices listed show other possible outcomes, and their statistical impact is considered with the
discussions of individual choices.

Approximately 20% to 25% of infected persons develop acute hepatitis (choice A), which is followed in 99% of
these cases by recovery and in about 1% of cases by fulminant hepatitis.

Approximately 5% to 10% of cases become "healthy" carriers (choice B).

Approximately 4% of cases develop persistent infection, 67% to 90% of which then recover (choice C) and 10%
to 33% of which have chronic hepatitis (choice D).
78

A 2-day-old baby girl suddenly develops abdominal distention, progressive pallid cyanosis, and irregular
respirations. The newborn also has "refused" to breast-feed for the past 18 hours. If the mother was treated for a
serious infection with antibiotics for 14 days up to and including the day of delivery, which of the following
medications did the mother most likely receive?

A. Aztreonam

B. Chloramphenicol

C. Clindamycin

D. Metronidazole

E. Sulfamethoxazole/trimethoprim

Explanation:

The correct answer is B. Gray "baby" syndrome is a disorder that occurs in newborns who have either received
chloramphenicol immediately after birth or whose mothers have received the medication close to the delivery
date. Symptoms typically appear in the following order: abdominal distention with or without emesis, progressive
pallid cyanosis, and vasomotor collapse, frequently accompanied by irregular respiration. Death can occur as
early as a few hours after onset of signs and symptoms. Other symptoms may include: loose, greenish stools, a
refusal to suck, ashen color (implied by the name gray baby syndrome), and lactic acidosis. Chloramphenicol is
an antimicrobial agent used in the treatment of serious infections when less toxic alternatives are inappropriate.

Aztreonam (choice A) is a beta-lactam antibiotic used primarily in the treatment of gram-negative infections of the
urinary tract, lower respiratory tract, and skin, and for intra-abdominal infections. The use of this agent in
pregnant or nursing women and infants is considered to be safe and effective.

Clindamycin (choice C) is an anti-infective agent used in the treatment of serious infections when less toxic
alternatives are inappropriate. Although the agent is considered to be safe and effective during pregnancy, it is
associated with the development of pseudomembranous colitis and agranulocytosis.

Metronidazole (choice D) is an antibiotic used primarily in the treatment of anaerobic infections. The use of
metronidazole should be restricted in pregnancy since newborns have a decreased ability to metabolize this
medication. When the elimination of metronidazole is decreased, the severity of adverse reactions increases.
Adverse reactions include peripheral neuropathy, seizures, irritability, and profound gastrointestinal
disturbances.

Sulfamethoxazole/trimethoprim (choice E) is associated with the development of kernicterus, which is a disorder


that can cause abnormal cerebral development in infants. The majority of infants with this disorder generally die
within a few weeks of birth. Those infants who survive are often mentally retarded, deaf, or physically impaired.

A 65-year-old man presents with fever, severe headache, and nuchal rigidity. Physical examination in the
emergency department shows a Glasgow coma score of 7. Lumbar puncture reveals cloudy cerebrospinal fluid
(CSF) with 1200 neutrophils/mm3, elevated protein, and decreased glucose. Which of the following is the most
probable etiologic agent of this condition?

A. Arbovirus

B. Herpesvirus

C. Mycobacterium tuberculosis
79

D. Neisseria meningitidis

E. Streptococcus pneumoniae

Explanation:

The correct answer is E. The clinical manifestations (fever, headache, nuchal rigidity, and low Glasgow coma
score), along with the CSF findings (increased neutrophils, elevated protein, and reduced glucose), strongly
indicate acute pyogenic (bacterial) meningitis as the underlying condition. Of the microorganisms listed, either
Neisseria meningitidis or Streptococcus pneumoniae can cause this form of meningitis; however, Streptococcus
pneumoniae is by far the most frequent organism causing acute meningitis in elderly patients.

Arboviruses and herpesviruses (choices A and B) can cause an encephalitis characterized by lymphocytic
infiltration of the brain parenchyma and leptomeninges. In this case, CSF findings would include an increased
number of lymphocytes and a normal glucose concentration.

Mycobacterium tuberculosis(choice C) may cause a chronic meningoencephalitis, with a prolonged clinical


course. It is characterized pathologically by a dense granulomatous infiltrate of the base of the brain.
Associated CSF findings include increased lymphocytes and normal or slightly decreased glucose.

Neisseria meningitidis(choice D) is the classic etiologic agent associated with acute pyogenic meningitis, but it
usually affects adolescents and young adults. In neonates, the most frequent organisms include Escherichia
coli and group B streptococci; in infants and children, Hemophilus influenzae; and in the elderly, Streptococcus
pneumoniae and Listeria monocytogenes.

A 12-year-old boy has a productive cough characterized by large volumes of foul-smelling sputum. Three years
ago, the patient was diagnosed with pancreatic insufficiency, as evidenced by repetitive gastrointestinal symptoms
of steatorrhea. After culture of the sputum, colorless, oxidase-positive colonies with a fruity aroma develop on the
agar. The function of which of the following proteins is most likely inhibited by the bacteria responsible for this
boy's infection?

A. A CFTR protein lacking a phenylalanine in exon 10 at position 508

B. A GTP-binding protein involved in the elongation step of protein synthesis

C. A GTP-binding protein similar to the one coupled with α2-adrenergic receptors

D. A GTP-binding protein similar to the one coupled with β-adrenergic receptors

E. A phosphorylation-regulated chloride channel in the apical membrane of epithelial cells

Explanation:

The correct answer is B. The boy is displaying the characteristic symptoms of cystic fibrosis with bronchiectasis
(accounting for the foul-smelling sputum) and pancreatic insufficiency, producing steatorrhea. However, the
question directly relates to the infectious agent causing the pneumonia, Pseudomonas aeruginosa, an
oxidase-positive bacteria with a fruity aroma. P. aeruginosa is a strictly aerobic gram-negative rod that produces
an exotoxin (exotoxin A) that ADP-ribosylates, and therefore inhibits, eukaryotic elongation factor 2 (eEF-2),
which is a G-protein involved in the eukaryotic translation of proteins.

The cystic fibrosis transmembrane conduction regulator (CFTR) protein (choice A) is the product of the cystic
fibrosis gene, a large 24 exon gene located on the long arm of chromosome 7 (7q31). This protein has two
nucleotide binding domains. In 70% of the families with the disease, a three base pair deletion at codon 508 of
exon 10 results in the deletion of a phenylalanine (DF508). This codon is critical for one of the nucleotide
80
binding sites of the CF gene product, resulting in poor function of a phosphorylation-regulated chloride ion
channel in the apical membrane of epithelial cells. The defective chloride channel leads to impaired secretion of
chloride in the lumen and promotes the absorption of sodium inside the cells. Water follows, concentrating the
airway secretions. The increased viscosity of the airway secretions predisposes the patient to repeated
infections.

A GTP-binding protein similar to the one coupled with α2-adrenergic receptors (choice C) refers to a
G-protein that inhibits adenylate cyclase, lowering cAMP. This G-protein is sensitive to Pertussis toxin.

A GTP-binding protein similar to the one coupled with β-adrenergic receptors (choice D) refers to a
G-protein that stimulates adenylate cyclase, increasing cAMP. This G-protein is sensitive to cholera toxin.

The product of the cystic fibrosis gene is a phosphorylation-regulated chloride channel in the apical membrane
of epithelial cells (choice E).

During the asymptomatic latent phase of AIDS, the virus is actively proliferating, and can be found in association
with

A. B lymphocytes

B. follicular dendritic cells in lymph nodes

C. ganglion cells

D. oligodendrocytes

E. peripheral nerves

Explanation:

The correct answer is B. Follicular dendritic cells in the germinal centers of lymph nodes are important
reservoirs of HIV. Although some follicular dendritic cells are infected with HIV, most viral particles are found on
the surface of their dendritic processes. Follicular dendritic cells have receptors to the Fc portion of
immunoglobulins that serve to trap HIV virions coated with anti-HIV antibodies. These coated HIV particles retain
the ability to infect CD4+ T cells as they traverse the dendritic cells.

B lymphocytes (choice A) have a surface marker (CD21 protein-a complement receptor) to which an
Epstein-Barr envelope glycoprotein can bind. The virus associates with the host cell genome, producing a latent
infection. These B cells undergo polyclonal activation and proliferation.

Ganglion cells (choice C), particularly the satellite cells around the ganglion cells in the dorsal root ganglia, can
be infected by varicella-zoster. Herpes type I and II infect neurons that innervate skin and mucous membranes.

Oligodendrocytes (choice D) are directly infected by two viruses, JC virus (a polyomavirus) and measles virus.
JC virus causes progressive multifocal leukoencephalopathy (PML), and measles virus produces a latent
syndrome called subacute sclerosing panencephalitis (SSPE).

Peripheral nerves (choice E) are indirectly affected by HIV virus in the AIDS-associated myopathy. The disease
is characterized by a subacute onset of proximal muscle weakness, sometimes with pain, and elevated levels of
creatine kinase. The muscles and nerves are infiltrated with mononuclear cells, including HIV-positive
macrophages.

A viral organism was isolated from a painful blister on the lip of a teenage girl. The agent was found to
double-stranded, linear DNA and was enveloped. The patient had a similar sore approximately 2 months ago.
Which of the following is the most likely causative organism?
81

A. Adenovirus

B. Coxsackie virus

C. Herpes simplex type 1 virus

D. Herpes zoster virus

E. Papilloma virus

Explanation:

The correct answer is C. Herpes simplex is an enveloped, linear DNA virus that is a very common infectious
agent; most adults will have anti-Herpes simplex antibodies in their serum, although many may not have ever
had any clinical signs of disease. The hallmark of this disease is painful skin vesicles often called "cold" sores,
or "fever" blisters to denote the precipitating event that preceded the appearance of the lesions. The virus has
a propensity to become latent in the host, finding safe refuge in nervous tissue. Activation of the infection
occurs following mild trauma (e.g., a visit to the dentist), hormonal changes (e.g., menses), and
immunosuppression (e.g., following organ transplantation). Other, more serious, manifestations of disease
include encephalitis, pneumonia, and hepatitis; these are particularly likely to be seen in immunodeficient
patients such as those with AIDS.

Adenoviruses (choice A) are naked, linear, double-stranded DNA viruses that cause acute, usually self-limiting,
influenza-like illnesses occurring in the fall and winter. The symptoms include pharyngitis, fever, cough, and
general malaise. Epidemic pharyngoconjunctivitis and pneumonia can occur in closed populations such as
military installations.

Coxsackie viruses (choice B) are naked, single-stranded, polycistronic viruses with an RNA genome. They are
divided into groups A and B based on their virulence in suckling mice. Coxsackie A group causes generalized
myositis and flaccid paralysis, which is rapidly fatal to infant mice, whereas group B produces less severe
lesions of the heart, pancreas, and central nervous system. In man, Coxsackie A causes herpangina and
hand-foot-and-mouth disease, while Coxsackie B is seen in patients with pleurodynia, myocarditis, and
pericarditis. Both groups cause upper respiratory infections, febrile rashes, and meningitis.

Herpes zoster (choice D), the varicella virus, is an enveloped, double-stranded DNA virus that is a very common
infectious agent in children. Chickenpox is a mild, self-limiting illness in children that is evidenced as a fever
followed by a macular rash that progresses to papules, then vesicles of the skin and mucous membranes.
Shingles is a recurrence of a previously latent varicella infection in which the virus has taken refuge in sensory
ganglia of spinal or cranial nerves. Various factors that decrease the immune status of the patient contribute to
the exacerbation of the infection. Severe dermatomal pain occurs with a vesicular eruption, fever, and malaise.

Papilloma viruses (choice E) are members of the Papovavirus family. They are non-enveloped and possess a
double-stranded, circular DNA genome. They cause skin, plantar, and genital warts; some serotypes of human
papilloma viruses (e.g., HPV-16) are associated with penile, laryngeal, and cervical carcinomas.

A viral organism was isolated from a painful blister on the lip of a teenage girl. The agent was found to
double-stranded, linear DNA and was enveloped. The patient had a similar sore approximately 2 months ago.
Which of the following is the most likely causative organism?

A. Adenovirus

B. Coxsackie virus

C. Herpes simplex type 1 virus

D. Herpes zoster virus


82

E. Papilloma virus

Explanation:

The correct answer is C. Herpes simplex is an enveloped, linear DNA virus that is a very common infectious
agent; most adults will have anti-Herpes simplex antibodies in their serum, although many may not have ever
had any clinical signs of disease. The hallmark of this disease is painful skin vesicles often called "cold" sores,
or "fever" blisters to denote the precipitating event that preceded the appearance of the lesions. The virus has
a propensity to become latent in the host, finding safe refuge in nervous tissue. Activation of the infection
occurs following mild trauma (e.g., a visit to the dentist), hormonal changes (e.g., menses), and
immunosuppression (e.g., following organ transplantation). Other, more serious, manifestations of disease
include encephalitis, pneumonia, and hepatitis; these are particularly likely to be seen in immunodeficient
patients such as those with AIDS.

Adenoviruses (choice A) are naked, linear, double-stranded DNA viruses that cause acute, usually self-limiting,
influenza-like illnesses occurring in the fall and winter. The symptoms include pharyngitis, fever, cough, and
general malaise. Epidemic pharyngoconjunctivitis and pneumonia can occur in closed populations such as
military installations.

Coxsackie viruses (choice B) are naked, single-stranded, polycistronic viruses with an RNA genome. They are
divided into groups A and B based on their virulence in suckling mice. Coxsackie A group causes generalized
myositis and flaccid paralysis, which is rapidly fatal to infant mice, whereas group B produces less severe
lesions of the heart, pancreas, and central nervous system. In man, Coxsackie A causes herpangina and
hand-foot-and-mouth disease, while Coxsackie B is seen in patients with pleurodynia, myocarditis, and
pericarditis. Both groups cause upper respiratory infections, febrile rashes, and meningitis.

Herpes zoster (choice D), the varicella virus, is an enveloped, double-stranded DNA virus that is a very common
infectious agent in children. Chickenpox is a mild, self-limiting illness in children that is evidenced as a fever
followed by a macular rash that progresses to papules, then vesicles of the skin and mucous membranes.
Shingles is a recurrence of a previously latent varicella infection in which the virus has taken refuge in sensory
ganglia of spinal or cranial nerves. Various factors that decrease the immune status of the patient contribute to
the exacerbation of the infection. Severe dermatomal pain occurs with a vesicular eruption, fever, and malaise.

Papilloma viruses (choice E) are members of the Papovavirus family. They are non-enveloped and possess a
double-stranded, circular DNA genome. They cause skin, plantar, and genital warts; some serotypes of human
papilloma viruses (e.g., HPV-16) are associated with penile, laryngeal, and cervical carcinomas.

Which of the following is a feature of gram-positive bacteria rather than gram-negative bacteria?

A. Lipid A-containing lipopolysaccharide

B. Lipoprotein in periplasmic space

C. Outer membrane

D. Peptidoglycan in periplasmic space

E. Thick peptidoglycan cell wall

Explanation:

The correct answer is E. Most of the features listed are those of gram-negative bacteria, which have a complex
cell envelope consisting of a cytoplasmic (inner) membrane, a periplasmic space containing peptidoglycan
(choice D) and lipoprotein (choice B), an outer membrane (choice C), and sometimes a capsule. The outer
membrane contains lipopolysaccharide (choice A) which is a major component of endotoxin. The peptidoglycan
83
cell wall of the gram-negative bacteria is thin, while that of the gram-positive bacteria is thick. Other features of
gram- positive bacteria include a fairly simple surface with cytoplasmic membrane, peptidoglycan, cell wall, and
sometimes, an outer capsule. The cell wall contains lipoteichoic acids.

A mailman gets a severe bite wound from a pit bull guarding a junkyard. The wound is cleansed and he receives
a booster injection of tetanus toxoid and an injection of penicillin G. Several days later, the wound is inflamed and
purulent. The exudate is cultured on blood agar and yields gram-negative rods. Antibiotic sensitivity tests are
pending. The most likely agent to be isolated is

A. Bartonella henselae

B. Brucella canis

C. Clostridium tetani

D. Pasteurella multocida

E. Toxocara canis

Explanation:

The correct answer is D. Pasteurella multocida is a gram-negative rod that is normal flora of the oral cavity of
dogs and cats. It often causes a local abscess following introduction under the skin by an animal bite. Most
cases occur in children who are injured while playing with a pet.

Bartonella henselae (choice A) is a very small, gram-negative bacterium that is closely related to the rickettsia,
although it is able to grow on lifeless media. It is the cause of cat-scratch disease (a local, chronic lymphadenitis
most commonly seen in children) and bacillary angiomatosis (seen particularly in AIDS patients). In this latter
patient population, the organism causes proliferation of blood and lymphatic vessels causing a characteristic
"mulberry" lesion in the skin and subcutaneous tissues of the afflicted individual.

Brucella canis(choice B) is a gram-negative rod that is a zoonotic agent. Its normal host is the dog, but when it
gains access to humans, it causes an undulating febrile disease with malaise, lymphadenopathy and
hepatosplenomegaly. The normal route of exposure is via ingestion of the organism.

Clostridium tetani(choice C) is a gram-positive spore-forming anaerobic rod. It causes tetanus (a spastic


paralysis caused by tetanospasmin, which blocks the release of the inhibitory neurotransmitters glycine and
gamma-aminobutyric acid [GABA]). There may be no lesion at the site of inoculation, and exudation would be
extremely rare.

Toxocara canis (choice E), a common intestinal parasite of dogs, is a metazoan parasite that causes visceral
larva migrans. Young children are most likely to be affected, as they are most likely to ingest soil contaminated
with eggs of the parasite.

The World Health Organization identifies an alarming increase in hospital admissions worldwide attributable to a
new and unexpected serotype of influenza A virus. The biological attribute of influenza A virus, which allows the
sudden appearance of dramatically new genetic variants, is also present in a limited number of other viral
families. Which of the following viruses also possesses this biological attribute?

A. Coronavirus

B. HIV

C. Measles virus
84
D. Rabies virus

E. Rotavirus

F. Rubella virus

G. St. Louis encephalitis virus

Explanation:

The correct answer is E. This case describes a pandemic of influenza A, which is caused by the ability of the
virus to undergo dramatic genetic changes of type by reassortment of its segmented RNA genome - a trait
called genetic shift. The only virus on the list that possesses a segmented genome is the rotavirus, in the
reovirus family, which possesses 10-11 segments in its genome.

Coronavirus (choice A) is not segmented and is a cause of the common cold.

HIV (choice B) is not segmented and is known for its genetic drift (minor mutational changes over time due to
an error-prone polymerase), not genetic shift.

Measles virus (choice C) is not segmented and is controlled largely by vaccination.

Rabies virus (choice D) is not segmented.

Rubella virus (choice F) is not segmented.

St. Louis encephalitis virus (choice G) is a flavivirus and is not segmented.

A 35-year-old woman presents to her gynecologist because of vaginal itchiness and discharge. Pelvic
examination demonstrates abundant white, curdy material in the vagina. Microscopic examination of the material
demonstrates fungal hyphae and yeast forms. Which of the following systemic diseases can predispose for this
condition?

A. Crohn's disease

B. Diabetes mellitus

C. Disseminated gonococcal infection

D. Rheumatoid arthritis

E. Systemic lupus erythematosus

Explanation:

The correct answer is B. The patient has vulvovaginitis secondary to Candida infection. Predisposing factors
include a high vaginal pH, diabetes, and use of antibiotics. The increased vulnerability in diabetes may reflect
increased glucose concentrations in vaginal secretions and relative immunosuppression.

Crohn's disease (choice A) can predispose for fistulas involving the vagina, rather than vulvovaginitis.

Dissemination of Neisseria gonorrhoeae(choice C) can cause septic arthritis.

Neither rheumatoid arthritis (choice D) nor systemic lupus erythematosus (choice E) are specifically associated
with Candida vulvovaginitis.
85

A 1-week-old female infant with symptoms of vomiting and anorexia has a temperature of 102° F. A bulging
fontanel is noted on physical examination. The most likely agent is

A. Haemophilus influenzae type b

B. Listeria monocytogenes

C. Neisseria meningitidis

D. Staphylococcus aureus

E. Streptococcus agalactiae

Explanation:

The correct answer is E. Vomiting, anorexia, high fever (above 100.4° F), and a bulging fontanel equals
neonatal meningitis until proven otherwise. Streptococcus agalactiae (group B strep) and Escherichia coli (not
an answer choice) are the most common causes in neonates up to 1 month of age. The next most reasonable
response would have been Listeria monocytogenes(choice B), another, though less common, cause of neonatal
meningitis.

Most cases of meningitis caused by Haemophilus influenzae(choice A) occur in children between 6 months and
6 years of age, 90% of which result from the capsular type b strain. It has become much less prevalent since the
H. influenzae type b conjugate vaccine has been routinely administered to infants.

Neisseria meningitidis(choice C) is the most common cause of epidemic meningitis. The two organisms most
often associated with sporadic cases are Haemophilus influenzae and Streptococcus pneumoniae (the most
common cause in adults over 30).

Staphylococcus aureus(choice D) is not a common cause of meningitis, except in patients with CSF shunts. It is
often responsible for abscesses, osteomyelitis, endocarditis, toxic shock syndrome, and food poisoning.

A 7-month-old child presents with a 4-day history of fever, deepening cough, and dyspnea. A chest x-ray shows
multiple interstitial infiltrates and hyperinflation of the lungs. Multinucleated giant cells with cytoplasmic inclusion
bodies are seen when a nasal wash is inoculated into culture. The most appropriate therapy includes
administration of which of the following drugs?

A. Acyclovir

B. Ganciclovir

C. Ribavirin

D. Trifluorothymidine

E. Zidovudine

Explanation:

The correct answer is C. Ribavirin is an antiviral drug approved for the treatment of severe respiratory syncytial
virus infection, the most common cause of pneumonia and bronchiolitis in children under 1 year of age. It
86
should be given by aerosol.

Acyclovir (choice A) is a guanosine analog that is useful for the treatment of primary and recurrent herpes
infections and herpes simplex virus encephalitis.

Ganciclovir (choice B) is a guanosine analog used in the treatment of cytomegalovirus retinitis and
cytomegalovirus infections in AIDS patients.

Trifluorothymidine (choice D) is a thymidine analog used topically for the treatment of recurrent epithelial
keratitis and primary keratoconjunctivitis due to herpes simplex viruses.

Zidovudine (choice E) is a thymidine analog that inhibits reverse transcriptase. It is active against human
retroviruses, including HIV-1, HIV-2, and HTLV-1.

A 6-year-old boy presents to the pediatric clinic with fever and earache. He has just finished an unsuccessful
course of amoxicillin. On physical exam, his right tympanic membrane appears injected. Which of the following
antimicrobials would be most appropriate to prescribe for this patient?

A. Amphotericin B

B. Bacitracin

C. Cefaclor

D. Erythromycin

E. Sulfamethoxazole

Explanation:

The correct answer is C. The drug of choice for otitis media in children is amoxicillin. But in refractory cases,
often due to bacterial resistance, switching to a different drug class is often effective. You must look for another
medication that is effective against common organisms responsible for pediatric otitis media, such as
Streptococcus pneumoniae (a gram-positive diplococcus) and Haemophilus influenzae (a gram-negative rod). A
second-generation cephalosporin, such as cefaclor, should cover both and is the best choice. Consequently, it
is commonly used in cases of amoxicillin-resistant otitis media. None of the other choices cover the proper
spectrum of organisms.

Amphotericin B (choice A) is an antifungal polyene. It works by binding to ergosterol in the fungal cell
membrane, creating an artificial pore. It is used to treat systemic mycoses such as Aspergillus, Blastomyces,
Candida, Coccidioides, Cryptococcus, and Histoplasma.

Bacitracin (choice B) is a topical agent used to fight infection with gram-positive organisms. It interferes with cell
wall synthesis.

Erythromycin (choice D) is a macrolide antibiotic that binds to the 23s rRNA portion of the 50s subunit of
ribosomes, inhibiting release of uncharged tRNA and stopping protein synthesis. Though effective against S.
pneumoniae, it is not particularly active against H. influenzae. Note that erythromycin may be used in
amoxicillin-resistant otitis media, but only when administered with a sulfonamide such as sulfisoxazole.

Sulfamethoxazole (choice E) is a sulfonamide. It is bacteriostatic and works by inhibiting folic acid synthesis. It
resembles p-aminobenzoic acid (PABA) structurally. When combined with trimethoprim (a dihydrofolate
reductase inhibitor) it exerts a bactericidal effect and serves as the drug combination of choice for complicated
urinary tract infections.

A previously healthy 18-month-old girl is brought to the office with 2 days of irritability, poor appetite, and pulling
87
at her left ear. She has no known allergies, and her vaccinations are up-to-date. On examination, the child's
temperature is 102.8 F. She is easily consoled by the mother and moves her neck spontaneously without
discomfort. There is a clear discharge from the nares. The left tympanic membrane is erythematous, dull, and
bulging. Which of the following virulence factors is generally absent in the strains of the causative organism that
produce otitis media, compared with those that produce epiglottitis or meningitis?

A. β-Lactamase

B. IgA protease

C. Lipopolysaccharide

D. Nonpilus adhesins

E. Pili

F. Pneumolysin

G. Polyribitol phosphate

Explanation:

The correct answer is G. This is most likely a case of Haemophilus influenzae otitis media. 95% of all cases of
invasive disease (epiglottitis, meningitis) due to H. influenzae are caused by type b organisms that possess a
polyribitol phosphate capsule. Otitis media is generally not caused by type b organisms.

β-Lactamase (choice A) is an important pathogenic feature of Moraxella catarrhalis, which is another


important cause of otitis media, but would not be an agent of epiglottitis or meningitis.

IgA protease (choice B) is produced by Streptococcus pneumoniae and Neisseria meningitidis. Both of these
cause meningitis, but not as commonly in this age group, and would not be the most common causes of otitis
media in this case.

Lipopolysaccharide (choice C) is present in all gram-negative bacteria and would not be a distinguishing feature
between those that cause otitis media and epiglottitis.

Nonpilus adhesins (choice D) are mediators of attachment to the epithelium and colonization of the oropharynx,
but would not be a primary difference between the agents of otitis media and epiglottitis.

Pili (choice E) also mediate attachment to the oropharynx, but would not be the major difference between the
agents of otitis media and epiglottitis.

Pneumolysin (choice F) is a cytotoxin produced by S. pneumoniae that destroys ciliated epithelial cells.

Which of the following organisms is most likely to be implicated as a cause of urethritis that persists after
antibiotic therapy for gonorrhea?

A. Actinomyces

B. Chlamydia

C. Mycobacteria

D. Nocardia
88
E. Rickettsia

Explanation:

The correct answer is B.Chlamydia, Mycoplasma, and Ureaplasma are not effectively treated by penicillins and
cephalosporins, and are important causes of post-gonococcal urethritis. Chlamydial urethritis can be diagnosed
by using fluorescent antibodies to identify inclusions in epithelial cells.

Actinomyces(choice A) is a mouth commensal that rarely causes a deeper oral infection.

Mycobacteria(choice C) cause chronic granulomatous diseases such as tuberculosis and leprosy.

Nocardia(choice D) can cause necrotizing pneumonia and disseminated disease.

Rickettsia(choice E) cause typhus and Rocky Mountain spotted fever.

Which of the following is more frequently associated with Klebsiella pneumoniae than with Pseudomonas
aeruginosa?

A. Artificial ventilation

B. Cystic fibrosis

C. Diabetes mellitus

D. Green-colored sputum

E. Upper lobe cavitation

Explanation:

The correct answer is E.Klebsiella pneumoniae is a well-recognized cause of community-acquired lobar


pneumonia associated with cavitation. It is found typically in alcoholic males over 40 years of age with
underlying diabetes or obstructive lung disease. Klebsiella pneumoniae mimics Streptococcus pneumoniae as a
pulmonary pathogen except that Klebsiella has a greater tendency to progress to lung abscess and empyema.
Pseudomonas aeruginosa is usually associated with patients on ventilators, particularly in intensive care units.
Immunocompetent patients usually have bilateral bronchopneumonia without cavitary lesions.

Artificial ventilation (choice A) is classically associated with P. aeruginosa infection. The organism thrives in a
wet environment such as respirators, cleaning solutions, disinfectants, sinks, vegetables, flowers, endoscopes,
and physiotherapy pools.

P. aeruginosa is a very important pathogen. Mucoid strains of this organism infect the airways in patients with
cystic fibrosis (choice B), leading to acute exacerbations and chronic progression of lung damage.

Both organisms cause disease in association with diabetes (choice C). Klebsiella pneumoniae produces
pulmonary disease and P. aeruginosa causes necrotic skin ulcers in diabetics.

Green-colored sputum (choice D) is associated with P. aeruginosa, since more than half of the clinical isolates
produce the blue-green pigment pyocyanin, which is helpful in identifying the organism.

A 23-year-old woman presents to the emergency room with pelvic pain. A Gram's stain of her cervical discharge
reveals multiple polymorphonuclear leukocytes, but none contain gram-negative diplococci. Which of the following
statements best describes the two organisms that most commonly cause this disorder?
89

A. Both are unlikely to recur because of acquired cell mediated immunity

B. Both are unlikely to recur because of antibody mediated immunity

C. Both induce endocytosis by epithelial cells

D. Both are obligate intracellular parasites

E. Both respond to β-lactam antibiotics

Explanation:

The correct answer is C. In young women, the most likely causes of cervicitis and pelvic inflammatory disease
(PID) are Neisseria gonorrhoeae and Chlamydia trachomatis. Gram's staining alone may not be able to
distinguish between the two in women, so culture is warranted. Both organisms induce endocytosis by epithelial
cells.

Neisseria gonorrhoeae is a gram-negative diplococcus. It is endocytosed by mucus-secreting epithelia, and is


exocytosed into subepithelial tissues leading to necrotizing acute inflammation and destruction of ciliated and
non-ciliated cells. Through direct extension, it can lead to PID and infertility in women. It can be detected by
visualizing the organism in polymorphonuclear leukocytes (PMNs) in Gram's stained clinical material more easily
in men than in women. Therefore, culture onto Thayer-Martin agar is usually required for diagnosis in women.
Since the organism can be cultured, it is not an obligate intracellular parasite (compare with choice D). The
organism can be killed by PMNs, antibody, and complement, however, many of its outer membrane proteins
undergo antigenic or phase variation or cause the production of blocking antibody that interferes with
bactericidal activity. Immunity to the organism does not develop (compare with choices A and B). Neisseria
gonorrhoeae is resistant to penicillin, a β-lactam antibiotic (compare to choice E), because of a
plasmid-mediated penicillinase and a chromosomally-mediated mutation that decreases the permeability of the
outer membrane and decreases the affinity of penicillin binding proteins. It can be treated with ceftriaxone, a
cephalosporin, which resists the penicillinase, or with quinolones and azithromycin.

Chlamydia trachomatis is an obligate intracellular parasite (choice D) that induces endocytosis and resides in
the phagosome of infected epithelial cells. It possesses a gram-negative envelope but lacks peptidoglycan, and
hence is intrinsically resistant to all antibiotics that inhibit peptidoglycan synthesis, such as the β-lactam
antibiotics (compare with choice E). Antibodies are ineffective at controlling infection. Cell mediated immunity
(choice A) is probably the major means of controlling this infection since a deficiency in cell mediated immunity
both increases susceptibility and severity in animal models. Chronic and clinically latent infections cause serious
morbidity, including pelvic inflammatory disease, fallopian tube dysfunction and blindness. Chlamydia infection
can be treated with macrolides, quinolones and tetracyclines.

While playing in the park, an 8-year-old girl is bitten in the leg by a neighbor's cat. She presents the next day with
fever and bone pain localized to her right calf. X-ray reveals a lytic lesion of the right tibia. Results of the bone
culture are pending. You expect the infecting organism to be

A. Brucella melitensis

B. Eikenella corrodens

C. Francisella tularensis

D. Pasteurella multocida

E. Yersinia pestis

Explanation:
90

The correct answer is D. This patient has osteomyelitis due to a cat bite that penetrated the periosteum.
Whenever you see dog or cat bites in a question stem, consider Pasteurella multocida as a primary cause of
wound infection. This organism is a short, encapsulated, gram-negative rod demonstrating bipolar staining.
Rapidly arising cellulitis is particularly indicative of this organism.

Brucella melitensis(choice A) is a small, acapsular, gram-negative rod that causes brucellosis (undulant fever)
and is associated with contact with goats or sheep. Brucella abortus and Brucella suis are variants associated
with contact with cows and pigs, respectively. The organisms enter the body through the skin or through
contaminated dairy products, such as unpasteurized imported goat's milk or cheeses.

Eikenella corrodens(choice B) is commonly found in human bites. It is a gram-negative rod that is part of the
normal flora of the human mouth.

Francisella tularensis(choice C) is a small, pleomorphic, gram-negative rod that causes tularemia (rabbit fever).
It occurs most commonly in rural areas. In the USA, rabbits are the main reservoir for this organism, which is
transmitted to humans by the Dermacentor tick or by contact with infectious animal tissues.

Yersinia pestis(choice E) is responsible for bubonic plague, which has been known to occur in the western USA.
Its main reservoir is the prairie dog, and its vector is the rat flea.

A 6-week-old infant is brought to the emergency room with a 10-day history of coughing and choking spells. The
white blood cell count is elevated with 80% lymphocytes. The child is gasping for breath, experiencing paroxysms
of coughing, and vomits twice in the examination room. Encapsulated, gram-negative rods grow out on
Bordet-Gengou media. Which of the following microorganisms is the likely cause of these symptoms?

A. Bordetella pertussis

B. Haemophilus influenzae type b

C. Klebsiella pneumoniae

D. Legionella pneumophila

E. Mycoplasma pneumoniae

Explanation:

The correct answer is A. The child has whooping cough, or more accurately if one goes by the chronology of
the symptoms, "coughing whoop," as the patient is subject to a series of coughing episodes that are followed by
a rapid inspiration of air; the "whoop" is caused by the rapid passage of air through a narrowed airway. The
infection of the respiratory tract by Bordetella pertussis causes a hyperreactivity of the respiratory apparatus;
even the slightest stimulus will trigger a coughing episode. The offending pathogen excretes adenylate cyclase
and also produces an exotoxin that inactivates the inhibitory subunit of the G-protein complex, thus activating
adenylate cyclase within the cells of the respiratory system. Bordet-Gengou agar is the blood-enriched (15% vs.
5% for normal blood agar) medium used for the laboratory isolation of this pathogen. Fluorescent antibody
testing is used to verify the diagnosis.

Haemophilus influenzae type b (choice B) was the major cause of infant meningitis before the Hib conjugate
vaccine nearly eradicated this pathogen from the United States. Nonencapsulated strains cause otitis media in
children and pneumonia in adults. This organism also has a peculiar growth requirement, requiring factor X
(hematin) and factor V (NAD).

Klebsiella pneumoniae(choice C) is a gram-negative, highly encapsulated rod that is a significant pulmonary


pathogen in individuals with a compromised respiratory apparatus. It is a common cause of aspiration
pneumonia and pulmonary abscesses in alcoholics and patients with chronic obstructive pulmonary disease.
The organism is readily grown on standard laboratory media such as blood agar or MacConkey's enteric agar.
91

Legionella pneumophila(choice D) is another fastidious, gram-negative, respiratory pathogen that may cause
either a fulminating disease or a mild "walking pneumonia-like" condition (i.e., an atypical pneumonia). The
organism can be cultured on a charcoal yeast extract medium, but identification is usually accomplished by
immunofluorescent examination of the pulmonary specimen.

Mycoplasma pneumoniae(choice E) is the most common cause of primary atypical pneumonia. The disease is
considered "atypical" because the patients have a very mild disease with low-grade fever, little in the way of
constitutional signs, and a non-productive cough. These organisms are fastidious and are difficult to grow in the
laboratory. Diagnosis is by immunofluorescent staining or by demonstration of cold hemagglutinins and strep
MG agglutinins in the serum of the patient.

A patient is admitted to a psychiatric hospital after having been picked up by the police for making inappropriate
sexual advances. A detailed psychiatric interview demonstrates deficits in memory, insight, judgement, personal
appearance, and social behavior. The patient is witnessed experiencing a possible epileptic seizure. Over a
period of several years, motor findings also develop, including relaxed, but expressionless facies, tremor,
dysarthria, and pupillary abnormalities. Which of the following tests performed on his cerebrospinal fluid would
most likely be diagnostic?

A. CSF glucose

B. FTA-ABS

C. Gram's stain

D. Lymphocyte count

E. Neutrophil count

Explanation:

The correct answer is B. The patient has neurosyphilis, specifically general paresis, a term that means "general
paralysis of the insane." In this late sequela of syphilitic infection which occurs 5 to 20 years after infection,
patients develop mental deterioration, which precedes motor system deterioration, leading eventually to
"general paralysis" with mutism and incontinence. The abnormalities may be conveniently recalled using
paresis as a mnemonic for personality, affect, hyperactive reflexes, Argyll Robertson pupils in the eyes, defects
in the sensorium, intellectual decline and deficient speech. Specific anti-treponemal tests such as FTA-ABS are
usually positive on both serum and cerebrospinal fluid.

The cerebrospinal (CSF) glucose (choice A) in neurosyphilis is usually normal.

Gram's stain (choice C) of CSF will not demonstrate spirochetes in neurosyphilis.

The CSF lymphocyte count (choice D) is typically elevated in neurosyphilis, but this is a non-specific finding.

The CSF neutrophil count (choice E) is usually normal in neurosyphilis.

A 65-year-old man develops a vesicular rash localized to a narrow circumferential band on one side of his chest.
The rash is very painful, and the vesicles are confluent with some ulceration. No other significant findings are
demonstrated on physical examination. Which of the following diagnoses is most likely to be correct?

A. Chicken pox
92
B. Herpes simplex I infection

C. Herpes simplex II infection

D. Measles

E. Shingles

Explanation:

The correct answer is E. This is shingles, the recurrent form of herpes zoster infection, which is usually (except
in the case of immunosuppressed patients) confined to a single dermatome. Isolated vesicles may be seen
outside the dermatome. The primary herpes zoster infection precedes the development of shingles by years or
decades; the prevalence of shingles rises steadily with age, to the point that 1% of people older than 80 years
have the condition. Shingles lesions are infections and should be considered an infectious hazard in the
hospital setting. Acyclovir can ameliorate the condition.

Varicella, or chicken pox (choice A), is the primary form of herpes zoster infection and affects face and trunk
diffusely.

Herpes simplex I (choice B) affects oral and perioral sites.

Herpes simplex II (choice C) primarily affects genital sites.

Measles (choice D) causes a blotchy rash, rather than a dermatomal one.

The electron transport system of Neisseria is located on which of the following structures?

A. Cytoplasmic membrane

B. Mesosome

C. Mitochondria

D. Nuclear membrane

E. Polyribosome DNA aggregates

Explanation:

The correct answer is A. Unlike eukaryotic cells in which the electron transport system is located on
mitochondria, the electron transport system of bacteria is located on the cytoplasmic (plasma) membrane.

Mesosomes (choice B) and polyribosome DNA aggregates (choice E) are also found in bacterial cells and
function in cell division and protein synthesis, respectively.

Bacteria do not have mitochondria (choice C) or membrane-bounded nuclei (choice D).

A Pap smear from a 30-year-old woman demonstrates protozoal parasites. These organisms are likely to be

A. Cryptosporidium parvum

B. Entamoeba histolytica

C. Giardia lamblia
93

D. Isospora belli

E. Trichomonas vaginalis

Explanation:

The correct answer is E.Trichomonas vaginalis (note "vagina" in the name) is the only organism listed to
primarily affect the genital tract rather than the intestinal tract. The vaginitis it causes is characterized by a
frothy, yellow discharge. The organism can be identified in Pap smears, or, more reliably, by special culture
techniques that are becoming more widely available. Infection in men is usually asymptomatic, but prostatitis
and urethritis can also occur. Treatment with metronidazole is indicated for both the patient and the sexual
partner(s).

The other organisms listed (choices A, B, C, and D) cause gastrointestinal disease.

A previously healthy European child who had been diving into a lake develops meningoencephalitis. The disease
is rapidly fatal, despite aggressive medical therapy. Which of the following organisms is most likely to be
responsible for this child's illness?

A. Acanthamoeba sp.

B. Balantidium coli

C. Entamoeba histolytica

D. Giardia lamblia

E. Naegleria fowleri

Explanation:

The correct answer is E. Two free-living amoeba have a specific propensity for causing meningoencephalitis:
Naegleria fowleri and Acanthamoeba sp. The former is seen in the clinical setting described in the question
stem, and the infection appears to develop when amoebae in lake water are forced across the cribriform plate
in the nose during swimming and particularly diving. Since the olfactory bulbs are actually part of the brain,
crossing the cribriform plate allows the amoeba access to the meninges and brain. Diagnosis can be difficult
because the causative organism may resemble human cells when seen in cerebrospinal fluid. Unfortunately,
treatment is also difficult because the organism is not very sensitive to presently available anti-protozoan drugs.

Acanthamoeba sp. (choice A) can also cause meningoencephalitis, but the usual setting is in
immunosuppressed patients.

Balantidium coli(choice B) is a ciliated organism that occasionally causes dysentery resembling that caused by
Entamoeba histolytica.

Entamoeba histolytica(choice C) causes amoebic dysentery and liver abscesses.

Giardia lamblia(choice D) is a flagellated organism that infects the small intestine, causing diarrhea.

A sexually active 18-year-old woman presents with a fever of 102 F for the past 24 hours and lower abdominal
pain and anorexia for the past 5 days. On physical examination, there is generalized tenderness of the abdomen,
and the cervix is erythematous with motion tenderness. There is no rash nor any lesions on the external genitalia.
A smear of the odorless cervical discharge contains sloughed epithelial cells and scant neutrophils. Which of the
following would likely be found in the exudate?
94

A. A naked, icosahedral double-stranded circular DNA virus

B. Iodine-staining intraepithelial inclusion bodies

C. Intraneutrophilic gram-negative diplococci

D. Intranuclear "owl's eye" inclusion bodies

E. Lactose-fermenting gram-negative bacilli

F. Pear-shaped flagellated protozoa

G. Pleomorphic, gram-negative rods

H. Spirochetes on dark-field microscopy

Explanation:

The correct answer is B. The presentation is typical for pelvic inflammatory disease (PID). Chlamydia
trachomata (serotypes D-K) is the most common bacterial cause of sexually transmitted disease (STD) in this
country and is the most likely agent on the list to produce the symptoms described. It is an ATP-defective
organism that must therefore live intracellularly in the human host and can be visualized inside epithelial cells
with iodine, Giemsa, or fluorescent-antibody stains. The remainder of the answer choices refer to other agents
that could be found in the female genital tract, either by sexual transmission or by contamination with fecal flora,
but they are not the best choices.

A naked, icosahedral double-stranded circular DNA virus (choice A) refers to human papilloma virus, which is
the most common cause of STDs in the U.S., but presents with anogenital warts.

Intraneutrophilic gram-negative diplococci (choice C) refers to Neisseria gonorrhoeae, which would be expected
to present with dysuria and neutrophilic exudate.

Intranuclear "owl's eye" inclusion bodies (choice D) refers to cytomegalovirus, a common STD in the United
States, but not a common agent of PID. Most cases in average adults are manifested by mononucleosis-like
symptoms.

Lactose-fermenting gram-negative bacilli (choice E) would be consistent with Escherichia coli. Although this
organism is the most common cause of urinary tract infections in women in the United States, it would not be
expected to cause PID.

Pear-shaped flagellated protozoa (choice F) refers to the protozoan parasite Trichomonas vaginalis, the only
protozoan STD in the world. Infection would be characterized by a malodorous, cheesy exudate, and there
would be more erythema of the external genitalia than of the cervix.

Pleomorphic, gram-negative rods (choice G) are consistent with Hemophilus ducreyi, which causes chancroid,
and presents in a different manner.

Spirochetes on dark-field microscopy (choice H) refers to Treponema pallidum, the causative agent of syphilis,
which would produce rash and/or chancre, depending on the stage of the infection.

Which of the following microorganisms is an obligate aerobe?

A. Bacteroides fragilis

B. Clostridium perfringens
95

C. Escherichia coli

D. Mycobacterium tuberculosis

E. Pseudomonas aeruginosa

Explanation:

The correct answer is D.Mycobacterium tuberculosis is an important obligate aerobe to remember for USMLE
Step 1. (Other obligate aerobes include all other Mycobacteria, Bordetella pertussis, and Francisella
tularensis.). This organism can obtain energy only from respiration and must use oxygen as the terminal
electron acceptor. Primary infection occurs in the upper lobes of the lung.

Bacteroides fragilis(choice A) is an obligate anaerobic gram-negative bacillus that constitutes the primary
organism found in the colon. It is the most common cause of anaerobic infections, including abdominal
abscesses and peritonitis.

Clostridium perfringens(choice B), along with all the other Clostridium species, is an obligate anaerobic,
spore-forming, gram-positive bacillus. C. perfringens is an important cause of infection following trauma and
surgery and can cause gas gangrene (which can be life-threatening if not treated). C. perfringens can also
cause food poisoning.

E. coli(choice C) and P. aeruginosa(choice E) are both facultative organisms, meaning that they can grow both
in the presence or absence of oxygen. E. coli can ferment under anaerobic conditions or respire using oxygen
as the terminal electron acceptor. P. aeruginosa cannot ferment. It respires only, using nitrate as the terminal
acceptor under anaerobic conditions and oxygen as the terminal electron acceptor under aerobic conditions.

A 27-year-old woman presents to the emergency department complaining of 10-12 episodes of nonbloody
diarrhea per day for the past 2 days, along with severe abdominal cramps, nausea, vomiting, and a low-grade
fever. She states that she just returned from a vacation to Mexico. While in Mexico, she did not drink any of the
local water and ate only cooked foods and a few fresh salads. If fecal leukocytes are present, the patient should
most likely be empirically treated with

A. acyclovir

B. ciprofloxacin

C. mebendazole

D. quinine

E. tetracycline

Explanation:

The correct answer is B. When individuals travel from one country to another with marked differences in climate
and sanitation standards, the risk for development of traveler's diarrhea is dramatically increased. This type of
diarrhea is most likely to develop within 2-10 days after ingesting local water and/or eating fresh fruits and
vegetables (such as a fresh salad) that could have been "washed off" with the local water. Traveler's diarrhea is
often accompanied by the appearance of approximately 10 or more episodes of diarrhea per day, as well as
severe abdominal cramps, nausea, vomiting, and a mild fever. Depending on the causative bacteria, blood
and/or fecal leukocytes may be present. Most cases of traveler's diarrhea are caused by enterotoxigenic
Escherichia coli, Shigella species, and Campylobacter jejuni. The most commonly used agents to treat traveler's
diarrhea are the fluoroquinolones, such as ciprofloxacin, ofloxacin, and norfloxacin.
Trimethoprim/sulfamethoxazole is most commonly used in treatment of children. None of the other agents are
96
indicated for the treatment of traveler's diarrhea.

Acyclovir (choice A) is an antiviral agent indicated for the treatment of herpes virus infections.

Mebendazole (choice C) is a broad-spectrum anthelminthic indicated for the treatment of "worm" infections, such
as Enterobius vermicularis, Trichuris trichiura, and Ascaris lumbricoides.

Quinine (choice D) is an antimalarial agent indicated for the treatment of malaria and severe leg cramps.

Tetracycline (choice E) is an antibacterial agent with a limited antibacterial spectrum; it is primarily used in the
treatment of acne vulgaris and gonorrhea infections.

Which of the following types of viruses would be most likely to undergo an abrupt, major antigenic shift permitting
reinfection of previously exposed individuals?

A. Coxsackie viruses

B. Hepadna viruses

C. Herpesviruses

D. Orthomyxoviruses

E. Paramyxoviruses

Explanation:

The correct answer is D. The phrase "antigenic shift" should tip you off to look for "influenza viruses" or
"orthomyxoviruses" in the answers. The orthomyxoviruses include influenza viruses A, B, and C. These viruses
are successful at reinfecting previously exposed individuals because their genome consists of about eight (the
viral packaging process is a little vague and sometimes entraps more pieces) separate segments of RNA. Minor
antigenic changes are frequent and are called antigenic drift. Additionally, major changes, called antigenic shift
rather than antigenic drift, rarely occur, possibly as a result of double infection of cells by human and animal
influenza viruses with resulting accidental exchange of whole RNA segments. Such dramatic antigenic shifts
have occurred four times for influenza A since 1933. The other viruses listed in the answers do not undergo
such dramatic shifts.

A young mother takes her baby to the pediatrician for the first time. The doctor notices the infant's teeth have
yellow discolorations. The antibiotic this mother most likely took during pregnancy

A. inhibits aminoacyl-tRNA binding

B. inhibits peptidyl transferase

C. interferes with cell wall synthesis

D. is a large, cyclic, lactone-ring structure

Explanation:

The correct answer is A. This question relates to a USMLE favorite side effect—the teeth mottling that
occurs when a child is exposed to tetracycline in utero. You should remember that tetracycline is
97
contraindicated in pregnancy and early childhood. Tetracycline is a bacteriostatic drug that binds to the 30s
subunit of ribosomes, preventing aminoacyl-tRNA from binding with complementary mRNA. This inhibits peptide
bond synthesis. Resistance is plasmid mediated.

Inhibition of peptidyl transferase (choice B) occurs with chloramphenicol, a broad-spectrum bacteriostatic agent
that binds to the 50s subunit of ribosomes. Resistance is plasmid mediated. It has high toxicity (GI disturbances,
aplastic anemia, and gray baby syndrome), so it is used mainly in severe infections or as a topical agent.

Interference with cell wall synthesis (choice C) occurs with penicillins and cephalosporins, the beta-lactam
antibiotics. Resistance to these drugs appears in organisms that have developed beta-lactamases
(penicillinases), enzymes that destroy the beta-lactam ring of these medications. The wider spectrum ampicillin,
amoxicillin, ticarcillin, and carbenicillin are particularly penicillinase susceptible.

Large, cyclic, lactone-ring structures (choice D) describe the macrolides: erythromycin, azithromycin, and
clarithromycin. They inhibit bacterial protein synthesis by reacting with the 50s ribosomal subunit and
preventing the release of the uncharged tRNA. Resistance is plasmid mediated. Common side effects include GI
irritation, skin rashes, and eosinophilia. Erythromycin is a popular choice for patients with penicillin
hypersensitivity. It is a cytochrome p450 inhibitor and therefore must be used with caution in patients taking
other drugs.

A 7- year-old girl develops behavioral changes, and her performance in school begins to deteriorate. Several
months later she develops a seizure disorder, ataxia, and focal neurologic symptoms. She is eventually
quadriparetic, spastic, and unresponsive. Death occurs within a year. This patient may have had which of the
following viral diseases at 1 year of age?

A. Chickenpox

B. German measles

C. Measles

D. Mumps

E. Parvovirus B19

Explanation:

The correct answer is C. The child has subacute sclerosing panencephalitis (SSPE), which is fortunately a very
rare, late complication of measles infection at an early age. The existence of this complication is part of the
rationale to immunize children to measles at an early age. It is thought that very young children's immune and
neurologic systems may permit the virus to become established in the brain. The exact mechanism of injury is
poorly understood, but the brain shows encephalitis involving both gray and white matter. 40% of cases die
within 1 year, and it is thought that the disease is probably always eventually fatal.

None of the other diseases progresses to SSPE.

A 32-year-old man from Yuma, Arizona presents to a hospital with complaints of a 4-day history of fever,
myalgias, and cough. He owns an automobile repair shop and had recently cleaned his garage, which was
infested by mice. On physical examination, he is tachypneic and must use accessory respiratory muscles to
breathe. Shortly following admission, he is intubated and diagnosed with adult respiratory distress syndrome.
What underlying infection should be considered?

A. California encephalitis
98

B. Cytomegalovirus

C. Hantavirus

D. Poliovirus

E. Rabies

Explanation:

The correct answer is C. Hantavirus pulmonary syndrome, first reported in 1993, consists of fever, myalgia, and
rapid development of respiratory failure leading to death. The initial cases were mostly among the Navajo
Indians; however, cases have been documented in many states and in Canada. It is most common in New
Mexico, Arizona, Colorado, and Utah. The causative agent is a Hantavirus belonging to the family Bunyaviridae.
The virus is now called Sin Nombre (meaning "without a name") virus. Infection is thought to be through
inhalation of aerosolized secretions from the common deer mouse. The disease typically begins with a
non-specific prodrome (fever, generalized myalgia, and gastrointestinal disturbances) followed 4 to 5 days later
by respiratory symptoms (cough, dyspnea, and tachypnea). This progresses rapidly to an adult respiratory
distress syndrome and, in many cases, death.

California encephalitis (choice A) is caused by an arbovirus transmitted by infected mosquitoes. Most cases are
in children less than 10 years of age. Signs and symptoms include fever, headache, photophobia, nausea, and
vomiting. Myalgias and arthralgias are typically present. Seizure activity is most frequently observed in infants
and young children. Most cases are in the midwestern U.S.

Cytomegalovirus (choice B) is a herpesvirus producing a variety of infections. Primary CMV infection is usually
asymptomatic in immunocompetent patients, but can cause a heterophile-negative mononucleosis syndrome.
Perinatal infections can occur in utero, intrapartum, or postpartum and can produce congenital malformations.
CMV can be transmitted by granulocytes in blood transfusions. In patients with AIDS or other
immunosuppressive states, CMV can produce severe disease, including retinitis, pneumonia, encephalitis,
adrenalitis, and gastrointestinal tract ulcerations.

Poliovirus (choice D) is a member of the Picornavirus family. It remains epidemic in parts of Asia and Africa even
though the wild-type has been eliminated from the western hemisphere. The virus affects cranial nerve nuclei
and anterior horn motor neurons of the spinal cord, producing a flaccid paralysis, which is usually asymmetric.
Vaccine-related polio can occur with the live virus vaccine.

Rabies (choice E) is a Rhabdovirus that produces encephalitis and/or myelitis. It should be considered in
persons who have recently traveled outside of the U.S. The most common sources of exposure are dogs, cats,
skunks, foxes, raccoons (in Florida and Connecticut), wolves, and bats. The infection is acquired by the bite of
a rabid animal or by inhalation. Rabies also has been reported to occur in patients after corneal transplantation.
The virus spreads along peripheral nerves to the central nervous system. Symptoms include hydrophobia and
copious salivation.

A 55-year-old man returns from a vacation to the beach. While on vacation, he and his family consumed several
meals consisting primarily of seafood, including lobster and raw oysters. Although the other members of his family
did not get sick, he developed a bullous rash over his lower extremities, hypotension, and confusion. His past
medical history is significant for liver disease and alcoholism. The most likely cause of this man's signs and
symptoms is

A. Aspergillus fumigatus

B. Campylobacter jejuni

C. Candida albicans

D. Streptococcus (Group A)
99
E. Vibrio vulnificus

Explanation:

The correct answer is E.Vibrio vulnificus septicemia is contracted by consuming raw oysters. Patients with
chronic renal insufficiency, liver disease, hematopoietic disorders, and a past history of alcoholism are
particularly at risk. His past medical history makes him more susceptible to septicemia than are his family
members. Septicemia cause by Vibrio vulnificus generally begins with chills, fever, and hypotension, and skin
lesions tend to occur 24-48 hours after the onset of the infection.

Aspergillus fumigatus(choice A) is an opportunistic mold responsible for infections of wounds and burns in
immunocompetent hosts; in immunocompromised hosts, it can invade visceral organs such as the lungs.

Campylobacter jejuni(choice B) are curved, gram-negative rods that cause enterocolitis with diarrhea, and, less
commonly, chronic gastritis.

Candida albicans(choice C) is a common opportunistic yeast that is found as normal flora of upper respiratory,
gastrointestinal, and vaginal mucosae. It ordinarily causes thrush and vulvovaginitis; it may disseminate in the
immunocompromised patient.

Group A streptococcus (Streptococcus pyogenes; choice D) is an important cause of bacterial pharyngitis


(Strep throat).

A 24-year-old female has fever, malaise, and a dry, nonproductive cough. She also complains of headache,
muscle aches, and leg pain. Lab values are significant for elevated cold agglutinins. Which of the following
microorganisms is responsible for her symptoms?

A. Haemophilus influenzae

B. Klebsiella pneumoniae

C. Legionella pneumophila

D. Mycoplasma pneumoniae

E. Streptococcus pneumoniae

Explanation:

The correct answer is D.Mycoplasma pneumoniae is a wall-less bacterium that causes interstitial pneumonia in
young adults. Elevated cold agglutinins (a classic clue) are found in about half of the patients. The cold
agglutinins are IgM antibodies. M. pneumoniae may be diagnosed by sputum or complement fixation.

Haemophilus(choice A) causes bronchopneumonia in babies and children, and may occur in debilitated adults.

Klebsiella(choice B) causes a bronchopneumonia with patchy infiltrates involving one or more lobes. Think red
currant jelly sputum. It frequently occurs in debilitated patients, diabetics, and alcoholics. Note that these
organisms are highly encapsulated and produce mucoid colonies on lab media.

Legionella(choice C) lives in contaminated water sources such as air conditioning systems.

Streptococcus pneumoniae(choice E) is the most common cause of pneumonia in the elderly population and in
those with poor nutrition. It may lead to a pleural abscess and rusty-colored sputum; it is also a common cause
of sepsis and meningitis in the elderly.
100

A Native American man is brought to a rural hospital in New Mexico. On arrival, he is unconscious with severe
bronchopneumonia. Family members state that he suffered the sudden onset of chills, fever, and headache
several days ago. One day later, the man complained of chest pain and difficulty breathing, and coughed up
blood-tinged sputum. Chest x-ray reveals patchy infiltrates and segmental consolidation. Which of the following
organisms is the most likely cause of this man's pneumonia?

A. Brucella abortus

B. Clostridium perfringens

C. Francisella tularensis

D. Listeria monocytogenes

E. Yersinia pestis

Explanation:

The correct answer is E. Any previously healthy person in the Southwestern United States who develops septic
shock or severe pulmonary disease should be evaluated for plague. Plague is not an extinct disease, but is still
encountered in sporadic cases in various places in the world, including Asia, Africa, parts of Europe, and the
American Southwest. The causative organism is Yersinia pestis, which is endemic in many wild animal
populations, and can be transmitted to humans either by direct contact or by arthropod bite. Human plague may
take many forms, including pestis minor (mild lymphadenopathy); bubonic plague (prominent
lymphadenopathy); pneumonic plague (as in this patient); and septicemic plague. The primary pneumonic form
typically presents as described. Antibiotics are most effective if given within the first 24 hours, which can be
problematic if medical staff do not suspect the disease. Since plague is rare in the United States, a high degree
of clinical suspicion is required to make a rapid diagnosis and to institute timely treatment. If the diagnosis is
missed, the mortality rate is quite high.

Brucella abortus(choice A) causes brucellosis, characterized by undulating fever, lymphadenopathy, and


hepatosplenomegaly.

Clostridium perfringens(choice B) causes gas gangrene and gastroenteritis.

Francisella tularensis(choice C) causes tularemia, associated with a spectrum of manifestations from an


influenza-like syndrome to adenopathy with ulceration at the site of inoculation.

Listeria monocytogenes(choice D) causes listeriosis. Infection during pregnancy may result in sepsis, abortion
or premature delivery. Infection in the neonate may produce meningitis. In immunocompromised adults, either
meningitis or sepsis may occur.

An elderly patient has pneumococcal pneumonia. She has no known drug allergies. Which of the following
antibiotics would be most appropriate?

A. Chloramphenicol

B. Cefotaxime

C. Erythromycin

D. Penicillin

E. Vancomycin
101

Explanation:

The correct answer is D. Penicillin remains the first-line drug of choice for pneumococcal pneumonia, except in
those patients with penicillin allergy and in the relatively few areas in which pneumococcal strains with high-level
penicillin resistance are found. Alternative therapies include erythromycin and vancomycin.

Chloramphenicol (choice A) is not usually used for pneumococcal pneumonia.

The third generation cephalosporin cefotaxime (choice B) is not usually used for pneumococcal pneumonia.

Erythromycin (choice C) is a good alternative therapy for pneumococcal pneumonia, but is usually used only
when a penicillin allergy is present.

Vancomycin (choice E) is not the first-line therapy, but it is a good alternative in patients allergic to penicillin or
when high-level penicillin resistance (relatively uncommon) is present.

A 30-year-old veterinarian visits her obstetrician for a first-trimester prenatal check-up. She has no complaints.
Routine physical exam is significant only for mild cervical lymphadenopathy. She is prescribed spiramycin but is
noncompliant. Her baby is born with hydrocephalus and cerebral calcifications. Which of the following organisms is
most likely responsible?

A. Isospora belli

B. Leishmania donovani

C. Plasmodium vivax

D. Toxoplasma gondii

E. Trypanosoma cruzi

Explanation:

The correct answer is D. Humans become infected with Toxoplasma gondii by ingesting cysts in contaminated
food or through contact with cat feces. The veterinarian in question was therefore particularly at risk of infection.
T. gondii is especially hazardous in pregnant women because the organism can be transmitted to the fetus
through the placenta. (It is part of the ToRCHeS group of congenital infections–Toxoplasma, Rubella,
CMV, Herpes/HIV, Syphilis). Since infected mothers are usually asymptomatic, cases often go unnoticed.
Occasionally, patients present with cervical lymphadenopathy, as did the veterinarian, and require treatment to
prevent complications in the fetus. Though newborns are also often asymptomatic, they are at risk for
developing the classic triad of chorioretinitis (at birth or later in life), hydrocephalus, and cerebral calcifications.
Note that T. gondii is also a common cause of CNS infections (e.g., encephalitis) in HIV-positive patients.

Isospora belli(choice A) is an intestinal protozoan that causes watery diarrhea, particularly in the
immunocompromised. Fecal-oral transmission of oocysts allows invasion of small intestinal mucosa, destroying
the brush border.

Leishmania donovani(choice B) causes kala-azar (visceral leishmaniasis), which is characterized by fever,


weakness, weight loss, splenomegaly, and skin hyperpigmentation. It is prevalent in regions of the
Mediterranean, Middle East, Russia, and China. The vector is the sandfly.

Plasmodium vivax(choice C) causes malaria and is transmitted by the female Anopheles mosquito, which
introduces sporozoites into the blood. These differentiate into merozoites that destroy erythrocytes.
Splenomegaly ensues. Other species of this organism also cause malaria: P. malariae and P. falciparum (which
102
causes a more severe form of the disease). Note that sickle cell trait confers resistance to this disease.

Trypanosoma cruzi(choice E) causes Chagas' disease, characterized by facial edema and nodules, fever,
lymphadenopathy, and hepatosplenomegaly. It affects cardiac muscle most severely and is a major cause of
cardiac disease worldwide. The reduviid ("kissing") bug is the vector that infects humans through bites. It is most
prevalent in Central and South America, with rare cases in the southern US.

A 30-year-old veterinarian on a cattle ranch presents with a 1-to-2-month history of malaise, chills, drenching
malodorous sweats, fatigue, and weakness. He has anorexia and has lost 15 pounds. He has intermittent fevers
that range up to 103 F (39.4 C). He complains of visual blurring. A physical examination reveals mild
lymphadenopathy, petechiae, and a cardiac murmur consistent with aortic insufficiency. What is the most likely
etiologic agent?

A. Bacillus anthracis

B. Brucella abortus

C. Coccidioides immitis

D. Erysipelothrix rhusiopathiae

E. Trichinella spiralis

Explanation:

The correct answer is B.Brucella abortus produces a chronic, granulomatous disease with caseating
granulomas. Most cases occur in four states (Texas, California, Virginia, and Florida), and are associated with
cattle, in which it produces spontaneous septic abortions. Most cases of brucellosis produce mild disease or
fevers of unknown origin. However, Brucella spp. can infect the cardiovascular system and cause a localized
infection. B. abortus is the most common species to cause endocarditis. The aortic valve is most commonly
involved, followed by the mitral valve, and then both valves. Most cases of brucellosis are associated with
occupational exposure, in persons such as veterinarians, ranchers, and those who handle carcasses.

Bacillus anthracis(choice A) is the causative agent for anthrax. It usually produces cutaneous disease
(malignant pustule or eschar) at the site of inoculation in handlers of animal skins. It can also produce a severe
hemorrhagic pneumonia (Woolsorter's disease) and septicemia. At-risk groups include those who handle animal
carcasses or skins.

Coccidioides immitis(choice C) is a dimorphic fungal disease producing a granulomatous pulmonary syndrome


that is more severe in dark-skinned individuals. Disseminated disease occurs most often in Filipinos, Mexicans,
and Africans. The infective form is the arthrospore; the diagnostic form in tissue is the spherule containing
endospores. The disease is endemic in the San Joaquin River Valley. At-risk groups include military personnel,
agricultural workers, construction workers, oil field workers, archaeology students, participants in outdoor
sports, and sightseers. Remote infections from fomites (cotton harvested in the Southwestern U.S.) have been
reported.

Erysipelothrix rhusiopathiae(choice D) is a pleomorphic, gram-negative rod that causes a localized skin


infection. It is an occupational disease of fishermen, fish handlers, butchers, meat-processing workers, poultry
workers, farmers, veterinarians, abattoir workers, and housewives.

Trichinella spiralis(choice E) is a nematode infection caused by the ingestion of larvae found in undercooked
meat. Pork is the most common contaminated meat. However, outbreaks in the northern parts of the U.S. have
been associated with eating undercooked infected bear meat. Symptoms include diarrhea, periorbital edema,
myositis, fever, and eosinophilia.
103

Six days after receiving several flea bites in a rat-infested shed in Southeastern New Mexico, a homeless
24-year-old-man develops fever, chills, and a rash that spreads from his abdomen to cover his extremities. He is
seen at the hospital emergency department, where blood is drawn for analysis. Eight days later, the public health
department reports the presence of antibody to one of the rickettsial group antigens. Which of the following is
the most likely diagnosis?

A. Endemic typhus

B. Epidemic typhus

C. Q fever

D. Rocky Mountain spotted fever

E. Scrub typhus

Explanation:

The correct answer is A. Endemic typhus is caused by Rickettsia typhi and is found worldwide. It is spread by
the feces of the rat flea, and its reservoir is the rat.

Epidemic typhus (choice B) is cause by Rickettsia prowazekii. It is prominent during times of war and social
upheaval and is spread by the feces of the body louse, Pediculus humanis. Humans are the chief reservoir,
although it is also found in populations of flying squirrels along the Atlantic coast of the U.S.

Q fever (choice C) is caused by Coxiella burnetii of the family Rickettsiaceae. Unlike the other rickettsial
illnesses, Coxiella burnetii is not usually transmitted to humans by the bite of an arthropod. Since the organism
is resistant to dehydration, it can be acquired by inhaling dust contaminated by animals with asymptomatic
infections, such as goats, sheep, and cattle.

Rocky Mountain spotted fever (RMSF) (choice D) is caused by Rickettsia rickettsii. It is transmitted by the bite
of an infected tick of the genus Dermacentor. Since the tick passes the infection transovarially to its progeny,
the tick is a reservoir. Rodents and other mammals are also reservoirs. Despite the name, most cases of RMSF
occur in the central states and southern Atlantic seaboard states. Oklahoma has the highest incidence of the
disease.

Scrub typhus (choice E) is caused by Rickettsia tsutsugamushi. It occurs mainly in Southeast Asia, where it is
transmitted by mites. Mites and rodents are the reservoirs for the organism.

A 54-year-old diabetic patient reports to his physician's office complaining of an unresolved skin lesion on his
foot. The lesion began several weeks ago as a blister and has since become a painful, erosive, expanding sore.
On examination, the affected site is now 5 cm in diameter, with a black necrotic center and raised red edges.
Which of the following toxins has a mechanism of action most similar to the toxin responsible for tissue damage in
this patient?

A. Anthrax toxin

B. Botulinum toxin

C. Cholera toxin

D. Clostridium perfringens alpha toxin

E. Diphtheria toxin
104
F. Escherichia coli labile toxin

G. Pertussis toxin

H. Shigatoxin

I. Streptococcal erythrogenic toxins

J. Tetanus toxin

K. Toxic shock syndrome toxin-1

Explanation:

The correct answer is E. This patient's wound is infected with Pseudomonas aeruginosa, and the characteristic
lesion described is called ecthyma gangrenosum. The Pseudomonas alpha toxin, which is responsible for the
tissue damage, inhibits protein synthesis by acting on EF-2 with a primary target cell in the liver. The diphtheria
toxin has a similar action, although its target cells are heart and nerve.

Anthrax toxin (choice A) is an adenylate cyclase that causes fluid loss from cells.

Botulinum toxin (choice B) is a neurotoxin that decreases acetylcholine synthesis.

Cholera toxin (choice C) acts to increase adenylate cyclase activity by ribosylation of GTP-binding protein.

Clostridium perfringens alpha toxin (choice D) is a lecithinase.

Escherichia coli labile toxin (choice F) works in a fashion similar to the cholera toxin.

Pertussis toxin (choice G) causes fluid loss by ribosylating Gi.

Shigatoxin (choice H) decreases protein synthesis by inhibiting the 60S ribosomal subunit.

Streptococcal erythrogenic toxins (choice I) act similarly to the diphtheria toxin, but do so by increasing cytokine
production.

Tetanus toxin (choice J) is a neurotoxin that inhibits the inhibitory neurotransmitters glycine and GABA.

TSST-1 (choice K) is a superantigen that acts by increasing cytokine production and decreasing liver clearance
of endotoxin.

An 18-year-old, previously healthy female presents to the student health service with fever, vomiting, and
diarrhea. On physical examination, she is hypotensive and has an erythematous, red, sunburn-like skin rash. She
is currently menstruating, and has been using super absorbent tampons. Which of the following findings from a
positive blood culture would confirm your suspected diagnosis?

A. Organisms are acid-fast

B. Organisms are coagulase positive

C. Organisms grow on EMB (eosin-methylene blue) agar

D. Organisms grow on Thayer-Martin media

E. Organisms have positive Quellung reaction


105
Explanation:

The correct answer is B. This is a multi-step microbiology question that requires you to diagnose the illness,
identify the microorganism, and remember its key feature. The first part should be easy: everything about this
vignette suggests toxic shock syndrome. The organism in question is therefore Staphylococcus aureus, which is
coagulase positive. All of the other choices are classic features of other important pathogenic microorganisms:

Acid-fast organisms (choice A), refers to Mycobacteria. (In addition, Nocardia species are partially acid fast).

EMB agar (choice C) refers to a selective and differential medium used to isolate and identify enteric
gram-negative bacteria. Gram-positive bacteria will not grow on EMB agar because the addition of eosin inhibits
their growth. Nonlactose fermenters will have colorless colonies, while fermentation of this sugar will cause the
colonies to appear pink or purple.

Thayer-Martin media (choice D) is a growth medium for pathogenic Neisseria species. It contains the antibiotic
vancomycin, which kills gram-positive organisms such as Staphylococcus aureus.

The Quellung reaction (choice E) can be used to identify the capsule type of a microorganism. Encapsulated
microorganisms, like the Pneumococci and Haemophilus, are mixed with specific antisera. If the antiserum is
directed against the microorganism's capsule type, the capsule will be opsonized, absorb water, and become
visible under a light microscope.

A 24-year-old woman presents with a 3-day history of fever, chills, chest pain, and cough productive of
rust-colored sputum. Past medical history includes a splenectomy 1 year ago. A chest x-ray film indicates
consolidation of the right lower lobe. Blood cultures are positive for α-hemolytic gram-positive diplococci.
Immunity to the causative organism is based on

A. alternative complement pathway activation

B. antibody to an α-helical coiled fimbria

C. IgA antibodies to C carbohydrate

D. IgG antibodies to C carbohydrate

E. IgG antibodies to a surface acidic polysaccharide

Explanation:

The correct answer is E. The patient in this question has pneumococcal pneumonia, which must be
considered in any patient with chills, fever, chest pain, and cough productive of purulent, rust-colored sputum.
Streptococcus pneumoniae is an α-hemolytic, gram-positive coccus that grows in chains. It can be easily
distinguished from other α-hemolytic streptococci because it is exquisitely sensitive to bile and bilelike
compounds, such as optochin. It is the most common cause of community-acquired pneumonia and the most
common cause of community-acquired meningitis in adults older than 30. The only recognized virulence factor
of S. pneumoniae is its carbohydrate capsule (which contains acidic polysaccharides). Antibody to a specific
capsule type is necessary to overcome infection. More than 80 capsule types have been recognized. The 23
types that most commonly cause disease are contained in a vaccine that is recommended for high-risk groups,
including the elderly and those undergoing splenectomy. Increased susceptibility is also found in patients with
Hodgkin disease, chronic lymphocytic leukemia, and myeloma. The vaccine should still be given to patients with
these conditions, but it is less successful.

The alternative complement pathway (choice A) is important in clearing Neisseria infections. Individuals with
deficiencies in C5 through C8 are at increased risk of disease from Neisseria.

The fimbria of Streptococcus pyogenes (Group A α-hemolytic streptococcus) is composed of an


α-helically coiled M protein. Antibody against a specific M type (choice B) will prevent infection. However,
raising antibodies to M proteins can lead to rheumatic fever, so Strep throat infections are routinely treated with
106
penicillin to prevent an antibody response.

The C carbohydrate is an antigen of α-hemolytic streptococci used to divide them into different groups.
Antibody against C carbohydrate (choices C and D) is not protective.

A 3-year-old boy presents with a 1-day history of loose stools, fever, abdominal cramping, headache, and
myalgia. He has no blood in the stool. A careful history reveals that he has several pet turtles. Which of the
following is most likely the causative agent of his diarrhea?

A. Chlamydia psittaci

B. Entamoeba histolytica

C. Salmonella spp.

D. Staphylococcus aureus

E. Yersinia enterocolitica

Explanation:

The correct answer is C. Salmonella spp., including S. enteritidis and S. typhimurium, produce a gastroenteritis
or enterocolitis. Patients with decreased gastric acidity, sickle cell disease, defects in immunity, or children
younger than 4 years have a more severe course of disease. Salmonella spp. are carried in nature by animal
reservoirs such as poultry, turtles, cattle, pigs, and sheep. The incubation period is 8-48 hours after ingestion
of contaminated food or water.

Chlamydia psittaci(choice A) produces an interstitial pneumonitis accompanied by headache, backache, and a


dry, hacking cough. A pale, macular rash is also found on the trunk (Horder's spots). Patients at risk include pet
shop workers, pigeon handlers, and poultry workers.

Entamoeba histolytica(choice B) produces a diarrhea (frequently bloody or heme-positive), right lower quadrant
crampy abdominal pain, and fever. Patients frequently have weight loss and anorexia. There is usually a history
of travel outside the U.S. Most cases are chronic. Complications include liver abscesses.

Staphylococcus aureus(choice D) produces a self-limited gastroenteritis due to the production of preformed,


heat-stable enterotoxins. The incubation period is 16 hours. The toxins enhance intestinal peristalsis and
induce vomiting by a direct effect on the CNS.

Yersinia enterocolitica(choice E) usually produces a chronic enteritis in children. These patients have diarrhea,
failure to thrive, hypoalbuminemia, and hypokalemia. Other findings include acute right lower quadrant
abdominal pain, tenderness, nausea, and vomiting. The infection mimics appendicitis or Crohn's disease.

Bilateral tonsillectomy is performed on an otherwise healthy 11-year-old female with recurrent upper respiratory
tract infections. On sectioning the tonsils, numerous small, yellow granules are noted. A granule crushed
between two slides has a dense, gram-positive center and numerous branching filaments at the periphery. The
granules are most likely composed of which of the following organisms?

A. Actinomyces israelii

B. Aspergillus fumigatus

C. Blastomyces dermatitidis

D. Candida albicans
107

E. Corynebacteria diphtheriae

Explanation:

The correct answer is A. Actinomyces are normal inhabitants of the gastrointestinal tract that grow under
anaerobic and microaerophilic conditions. Although they are gram-positive rods, they grow as branching
filaments and have been confused with fungi. The yellow colonies (sulfur granules) are found in low-oxygen
niches like the tonsils and in actinomycotic abscesses.

Aspergillus fumigatus(choice B)may be present in the respiratory tract as an opportunistic pathogen; however,
fungus balls are generally seen only in pre-existing cavities (e.g., bronchiectasis, TB), not in the tonsils.

Blastomyces dermatitidis(choice C) is a respiratory pathogen that is seen as thick-walled yeasts within


granulomas.

Candida albicans(choice D), also a normal inhabitant of the oral cavity, would present as whitish plaques and
would appear microscopically as budding yeasts.

Diphtheria, caused by Corynebacteria diphtheriae(choice E), is a gram-positive rod. The disease presents with
a gray-white membrane in the oropharynx, and large colonies would not be appreciated.

A patient with a cavitary lung lesion coughs up sputum that contains thin, acid-fast positive rods. Which of the
following features would most likely be associated with these bacteria?

A. Nutritional requirement for factors V and X

B. Streptokinase

C. Toxic shock syndrome toxin

D. Visible under dark field illumination

E. Waxy envelope

Explanation:

The correct answer is E. Mycobacteria, such as the causative organism of this patient's tuberculosis, are "acid
fast" because they have an envelope that contains large amounts of lipids and even true waxes (unlike
envelopes of other types of bacteria) that prevents the acid-fast stain (carbolfuchsin) from leaking out.

The other characteristics listed in the answers are commonly tested features of specific bacteria:

Nutritional requirement for factors V and X (choice A) is a feature of Haemophilus influenzae.

Streptokinase (choice B) is a feature of Streptococci.

Toxic shock syndrome toxin (choice C) is a feature of Staphylococcus aureus.

Visibility under dark field illumination (choice D) is a feature of the syphilis organism Treponema pallidum.

An 8-year-old boy is brought to the emergency room with a 3-day history of fever to 102 degrees F and
abdominal pain. He also complains of pain in his right knee and right elbow. He was seen four weeks ago
108
because of a sore throat and a rash. A throat culture performed at that time grew gram-positive cocci in chains.
Amoxicillin was prescribed, but the boy's mother did not fill the prescription. On physical exam his temp is 101.7F,
HR 96, and BP 100/60. Cardiac exam reveals a pansystolic blowing murmur heard best at the apex. His right
elbow is tender on extension and flexion with mild swelling. Laboratory tests reveal a positive C-reactive protein,
an ESR of 40 and a WBC of 22,000 with a left shift. EKG shows a prolonged PR interval. Which of the following
tests would be positive for the microorganism responsible for this patient's illness?

A. Catalase test

B. Coagulase test

C. Sensitivity to bacitracin

D. Sensitivity to novobiocin

E. Sensitivity to optochin

Explanation:

The correct answer is C. This case is classic for Rheumatic fever, including two of the major Jones criteria
(carditis, polyarthritis) and several minor criteria (fever, arthralgia, elevated ESR, leukocytosis, C-reactive
protein, prolonged PR interval). Rheumatic fever is a sequela of untreated infection with Group A
Streptococcus (S. pyogenes). S. pyogenes is differentiated from the other beta-hemolytic Strep by its sensitivity
to the antibiotic bacitracin.

The catalase test (choice A) is used to differentiate Staphylococci from Streptococci. Staphylococci are
catalase positive, Streptococci are catalase negative. So, S. pyogenes would be catalase negative.

The coagulase test (choice B) is used to differentiate Staph aureus from the other Staph spp. Staph aureus is
coagulase positive; the others are negative.

Sensitivity to novobiocin (choice D) is used to differentiate Staph saprophyticus (resistant) from Staph
epidermidis (sensitive).

Sensitivity to optochin (choice E) is used to differentiate Strep pneumoniae (sensitive) from viridans Strep
(resistant).

An immigrant from the Far East develops malaise, fever, and rigors, followed by upper right quadrant abdominal
pain, vomiting, jaundice, and itching. His urine is dark and his feces are pale. Infestation with which of the
following parasites is most strongly suggested by this patient's presentation?

A. Clonorchis sinensis

B. Enterobius vermicularis

C. Plasmodium ovale

D. Taenia solium

E. Trypanosoma cruzi

Explanation:

The correct answer is A. The patient is suffering from suppurative (bacterial) cholangitis, which can occur as a
109
complication of infestation by the roundworm, Ascaris lumbricoides and by the liver flukes, Clonorchis sinensis
and Fasciola hepatica. Biliary tract obstruction produces jaundice and extreme itching, with dark urine and pale
feces. Therapy typically includes emergency endoscopic sphincterectomy to improve biliary drainage,
antibiotics, and anthelminthic agents.

Enterobius vermicularis(choice B) causes pinworm infections.

Plasmodium ovale (choice C) causes malaria.

Taenia solium (choice D) is the pork tapeworm. Adult tapeworms cause taeniasis, while the larvae are
responsible for cysticercosis.

Trypanosoma cruzi(choice E) causes Chagas' disease.

A 38-year-old woman vacationing in Connecticut is bitten by a tick. She does not seek medical treatment and
eventually develops chronic arthritis of the knee and hip joints and paralysis of the left facial muscles. A physical
examination during the early stages of the disorder would most likely have revealed

A. aphthous ulcers in the mouth

B. erythema chronicum migrans

C. flaccid paralysis of limb flexors

D. purpuric lesions in a bathing trunk distribution

E. spastic paralysis of limb extensors

Explanation:

The correct answer is B. Lyme disease should be suspected in a patient who is bitten by a tick in the
northeastern U.S. Lyme disease was named after a township in eastern Connecticut where the disease was
endemic. The disease is spread via a tick vector of the genus Ixodes, which transmits a spirochete that causes
a systemic illness. Erythema chronicum migrans is usually the first sign of the illness. This is a large red patch
on the buttocks or chest that slowly expands as the center blanches. Generally, patients also have
constitutional symptoms, such as fever and chills, during this phase. Stiff neck may develop, along with other
signs of meningeal irritation, because of an aseptic meningitis. Other neurologic complications of Lyme disease
include Bell's palsy due to involvement of branches of the facial nerve. Arthritis is a prominent feature in about
half the patients with Lyme disease. It tends to appear several months after the infection but may persist for
several years. The course of the chronic arthritis shows exacerbations and remissions; the most commonly
affected joints are the knees and hips. Cardiac abnormalities in Lyme disease include pericarditis and heart
block.

Skin manifestations do not include aphthous ulcers (choice A).

Flaccid or spastic paralysis of limbs (choices C and E) does not accompany Lyme disease; neurologic
involvement is generally limited to cranial nerves and meningitis.

Purpura (choice D) is associated with vasculitis and does not occur in Lyme disease.

Global eradication of Lyme disease is unlikely because

A. Borrelia burgdorferi can be maintained in nature indefinitely by a tick vector

B. Borrelia burgdorferi is resistant to antibiotics and disinfectants


110

C. Borrelia burgdorferi is resistant to environmental stresses

D. human disease may reactivate after the primary infection (Brill-Zinsser disease)

E. humans are the primary reservoir for Borrelia burgdorferi

Explanation:

The correct answer is A.Borrelia burgdorferi can be maintained in nature indefinitely by a tick vector. This
organism is the tick-transmitted spirochete that causes Lyme disease. The tick, Ixodes dammini, can infect both
the white-footed mouse and large mammals such as deer during its life cycle, making them reservoirs. The tick
itself is a reservoir, however, since it acquires the disease through transovarial passage of the organism.
Together, these factors make Lyme disease an endemic infection with little hope for eradication.

Borrelia burgdorferi is not resistant to antibiotics and disinfectants (compare with choice B). The spirochete can
be successfully treated with penicillins, tetracycline, and ceftriaxone.

Borrelia burgdorferi is a delicate spirochete and is not resistant to environmental stresses (compare with choice
C).

Brill-Zinsser disease (choice D) is the reactivation of epidemic typhus infection caused by Rickettsia prowazekii.
It can occur many years after an infection that was not treated with antibiotics.

Humans are incidental hosts, rather than the primary reservoir, for Borrelia burgdorferi(choice E). The primary
reservoirs are ticks, mice, and large mammals.

A 54-year-old farmer in rural Pennsylvania presents to his physician with chronic cough. Chest x-ray
demonstrates a mass lesion with hilar lymphadenopathy. Biopsy of the mass demonstrates multiple, tiny yeast
forms within macrophages. Which of the following is the most likely diagnosis?

A. Blastomycosis

B. Coccidioidomycosis

C. Histoplasmosis

D. Paracoccidioidomycosis

E. Sporotrichosis

Explanation:

The correct answer is C. Histoplasmosis is caused by Histoplasma capsulatum, a dimorphic fungus that grows as
a mold in the wild, but as a tiny yeast inside macrophages in humans. The disease is typically asymptomatic or
mild enough to go undetected, but when symptomatic, it presents with cough, fever, and malaise.

Blastomycosis (choice A) is characterized by a larger, round-budding yeast form, seen free in the tissues.

Coccidioidomycosis (choice B) is seen mainly in the desert parts of the Southwest U. S. In the lungs, spherules
containing endospores are seen.

Paracoccidioidomycosis is endemic in Latin America, especially in Brazil. The infected cells show a typical "pilot's
wheel" appearance due to multiple yeasts sprouting out of a single parent cell (choice D).
111
Sporotrichosis (choice E) most often produces a localized cutaneous infection, following inoculation
occurring in
association with minor skin trauma during gardening.

Two weeks after birth, a neonate develops sepsis, skin vesicles, and conjunctivitis. Over the next several days,
the baby's condition deteriorates with development of seizures, cranial nerve palsies, and lethargy. The baby dies
approximately one week after onset of symptoms. Which of the following infectious agents would most likely cause
this clinical presentation?

A. Cytomegalovirus

B. Herpes simplex

C. Rubella

D. Syphilis

E. Toxoplasmosis

Explanation:

The correct answer is B. All of the agents listed, including choices A, C, D and E, can cause devastating
congenital infections with high mortality and often with major organ malformation (the TORCH agents:
Toxoplasma, other, rubella, cytomegalovirus, herpes simplex). However, it is herpes simplex type II, typically
acquired during delivery, that causes the devastating neonatal encephalitis described in the question stem. The
mortality rate for neonatal herpes is about 65%, and only 10% of the babies escape without neurologic
sequelae. A point worth remembering is that adult herpes encephalitis (in non-immunosuppressed individuals) is
usually due to herpes simplex I, while neonatal and congenital herpes are usually due to herpes simplex II.

A one-week-old baby develops nuchal rigidity and fever. A lumbar puncture is performed and the cerebrospinal
fluid demonstrates large numbers of neutrophils. Which of the following is the most likely causative agent?

A. Coxsackievirus

B. Escherichia coli

C. Herpes virus

D. Mycobacterium tuberculosis

E. Neisseria meningitidis

Explanation:

The correct answer is B. The nuchal rigidity (stiff neck) suggest meningitis. Numerous neutrophils in the
cerebrospinal fluid suggests a bacterial pathogen. The best answer of those listed is Escherichia coli, which is
normally a gut organism, but can infect neonates who acquire the organism during passage through the birth
canal.

Coxsackievirus (choice A) is a cause of acute lymphocytic meningitis.

Herpes virus (choice C) is a cause of lymphocytic meningitis.


112
Mycobacterium tuberculosis(choice D) is a cause of chronic meningitis.

Neisseria meningitidis(choice E) causes bacterial meningitis, usually in the second or third decade of life.

A patient presents to a physician because of pain during defecation accompanied by blood in the stool. Physical
examination demonstrates a large perianal mass. Pathologic examination of the rectal mass following resection
demonstrates a condyloma in which transformation to frank carcinoma has occurred. Which of the following
viruses would most likely be associated with these lesions?

A. Epstein-Barr virus (EBV)

B. Hepatitis B virus (HBV)

C. Human herpesvirus type 8 (HHV8)

D. Human papilloma virus (HPV)

E. Human T-cell leukemia virus (HTLV-1)

Explanation:

The correct answer is D. The tumor is anal carcinoma arising in a condyloma. Both condyloma and anal
carcinoma are related to human papilloma virus (HPV), which is also associated with cervical and penile
condylomas and carcinomas.

EBV (choice A) is associated with Burkitt's lymphoma and nasopharyngeal carcinoma.

HBV (choice B) is associated with hepatocellular carcinoma.

HHV8 (choice C) is associated with Kaposi's sarcoma.

HTLV-1 (choice E) is associated with adult T-cell leukemia.

A 23-year-old woman with a history of sickle cell disease presents with fever and severe bone pain localized to
her left tibia. X-ray reveals a lytic lesion and blood cultures reveal infection. A bone culture grows gram-negative
rods. Which of the following best describes the infecting organism?

A. It is a facultative intracellular parasite

B. It is a nonmotile facultative anaerobe

C. It is comma-shaped and sensitive to acidic pH

D. It is motile and does not ferment lactose

E. It is motile and oxidase positive

Explanation:

The correct answer is D. The presence of sickle cell disease in a question stem is usually a significant clue.
This question tests if you know that patients with sickle cell anemia are more susceptible to osteomyelitis
caused by Salmonella. (The patient's fever, bone pain, and x-ray results indicate osteomyelitis). But note that
Staphylococcus aureus (gram-positive coccus) is the most common cause of osteomyelitis in both sicklers and
113
nonsicklers. If it had not been ruled out on bone culture, you should have looked for it in the answer choices.
Notice that you were required to know more than just the organism's name; you needed to know its
distinguishing features. Choice D describes Salmonella (a gram-negative rod) accurately. Salmonella exists in
over 1800 serotypes and is known to contaminate poultry.

A facultative intracellular parasite (choice A) is Legionella, a catalase-positive gram-negative rod. It


contaminates air-conditioning cooling towers and causes Legionnaire's disease (a type of pneumonia).

A nonmotile, facultative anaerobe (choice B) is Shigella, a gram-negative rod that does not produce H2S. All
Shigella contain an endotoxic lipopolysaccharide. The organism causes bacillary dysentery, with abdominal
cramps, fever, and mucoid, bloody diarrhea.

A comma-shaped organism that is sensitive to acidic pH (choice C) is Vibrio cholerae, a gram-negative rod that
causes severe enterotoxin-induced diarrhea, with "rice-water" stools and dehydration. The toxin acts by
stimulating adenylyl cyclase to overproduce cAMP in the brush border of the small intestine.

A motile and oxidase positive organism (choice E) is Pseudomonas, a gram-negative rod with pili that
sometimes produces a polysaccharide slime layer. P. aeruginosa is the prototype and commonly colonizes the
lungs of patients with cystic fibrosis. It is associated with blue-green pus.

A 42-year-old Hispanic man is brought to the emergency room by ambulance after suffering a grand mal seizure
at home. There is no history of recent illness, fever, headache, seizures, or head trauma. He drinks alcohol
occasionally and denies any other drug use. The patient has been a resident of the United States for 15 years,
but occasionally travels to his previous home in Honduras. Neurologic exam shows the patient to be alert and
oriented. No focal abnormalities are noted. A CT scan of the head reveals multiple punctate calcifications, and
two enhancing cystic lesions with surrounding edema. What is the most likely diagnosis?

A. Amebiasis

B. Cytomegalovirus infection

C. Echinococcosis

D. Neurocysticercosis

E. Toxoplasmosis

Explanation:

The correct answer is D. Cysticercosis is a parasitic infection caused by the larval cysts of the tapeworm Taenia
solium. Patients acquire the infection by ingesting the eggs, which reach a larval stage in various tissues. When
the central nervous system (CNS) is involved, the condition is known as neurocysticercosis. It is the most
common parasitic infection of the CNS. Infection with this organism is most frequently encountered in individuals
from Mexico, South Central America, the Philippines, and Southeast Asia. In the CNS, the cysts act as
space-occupying lesions and can cause hydrocephalus and/or seizures. Seizures are the most common initial
presentation of patients with neurocysticercosis and may be focal or generalized. Signs of increased intracranial
pressure such as headache, nausea, vomiting, or visual changes may also be present.

Amebiasis (choice A) is caused by Entamoeba histolytica. Patients typically present with diarrhea (often bloody),
right lower quadrant abdominal pain, and fever. Amebic abscesses in the liver are a complication due to
invasion of the portal venous system by the amoeba.

Cytomegalovirus (choice B) produces neonatal infections and infections in immunocompromised patients such
as AIDS patients. CD4 counts are usually <100 cells/mm3. The most common clinical presentation is
chorioretinitis producing floaters, visual field deficits, and painless loss of vision. CMV also produces
encephalitis and may produce calcifying lesions in the CNS.
114
Echinococcosis (choice C) is a parasitic nematode infection caused by either Echinococcus granulosis or E.
multilocularis. The disease is hydatid cyst disease. The patient ingests the eggs from dogs and becomes an
intermediate host. The cysts are classically in the liver, are calcified, and the patient shows eosinophilia.

Toxoplasmosis (choice E) is a protozoan infection acquired by eating undercooked meat or by exposure to cat
feces. Primary toxoplasmosis is usually asymptomatic. In patients with normal immunity, the organism can cause
a heterophile-negative, mononucleosis-like syndrome. In patients with AIDS, it causes ring-enhanced focal brain
lesions and pneumonia.

Which of the following viruses produce disease or sequelae that is/are more severe if the infection
occurs at a very young age?

A. Epstein-Barr virus

B. Hepatitis B virus

C. Measles virus

D. Poliovirus

E. Varicella zoster virus

Explanation:

The correct answer is B. Infection with Hepatitis B virus (HBV) at birth or a very young age is
associated with chronic HBV infection and the development of hepatocellular carcinoma later in
life. In fact, infants born to Hepatitis B surface antigen (HBsAg)-positive mothers are
commonly infected, and approximately 90% become chronic carriers of the virus. Chronic carriers
suffer from hepatocellular carcinoma at an incidence over 200 times higher than a noncarrier.
The current recommendation for infants born of HBsAg-positive mothers is administration of
hepatitis B immunoglobulin (HBIg) in the delivery room, with the first dose of the hepatitis B
vaccine given at the same time or within 1 week. The second and third dose of the vaccine are
then given at 1 and 6 months. With this protocol, 94% protection is achieved.

The Epstein-Barr virus (EBV; choice A) is the agent of heterophile-positive infectious


mononucleosis. In children, primary EBV infection is often asymptomatic.

The measles virus (choice C) often causes a more severe disease in adults. Over the age of 20,
the incidence of complications, including pneumonia, bacterial superinfection of the
respiratory tract, bronchospasm and hepatitis, is much higher than in children.

Poliovirus (choice D) causes asymptomatic or inapparent infections 95% of the time. Frank
paralysis occurs in approximately 0.1% of all poliovirus infections. However, the probability
of paralysis increases with increasing age.

Varicella zoster virus (choice E) is the agent of chickenpox and shingles. In immunocompetent
children, it is a benign illness with a mortality of less than 2 per 100,000 cases. This risk
is increased over 15-fold in adults. Much of the increase is due to varicella pneumonitis, a
complication that occurs more frequently in adults.

A 40-year-old otherwise healthy gardener presents with several subcutaneous nodules on his right hand,
where he had cut himself on rose thorns. Physical exam reveals several erythematous fluctuant
lesions. Which of the following organisms is most likely responsible for his condition?

A. Aspergillus

B. Malassezia

C. Onchocerca
115
D. Rhizopus

E. Sporothrix

Explanation:

The correct answer is E. Whenever you see a question on a gardener who works with roses, think
"Sporothrix schenckii." This organism is responsible for "rose gardener's disease," known
technically as sporotrichosis. The organism enters through skin breaks in the fingers or hands,
causing a chancre, papule, or subcutaneous nodule with erythema and fluctuance. Ulcerating
lesions appear along lymphatic channels, but the lymph nodes are not commonly infected.
Potassium iodide is the treatment of choice for the subcutaneous manifestations.

Aspergillus (choice A) causes pulmonary aspergillosis&ndash;a systemic mycotic infection. There


is an allergic type that is caused by a hypersensitivity reaction to the organism and an
infectious type that occurs more commonly in the immunocompromised. Hemoptysis is a common
symptom.

Malassezia (choice B), specifically M. furfur, causes tinea versicolor, a superficial mycotic
infection.

Onchocerca (choice C), specifically O. volvulus, causes river blindness. This organism is a
helminth that is transmitted by black flies in Africa, Central America, and South America.

Rhizopus (choice D) causes rhinocerebral infections in diabetics with ketoacidosis.

A 33-year-old man in Arkansas presents to the local clinic with fever, chills, headache, and generalized
myalgias and arthralgias. On physical examination, an ulcer with a black base is apparent on
his right hand, and epitrochlear lymphadenopathy is noted. A careful history reveals that the
man is a hunter and trapper, recently trapping and skinning rabbits. Which of the following
organisms is the most likely cause of this man's illness?

A. Borrelia

B. Brucella

C. Francisella

D. Leptospira

E. Listeria

Explanation:

The correct answer is C. The disease is tularemia, which may be spread by handling rabbits or
rabbit skins, or by bites from ticks that feed on the blood of wild rabbits. The causative
organism is Francisella tularensis, a gram-negative coccobacillus. The disease begins as a
rupturing pustule followed by an ulcer, with involvement of regional lymph nodes. More serious
cases can be complicated by bacteremia, splenomegaly, rash, pneumonia, or endotoxemic shock.

Borrelia(choice A) spp. cause relapsing fever (transmitted by ticks and lice) and Lyme disease
(transmitted by ticks).

Brucella(choice B) causes brucellosis after ingestion of contaminated milk or contact with


infected livestock.

Leptospira (choice D) causes leptospirosis and Weil's disease; the organism is acquired by
ingestion of water contaminated with animal urine.

Listeria (choice E) causes listeriosis after contact with contaminated milk, vegetables, or
with transplacental transmission.
116

A 28-year-old HIV-positive male complains of pain on swallowing. Physical examination is remarkable for
white plaque-like material on his tongue and buccal mucosa, which is scraped and sent to the
laboratory. Based on these findings, and on the laboratory results, the man is diagnosed with
acquired immunodeficiency syndrome (AIDS). With which of the following agents is the man most
likely infected?

A. Candida albicans

B. Cytomegalovirus

C. Herpes simplex I

D. Human herpesvirus 8

E. Human papilloma virus

Explanation:

The correct answer is A.Candida albicans produces oral thrush, an AIDS-defining lesion, which
is common in acute HIV disease, and becomes increasing common as the CD4 + cell count falls.
The lesions are usually painless. Diagnosis is by demonstration of pseudohyphae using a wet
smear with confirmation by culture.

Although cytomegalovirus (choice B) is associated with numerous clinical scenarios in the AIDS
population, including odynophagia (painful swallowing), it would not produce white plaques on
the oral mucosa.

Herpes Simplex I (choice C) produces vesicular lesions occurring in clusters in the oral
cavity. There is an increased risk of herpes infections in the AIDS group, but the lesions do
not resemble those described in the question.

Human herpesvirus 8 (choice D) is the causative agent of Kaposi's sarcoma, a malignancy arising
from endothelial cells that appears as hemorrhagic nodules in different organ systems. It is
the most common cancer in the HIV infected population.

Human papilloma virus (choice E) is associated with a variety of lesions, including warts and
intraepithelial neoplasias of the vulva and cervix. It is associated with anal condyloma, which
can occur in the AIDS population.

A 26-year-old obstetric patient becomes acutely ill during her first trimester with infectious
mononucleosis-like symptoms, but her heterophil antibody test was negative. A careful history
reveals that the family has two cats in the house. The appropriate laboratory tests indicate
the expectant mother is infected with Toxoplasma gondii. Months later, the woman delivers a
full term baby with no obvious signs of infection with the protozoan parasite. The best test to
diagnose acute infection in the neonate would be a parasite-specific ELISA for which isotype of
immunoglobulin?

A. IgA

B. IgE

C. IgG 1

D. IgG 4

E. IgM

Explanation:

The correct answer is E. IgM immunoglobulin directed against Toxoplasma would provide evidence
117
of infection in the newborn baby. IgM is the only antibody that a baby can form with an acute
infection. It is the first antibody that is formed in an infection and it is also the antibody
that is present on the surface of immature and mature B cells.

IgA (choice A) is the antibody that is produced in response to mucosal infections. It cannot
cross the placenta. The baby's immune system is still not well developed at birth and cannot
produce IgA immunoglobulin. In adults or older children, IgA is exists in serum in a monomeric
form but is present in seromucous secretions as a dimer.

IgE (choice B) is the antibody that is produced in response to an allergen. It cannot cross the
placenta, and is not present in the baby at birth or for some time after birth. The baby's
immune system is still not well developed at birth and can initially only form IgM
immunoglobulin.

IgG 1 (choice C) and IgG4 (choice D) immunoglobulins might be present in the baby, but would
not indicate infection in the infant. The presence of these isotypes would indicate that the
mother was infected with the organism and produced antibody that was transported across the
placenta in utero. IgG is the only immunoglobulin that can cross the placenta, providing
protection to the baby during the first few months of life.

Which of the following organisms is a natural transformer?

A. Escherichia coli

B. Neisseria gonorrhoeae

C. Plasmodium vivax

D. Pseudomonas aeruginosa

E. Staphylococcus aureus

Explanation:

The correct answer is B. Transformation is the uptake and integration of naked DNA by a
bacteria from the environment. Transformation can be induced in the laboratory (a technique
used to introduce gene-carrying plasmids into bacteria) or, relatively uncommonly, occurs
naturally. Only a few medically important species undergo natural transformation: Haemophilus
species, Streptococcus species, Neisseria gonorrhoeae, and Helicobacter pylori.

Which of the following structures is found only in Gram-negative microorganisms?

A. Cell envelope

B. Exotoxin

C. Peptidoglycan

D. Periplasmic space

E. Teichoic acids

Explanation:

The correct answer is D. This question requires you to appreciate the key structural difference
between gram-positive and gram-negative microorganisms. Most gram-negatives (other than
exceptional microorganisms, such as Mycoplasma, that lack a cell wall) have a more complex cell
envelope than gram-positive microorganisms. It includes both a cytoplasmic membrane as well as
an outer membrane. Between these two membranes is the periplasmic space, which contains enzymes
such as phosphatase and penicillinase, binding proteins for the transport of various nutrients,
and peptidoglycan, as well as a portion of the lipoprotein that firmly anchors the outer
118
membrane to the peptidoglycan. Gram-positives do not have outer membranes.

Choice A, the cell envelope, is incorrect because both gram-positive and gram-negative
microorganisms have this structure, which is defined as all the layers that enclose the cytosol
of the bacterium. It is the composition of the envelope that differs between gram-positives and
gram-negatives.

Choice B, exotoxin, is not exclusive to gram-negative microorganisms, but is also found in some
gram-positives. By contrast, endotoxin (lipopolysaccharide; LPS) is found exclusively in gram-
negatives.

Choice C, peptidoglycan, is found in the cell walls of both gram-positive and gram-negative
microorganisms. Note that there is a larger amount of peptidoglycan in gram-positive
microorganisms.

Choice E, teichoic acids, are found exclusively in gram-positive organisms.

A 23-year-old, ill-appearing woman comes to the emergency room with a fever. She notes the recent
development of nausea, diarrhea, and a rash. Her last menstrual period began 3 days ago.
Physical examination is remarkable for blood pressure of 90/45 mm Hg and heart rate of 120 beats
per minute. A diffuse erythematous rash with areas of desquamation over the hands and feet is
noted. Infection with which of the following agents is the most likely cause of these signs and
symptoms?

A. Clostridium perfringens

B. HIV-1

C. Shigella dysenteriae

D. Staphylococcus aureus

E. Staphylococcus epidermidis

Explanation:

The correct answer is D. This patient has toxic shock syndrome (TSS), a multisystem syndrome
caused by a toxin (TSST-1) formed by certain strains of S. aureus. TSS usually affects several
organ systems (gastrointestinal, renal, hepatic, hematopoietic, musculoskeletal, pulmonary) and
can result in death. TSS has historically been associated with the use of tampons in young
women, but can also occur in other patient populations. Fever, hypotension, diarrhea, and
diffuse rash with desquamation of the hands and feet are common symptoms. Management of shock,
renal failure, and adult respiratory distress syndrome (ARDS) are a priority if these
conditions are present, in addition to appropriate antibacterial treatment.

C. perfringens(choice A) causes gas gangrene, with necrosis of soft tissues, usually after a
traumatic wound. It is also a cause of food poisoning.

Although HIV-1 (choice B) can cause many diverse findings and should never be immediately ruled
out, the findings in this patient are most specific for TSS.

Shigella dysenteriae(choice C) is a cause of dysentery characterized by fever, abdominal


cramps, and bloody diarrhea.

S. epidermidis(choice E) is part of the normal skin flora, but is notorious for causing
infections of intravenous lines and prosthetic heart valves.

A pregnant southeast Asian immigrant presents for prenatal care. Her past medical history is significant
for a severe illness 3 years ago characterized by fatigue, nausea, anorexia, vomiting, jaundice,
joint pains, and generalized skin lesions that slowly disappeared. She has felt well recently.
Which of the following laboratory tests should be ordered to investigate the patient's past
119
illness?

A. Hepatitis B surface antigen (HBsAg)

B. IgG cytomegalovirus (CMV) antibody levels

C. IgM antibody to HBsAg

D. IgM antibody to hepatitis B core antigen

E. Quantitation of hepatitis A virus (HAV) IgM antibody

Explanation:

The correct answer is A. The clinical signs suggest that this woman had hepatitis B three years
ago and the fact that she is now feeling better also suggests that she recovered from this
infection. Since she is pregnant, it is necessary to find out if she still has the organism in
her liver (chronic hepatitis B infection) by performing a test for HBsAg. This test will be
negative if she has completely recovered from the disease, but it would be positive if she is a
chronic carrier.

The clinical signs of the disease she had 3 years ago do not match those of CMV (choice B), and
CMV does not produce chronic infections.

Measurement of IgM antibody to hepatitis B core antigen (choice D) would be of no value at this
time. This antibody is positive in acute cases of hepatitis B, but would no longer be positive
in this case. Measurement of IgM anti-hepatitis B core antigen is one of the most important
tests in the hepatitis profile because the appearance of the antibody correlates with the
disappearance of HBsAg.

Examination of IgM antibody to HBsAg (choice C) would be of no value at this time. This IgM
antibody would be formed early during the recovery from acute hepatitis B, and would class
switch to IgG later on in the disease.

Quantitation of hepatitis A virus (HAV) IgM antibody (choice E) would be useless, since
hepatitis A virus (HAV) only causes acute infection.

The rationale for giving prophylactic antibiotics at the beginning of a surgical procedure can be
thought of as an attempt to prevent the bacteria from entering which of the following growth
phases?

A. Death phase

B. Lag phase

C. Log phase

D. Phase of decline

E. Stationary phase

Explanation:

The correct answer is C. A single dose of a prophylactic antibiotic given as surgery is about
to begin (sometimes given intravenously to assure the timing) has become more common because
it is associated with a decreased wound infection rate. Such antibiotic usage does not prevent
organisms from entering the tissues, but effectively prevents them from becoming established
as they try to grow and divide (e.g., leave the lag phase (choice B) and enter the log or
exponential phase of colony growth). The stationary phase (choice E) and phase of decline
(choice D) (formerly called death phase, choice A) would occur much later, after a colony had
been established (which is what the surgeons were trying to prevent).
120

A 10-year-old girl presents with sore throat and fever. She denies any cough or rhinorrhea. A throat
culture grows bacitracin-sensitive bacterial colonies. The infecting organism would be protected
from the lytic action of detergents by its

A. keratin-like proteins in the spore coat

B. lipopolysaccharide in the outer membrane

C. peptidoglycan layer

D. periplasmic space

E. Ca2+ chelators

Explanation:

The correct answer is C. This girl has streptococcal pharyngitis. The infecting organism is
group A beta-hemolytic streptococcus (S. pyogenes); its growth is inhibited by the placement of
a bacitracin disk on the throat culture plate. (Beta-hemolysis occurs as the result of the
bacterial hemolysin streptolysin S). This is a gram-positive bacterium and therefore possesses
a very thick peptidoglycan layer that would protect it from lysis by detergents. (Note that
gram-positives also contain teichoic acid.) In contrast, gram-negatives have a thin
peptidoglycan layer.

Keratin-like proteins in the spore coat (choice A) and calcium ion chelators (dipicolinic acid)
(choice E) are found in spores formed by species of Bacillus and Clostridium. These protect the
spores from the elements: dehydration, heat, chemicals, radiation, etc.

Lipopolysaccharide in the outer membrane (choice B) and a periplasmic space (choice D) are
found in gram-negative organisms. The lipopolysaccharide is an endotoxin and the periplasmic
space contains beta-lactamase in some species.

An antibiotic, such as penicillin, which modifies cell wall synthesis, tends to be most effective during
which phase of bacterial growth in a closed system?

A. Lag phase

B. Log phase

C. Phase of decline

D. Stationary phase

Explanation:

The correct answer is B. Bacterial growth in a closed system is characterized by four phases:

(1) In the initial lag phase, no growth occurs as the organisms adapt to the new environment.

(2) In the exponential, or log phase, the organisms grow at the fastest rate and antibiotics
that interfere with cell growth or division are most likely to be effective.

(3) In the stationary phase, when nutrients have been largely exhausted, organisms tend to
stop growing but may remain viable for long periods of time.

(4) In the phase of decline, cell deaths increase due to cell starvation or exposure to
toxins.

A 14-year-old patient is brought in by his parents because of a sore throat. On physical examination, he
is febrile, and has pharyngeal erythema with a tonsillar abscess. A throat culture on sheep
121
blood agar yields colonies of gram-positive cocci that are surrounded by a zone of complete
hemolysis. The organism was also plated on mannitol salt agar; it grew well and caused the
medium to turn yellow. Which of the following microorganisms is the most likely cause of the
patient's illness?

A. Corynebacterium diphtheriae

B. Haemophilus influenzae

C. Staphylococcus aureus

D. Streptococcus salivarius

E. Streptococcus pyogenes

Explanation:

The correct answer is C. The description of the agent is consistent with a staphylococcal
organism (catalase-positive, gram-positive cocci that grows on mannitol salt agar. The organism
is most likely S. aureus, as it was able to ferment mannitol (as evidenced by the color change
in the mannitol salt agar after incubation) and was beta-hemolytic. Other characteristics of
this organism are coagulase production and excretion of DNAse from colonies. Staphylococcal
organisms are also notorious for formation of abscesses; the patient had an abscess in the
tonsillar region.

The diphtheria bacilli (Corynebacterium diphtheriae; choice A) are gram-positive, pleomorphic


rods that are arranged in palisades. They are non-hemolytic and would not grow on mannitol salt
agar. These organisms produce a whitish-gray pseudomembrane on the pharynx or the larynx;
constitutional symptoms from toxemia are the major features of diphtheria.

Epiglottitis is the most common disease of the upper respiratory tract produced by Haemophilus
influenzae(choice B), a gram-negative, encapsulated rod. It is also a common cause of otitis
media in children and may cause bronchitis, bronchiolitis, and pneumonia in adults.

Streptococcus salivarius(choice D) is a gram-positive coccus that is usually alpha-hemolytic.


It is normal flora of the oral cavity and is sometimes implicated in subacute bacterial
endocarditis, but is not associated with tonsillar abscesses.

Streptococcus pyogenes(choice E) is a beta-hemolytic, gram-positive coccus that grows in


chains, as opposed to the random, grape-like clusters of the staphylococci. These organisms are
the most common cause of pharyngitis, which is usually manifested by severe sore throat, fever,
a beefy red pharynx, and a tonsillar exudate.

A 14-year-old boy is brought to the emergency department by his parents because of high fever, headache,
and stiff neck for the past 36 hours. He has become confused over the course of the morning. He
uses no medications, has no allergies, and is not sexually active. On examination, he has a
temperature of 38 C, moderate nuchal rigidity, and marked photophobia. Lumbar puncture is
performed, and the cerebrospinal fluid shows: WBC 3500 with 95% PMNs, elevated protein,
decreased glucose, and gram-positive cocci in pairs and short chains. By which of the following
mechanisms does this organism mediate its attachment to the respiratory mucosa?

A. Production of a C carbohydrate

B. Production of a hemolytic exotoxin

C. Production of a hyaluronic acid capsule

D. Production of an IgA protease

E. Production of an M protein
122
Explanation:

The correct answer is D. This is a case of Streptococcus pneumoniae meningitis. This organism
is able to attach to the respiratory mucosa because it has teichoic acids in its envelope, and
because it produces an IgA protease. The IgA protease physically cleaves the immunoglobulin
molecules, leaving the Fc parts to coat the bacterium. This allows the organisms to bind to the
Fc receptors on various mucosal cells.

Streptococcus pneumoniae does not produce a C carbohydrate (choice A), and thus cannot be
grouped by the Lancefield terminology.

Although the organism does produce the pneumolysin, which is an alpha-hemolytic exotoxin
(choice B), this toxin mediates damage to the respiratory epithelium and inhibition of
leukocytic responses, not attachment to the mucosa.

Streptococcus pneumoniae does not produce an hyaluronic acid capsule (choice C); Streptococcus
pyogenes does. Most capsules serve to inhibit phagocytosis, rather than to mediate adhesion.

Only the group A Streptococci possess an M protein (choice E), which is used to "type" them and
helps inhibit phagocytosis.

In a closed system, spores are formed during which of the following phases of bacterial growth?

A. Decline phase

B. Exponential phase

C. Lag phase

D. Log phase

E. Stationary phase

Explanation:

The correct answer is E. Spore formation usually occurs during the stationary phase, when cell
growth ceases because of a developing lack of nutrients or accumulation of toxins.

During the phase of decline (choice A), the lack of nutrients and the accumulation of toxin
become so severe that any viable organisms usually die before they can form spores.

The exponential phase (choices B) and log phase (choice D) are descriptors for the steady state
of active growth occurring after the lag phase and before the stationary phase. Many
antibiotics are most effective in this period.

The lag phase (choice C) is the initial period of adaptation, prior to growth, which occurs
when organisms are introduced to a new environment.

A 33-year-old HIV-positive male complains of headache and blurred vision. Physical exam reveals
papilledema and ataxia. Head CT is normal but CSF obtained by lumbar puncture reveals
encapsulated organisms observable with India ink. What is the treatment of choice for this
infection?

A. Amphotericin B

B. Isoniazid

C. Ketoconazole

D. Metronidazole
123
E. Nystatin

Explanation:

The correct answer is A. Amphotericin B is the most appropriate drug listed for the treatment
of cryptococcal meningitis. It is a polyene antibiotic that binds to ergosterol in the fungal
cell membrane, creating an artificial pore. Flucytosine is often prescribed as an adjunct
medication. Fluconazole is used long-term to prevent recurrence in AIDS patients.

Isoniazid (choice B) inhibits the biosynthesis of mycolic acids in the mycobacterial cell wall.
It is the primary drug used against tuberculosis. It is used alone for TB prophylaxis and is
used in combination with other antituberculars to treat patients with active disease.

Ketoconazole (choice C) is an orally administered imidazole antifungal medication. It inhibits


14-alpha-demethylase to block the synthesis of fungal cell membrane ergosterol. Note the
difference in mechanism between the polyenes, which alter ergosterol structure, and the
imidazoles, which block ergosterol synthesis. Ketoconazole is often used to treat
coccidioidomycosis (prevalent in California), histoplasmosis (prevalent in the Midwest),
blastomycosis (prevalent in the Eastern US), paracoccidioidomycosis (prevalent in Latin
America), and mucocutaneous candidiasis.

Metronidazole (choice D) is an antiprotozoal drug useful in treating a variety of parasitic


infections. It is the drug of choice for trichomoniasis and giardiasis and provides general
anaerobic coverage. This makes it useful for treating postsurgical abdominal and pelvic
Bacteroides fragilis infections or flare-ups of intestinal diverticulitis.

Nystatin (choice E) is an antifungal polyene that's usually used topically but can be taken
orally for oral and esophageal candidiasis. Candidal infections of the skin, mucous membranes,
and vagina usually respond well to this drug. It may also be used to prevent intestinal fungal
overgrowth in patients on chemotherapy.

A 37-year-old man is admitted to the hospital with shortness of breath, cyanosis, and fever. Chest x-ray
films reveal consolidation of the right lower lobe with relative sparing of the remaining lobes.
A clinical diagnosis of lobar pneumonia is made and supported by the results of sputum cultures.
Which of the following is the genus of the bacterium most likely to be isolated from this
patient's sputum?

A. Haemophilus

B. Klebsiella

C. Streptococcus, alpha-hemolytic

D. Streptococcus, beta-hemolytic

E. Streptococcus, gamma-hemolytic

Explanation:

The correct answer is C. The clinical and radiologic characteristics of this patient's
condition are consistent with lobar pneumonia, a respiratory infection that, in its classic
presentation, involves a single pulmonary lobe. In 90% to 95% of cases, the etiologic agent is
Streptococcus pneumoniae (AKA pneumococcus), an alpha-hemolytic streptococcus present in the
throat of 40% to 70% of healthy individuals. A small minority of cases of lobar pneumonia are
due to Klebsiella pneumoniae, staphylococci, streptococci other than pneumococcus, Haemophilus
influenzae, Pseudomonas, and Proteus. Penicillin is the drug of choice for pneumococcal
pneumonia, but sputum cultures are necessary to identify the infectious agent and determine its
antibiotic sensitivity. S. pneumoniae is a major cause of purulent meningitis in the elderly.
Other alpha-hemolytic streptococci (viridans streptococci) cause subacute endocarditis in
patients with previously altered cardiac valves. Streptococcus mutans and other oral
streptococci have been associated with dental caries.
124

Bacteria belonging to the genera Haemophilus(choice A) and Klebsiella (choice B) cause


respiratory infections; however, in immunocompetent individuals, these bacterial infections
usually result in bronchopneumonia, which leads to multilobar, and often bilateral, pulmonary
involvement.

Beta-hemolytic streptococci (choice D) cause countless infections in humans. Recall that


classification of &beta;-hemolytic streptococci is based on their surface antigens known as
Lancefield antigens. Human diseases caused by this group of bacteria include:

- Streptococcus pyogenes (group A): pharyngitis, scarlet fever, erysipelas, impetigo, rheumatic
fever, and glomerulonephritis

- Streptococcus agalactiae (group B): neonatal sepsis and urinary infections

- Enterococcus faecalis (group D): endocarditis and urinary infections

Gamma-hemolytic streptococci (choice E) are streptococci that do not produce hemolysins


(nonhemolytic streptococci) and are not a significant cause of human disease.

A 67-year-old man with moderate renal dysfunction presents with influenza-like symptoms. The patient
also has a red macular rash that first appeared on the ankles, then spread centrally. He reports
nausea, vomiting, and profound restlessness. He states that he was recently hiking with some
friends in the mountains, about a week ago. Rickettsia rickettsiae is demonstrated by
immunohistochemistry on skin biopsy. Which of the following agents would be most appropriate to
treat this patient's infection?

A. Demeclocycline

B. Doxycycline

C. Methacycline

D. Oxytetracycline

E. Tetracycline

Explanation:

The correct answer is B. This patient has Rocky Mountain spotted fever. This diagnosis can be
confirmed with the immunohistologic demonstration of R. rickettsiae in the skin biopsy. The
tetracyclines are a class of antibiotics commonly used in the treatment of rickettsial
infections, acne, and various sexually transmitted diseases, as well as in the treatment of
infections caused by susceptible organisms in penicillin-allergic patients. The key to this
question is knowing which tetracycline antibiotic is safest to administer to a patient with
renal dysfunction. Doxycycline is secreted in an inactive form into the intestinal lumen and
eliminated in the feces; therefore, its half-life is largely independent of renal or hepatic
function. The other tetracyclines listed are concentrated by the liver in the bile and excreted
in the urine and feces unchanged. Therefore, dosage adjustments need to be made in patients
with renal impairment because of accumulation of the drug in the body. In other words,
demeclocycline (choice A), methacycline (choice C), oxytetracycline (choice D), and
tetracycline (choice E) will accumulate in patients with renal impairment.

A 33-year-old HIV-positive male complains of headache and blurred vision. Physical exam reveals
papilledema and ataxia. Head CT is normal but CSF obtained by lumbar puncture reveals
encapsulated organisms observable with India ink. Which of the following is true concerning this
organism?

A. It can also be identified with methenamine silver stain

B. It consists of branching septate hyphae


125

C. It exists as a mycelial form at room temperature and as yeast at 37° C

D. It is an encapsulated nondimorphic yeast found worldwide

E. It is a nonencapsulated dimorphic yeast that reproduces by budding

Explanation:

The correct answer is D. This patient has cryptococcal meningitis, as evidenced by the
"encapsulated organisms observable with India ink" in the CSF (a classic clue). Cryptococcus is
a nondimorphic yeast, meaning that it exists only in the yeast form. It is encapsulated (that's
why the India ink stain works so well) and it reproduces by budding. It is found worldwide in
bird droppings (think pigeons). It can also cause transient pulmonary illness in otherwise
healthy individuals.

The methenamine silver stain (choice A) is used primarily to demonstrate Pneumocystis carinii
in tissues.

Branching septate hyphae (choice B) are characteristic of Aspergillus fumigatus, among other
fungi.

Mycoses that exist in mycelial and yeast forms (dimorphism; diphasic; choice C) are Histoplasma
capsulatum, Coccidioides immitis, Blastomyces dermatidis, and Sporothrix schenckii.

C. neoformans is not dimorphic and it has a capsule (choice E).

A 5-year-old child who has not had routine pediatric care develops a febrile disease with cough and a
blotchy rash, and is brought to the emergency room. On physical examination, there is cervical
and axillary lymphadenopathy. Also noted is an erythematous, maculopapular rash behind the ears
and along the hairline, involving the neck and, to a lesser extent, the trunk. Examination of
this patient's oropharynx would likely reveal which of the following lesions?

A. Adherent thin, whitish patch on gingiva

B. Cold sores on the lips

C. Curdy white material overlying an erythematous base on the oral mucosa

D. Large shallow ulcers on the oral mucosa

E. Multiple small white spots on the buccal mucosa

Explanation:

The correct answer is E. The disease described is measles (rubeola), which has the typical
presentation described in the question stem. Measles is caused by a Morbillivirus, an RNA virus
belonging to the Paramyxovirus family. Koplik's spots, which are pathognomonic for measles, are
small, bluish-white spots on the buccal mucosa in the early stages of measles. These lesions
appear just before the onset of the characteristic rash (which can also involve the
extremities) and fade as the rash develops.

Leukoplakia is a premalignant condition characterized by adherent whitish patches on the


gingiva (choice A) and other sites in the oral cavity.

Cold sores of the lips (choice B) are due to infection with herpes viruses.

Candida infection (thrush) produces curdy white material loosely attached to an erythematous
base (choice C).

Aphthous ulcers are large shallow ulcers of the oral mucosa (choice D), commonly known as
126
canker sores.

Which of the following is associated with Babesiosis rather than with Ehrlichiosis or Lyme disease?

A. A characteristic rash

B. Arthritis of the knee

C. Hemolytic anemia

D. Macrophages containing intracellular organisms

E. Transmission by ixodid ticks

Explanation:

The correct answer is C. Babesiosis is a syndrome characterized by malaise, fatigue, chills,


fever, myalgia, and arthralgia lasting for weeks. The protozoans Babesia microti and Babesia
divergens cause most human babesiosis. The organisms parasitize red blood cells, and have been
transmitted by blood transfusions. Hemolytic anemia and hepatosplenomegaly are features of
infection with these organisms; asplenic persons may have a life-threatening infection.

In Lyme disease, a characteristic rash (choice A), known as erythema chronicum migrans, forms
at the tick-bite site. In ehrlichiosis and babesiosis, low platelet counts may produce
nonspecific purpuric lesions, but there is not characteristic rash.

Arthritis of the knee (choice B) is a classic sign in untreated Lyme disease caused by Borrelia
burgdorferi.

Macrophages containing intracellular organisms (choice D) is the classic finding in multiple


tissues infected with Ehrlichia chaffeensis.

Ixodid ticks or hard-bodied ticks (choice E) are vectors for Babesiosis, Lyme disease, and
human granulocytic ehrlichiosis.

A United Nations representative from a poor African country has an influenza-like illness that resolves
in a few days. Less than a week later, however, he develops muscle pain, spasms, and sensory
disturbances. Two days after this, flaccid paralysis occurs. Which of the following is most
likely to be immediately life-threatening in this patient?

A. Acute renal failure

B. Bowel paralysis

C. Fulminant liver failure

D. Gastrointestinal bleeding

E. Respiratory paralysis

Explanation:

The correct answer is E. The disease is poliomyelitis. Most infections with poliovirus cause
only the influenza-like symptoms, but a small percentage progress to paralytic poliomyelitis.
The most common causes of death are aspiration and airway obstruction as a result of bulbar
paralysis and paralysis of respiratory muscles. Arrhythmias can also be life-threatening.

Acute renal failure (choice A) is usually not seen in poliomyelitis, although the bladder may
become paralyzed.

Bowel paralysis (choice B) can be seen, but is not usually life-threatening.


127

Fulminant liver failure (choice C) is not a feature of poliomyelitis.

Gastrointestinal bleeding (choice D) can be seen in poliomyelitis, but is not usually life-
threatening.

Evaluation of an adult third world immigrant to this country demonstrates chronic headaches accompanied
by chronic mild nuchal rigidity. Cerebrospinal fluid sampling demonstrates a chronic
inflammatory infiltrate with lymphocytes, plasma cells, macrophages, and fibroblasts. Which of
the following is the most likely etiologic agent?

A. Herpes virus

B. Mumps virus

C. Mycobacterium tuberculosis

D. Neisseria menigitidis

E. Streptococcus pneumoniae

Explanation:

The correct answer is C. Nuchal rigidity suggests meningitis. It is convenient to classify


meningitis based on the cerebrospinal fluid (CSF) findings: 1) acute pyogenic meningitis if
neutrophils are markedly increased; 2) acute lymphocytic meningitis if lymphocytes (alone) are
markedly increased, and 3) chronic meningitis if lymphocytes, plasma cells, macrophages, and
fibroblasts are increased. This patient has chronic meningitis. The classical cause of chronic
meningitis is tuberculosis, whose etiologic agent is Mycobacterium tuberculosis. Other causes
include other indolent meningeal infections such as syphilis, brucellosis, and chronic fungal
infections. The granulomas that are characteristic findings in other tissues may or may not be
present in the meningeal tissue, and are usually not recognizable in CSF. Tubercular meningitis
is now uncommon in this country. In immigrants from third world countries, a history of
pulmonary tuberculosis may be distant, undiagnosed, or deliberately concealed.

Herpes virus (choice A) and mumps virus (choice B) are causes of acute lymphocytic meningitis.

Neisseria menigitidis(choice D) and Streptococcus pneumoniae(choice E) are causes of acute


pyogenic meningitis.

A 14-year-old girl with cystic fibrosis is admitted to the hospital with fever and shortness of breath,
and is diagnosed with pneumonia. During a respiratory therapy session, she coughs up mucus that
is distinctly greenish in color. Which of the following organisms should be suspected?

A. Klebsiella pneumoniae

B. Mycoplasma pneumoniae

C. Pneumocystic carinii

D. Pseudomonas aeruginosa

E. Streptococcus pneumoniae

Explanation:

The correct answer is D. The clues suggesting infection with Pseudomonas aeruginosa are the
green-tinged sputum (due to the formation of blue and green pigments by P. aeruginosa), and the
association with cystic fibrosis. Unfortunately, P. aeruginosa is very difficult to treat with
most antibiotics, and presents a very difficult therapeutic challenge in the management of
cystic fibrosis patients.
128

Klebsiella pneumoniae(choice A) is most often seen in alcoholics, the elderly, and diabetics.

Mycoplasma pneumoniae(choice B) is a common cause of community-acquired atypical pneumonia, and


is characterized by a dry, non-productive cough.

Pneumocystic carinii(choice C) usually causes pneumonia in immunocompromised patients, e.g.,


AIDS patients.

Streptococcus pneumoniae(choice E) is an important cause of typical community-acquired lobar


pneumonia.

A truck driver was involved in a serious accident and received second- and third-degree burns over his
body. He was placed in the burn unit and, on his twelfth day of his admission, developed a wound
infection with a bluish-green exudate. Treatment with chloramphenicol and tetracycline was
unsuccessful. A gram-negative, motile organism was isolated that was oxidase-positive, did not
ferment lactose, sucrose, or glucose, but grew on MacConkey's agar and produced a fruity aroma
on that medium. Which of the following organisms was most likely isolated?

A. Candida albicans

B. Clostridium perfringens

C. Escherichia coli

D. Klebsiella pneumoniae

E. Proteus mirabilis

F. Proteus vulgaris

G. Pseudomonas aeruginosa

H. Serratia marcescens

Explanation:

The correct answer is G.Pseudomonas aeruginosa is a very common opportunist in burn patients,
in whom it classically causes secondary wound infections and septicemia. It may also cause
cystitis in patients with urinary catheters and pneumonia in patients with cystic fibrosis. The
organism is found in water and usually gains access to the body via this source, as a
contaminant in the water used in respirators or in water baths, etc. used to cleanse wounds.
This organism is a non-fermenter, that is, it does not metabolize sugars by classic pathways.
It produces a blue-green, water-soluble pigment (pyocyanin), and has a fruity odor when growing
on laboratory media. It has a propensity for developing antibiotic resistance; current therapy
employs the synergistic combination of an aminoglycoside, such as amikacin, with a cell wall
synthesis inhibitor (carbenicillin, ticarcillin, or piperacillin).

Candida albicans(choice A) is a normal flora yeast that will appear as very large, gram-
positive, spherical-to-ovoid organisms with budding daughter cells in Gram-stained
preparations. Candidiasis is an opportunistic infection in individuals with a compromised
immune system. The fungus usually causes mucocutaneous lesions, but in severely compromised
individuals like AIDS patients, systemic disease may occur. Oral candidiasis appears as creamy,
white patches of exudate that can be scraped off an inflamed tongue or buccal mucosa.

Clostridium perfringens(choice B) is a gram-positive, spore-forming, anaerobic rod. It is a


common cause of gas gangrene when it is introduced into a wound. The organism produces a
variety of toxins and enzymes that enable it to destroy muscle tissue and spread through the
soft tissues of the body.

Escherichia coli(choice C) is a lactose-fermenting, gram-negative rod commonly seen as normal


129
flora of the intestine of man. It is the most common cause of urinary bladder infections,
pyelonephritis, and sepsis in patients with indwelling urinary catheters. It is also the major
cause of traveler's diarrhea with watery stools and is a very important pathogen in neonates,
who become infected during passage through the birth canal.

Klebsiella pneumoniae(choice D) is a gram-negative, highly encapsulated rod that is a


significant pulmonary pathogen in individuals with a compromised respiratory apparatus. It is a
common cause of aspiration pneumonia and pulmonary abscesses in alcoholics and patients with
chronic obstructive pulmonary disease. The organism is readily grown on standard laboratory
media such as blood agar or MacConkey's enteric agar.

Proteus mirabilis(choice E) and P. vulgaris(choice F) are highly motile, gram-negative rods


that ferment glucose, but not lactose. Other distinguishing features include "swarming" growth
on solid media (due to the high degree of motility and production of urease and hydrogen
sulfide). These organisms are not commonly associated with wound infections but usually cause
urinary tract infections. The ability to break down urea is thought to contribute to the
development of struvite kidney stones due to the elevation of urine pH by production of
ammonia.

Serratia marcescens(choice H) is a gram-negative organism that is found in the environment in


water, soil, and, occasionally, as normal flora of humans. It is an opportunistic pathogen that
causes respiratory disease in hospitalized patients. Many strains produce a pigment, but the
colonies are usually red, pink, or orange. Interestingly, the pathogenic varieties are most
often non-pigmented.

A 3-month-old infant presents with a 3-day history of fever, cough, and poor feeding. On examination,
the baby appears ill and has a temperature of 102 F and a respiratory rate of 32. A chest x-ray
film shows bilateral patchy infiltrates in the lungs. Which of the following is the most likely
etiologic agent?

A. Coronavirus

B. Influenza type A

C. Parainfluenza type 1

D. Respiratory syncytial virus

E. Rhinovirus

Explanation:

The correct answer is D. Respiratory syncytial virus is the most common cause of bronchiolitis
and pneumonia in children younger than 1 year. Outbreaks occur seasonally in winter and early
spring. Infection does not result in lasting immunity, and reinfection can occur.

Coronavirus (choice A) causes the common cold (nasal obstruction and discharge, sneezing, no
fever or mild fever, occasional sore throat, and/or cough) and acute pharyngitis (sore throat,
with or without cervical adenopathy, ulceration, and conjunctivitis).

Influenza type A (choice B) is the leading cause of influenza. Influenza is a systemic illness
characterized by the sudden onset of fever, headache, myalgias, malaise, and prostration,
followed by cough, nasal obstruction, and sore throat. The lower respiratory tract may also be
involved.

Parainfluenza viruses (choice C) are the leading cause of croup, or acute


laryngotracheobronchitis, in children. This infection involves both the upper and lower
respiratory tracts. Inflammation in the subglottal area leads to hoarseness, dyspnea, a barking
cough, and inspiratory stridor.

Rhinovirus (choice E) is the most common cause of the common cold.


130

A 3-year-old girl with a history of hydrocephalus is brought to the neurologist by her parents with a
severe headache and fever. The girl underwent a revision of a ventricular-peritoneal shunt 1
month ago, and the neurologist suspects that an infection has occurred. Which of the following
organisms would most likely be isolated from the shunt tubing?

A. Bacteroides fragilis

B. Corynebacterium diphtheriae

C. Escherichia coli

D. Staphylococcus epidermidis

E. Streptococcus pneumoniae

Explanation:

The correct answer is D. Staphylococcal meningitis is fairly rare, occurring mostly in patients
with indwelling ventricular-peritoneal shunts. Staphylococcus epidermidis, which normally
colonizes the skin, is the most common organism causing this disorder; Staphylococcus aureus
meningitis occasionally occurs.

Bacteroides fragilis(choice A) is a common cause of anaerobic infections, including sepsis and


peritonitis, but it does not commonly cause meningitis.

Diphtheroids (including Corynebacterium diphtheriae, choice B) are sometimes isolated from


indwelling shunts, but in many cases they are simply contaminants, since some diphtheroids are
normal skin flora.

Escherichia coli(choice C) is isolated from 30% to 50% of neonates with bacterial meningitis,
but is not typically associated with shunt infections.

Streptococcus pneumoniae(choice E) is the most common cause of bacterial meningitis in people


older than 30 years; it is much less common in children.

A 1-year-old child develops voluminous watery diarrhea and vomiting. She is brought to the pediatrician
by her parents and evaluated, then sent home with instructions for the parents to give the
child an electrolyte replacement solution. Which of the following viruses is the most likely
cause of the child's diarrhea?

A. Coronavirus

B. Lymphocytic choriomeningitis virus

C. Norwalk agent

D. Orbivirus

E. Rotavirus

Explanation:

The correct answer is E. Rotavirus is the major cause of diarrhea in infants and children under
the age of 2. The replicates in the intestinal mucosa, producing a profuse, watery, non-bloody
diarrhea, often coupled with nausea and vomiting. Transmission is by the fecal-oral route.

Coronaviruses (choice A) usually cause cold-like illnesses.

Lymphocytic choriomeningitis virus (choice B) can cause headache, malaise, myalgia,


conjunctivitis, and, occasionally, meningitis.
131

Norwalk agent (choice C) can also cause diarrhea, but usually affects patients older than 2
years.

Orbivirus (choice D) is the cause of Colorado tick fever, which is the only tick-borne viral
disease in the United States.

A 3-year-old child with cystic fibrosis presents with weight loss, irritability, and a chronic
productive cough. On physical exam, he is febrile and lung exam reveals intercostal retractions,
wheezing, rhonchi, and rales. Chest x-ray demonstrates patchy infiltrates and atelectasis and
Gram's stain of the sputum reveals slightly curved, motile gram-negative rods that grow
aerobically. The microorganism responsible for this child's pneumonia is also the most common
cause of which of the following diseases?

A. Croup

B. Epiglottitis

C. Meningitis

D. Otitis externa

E. Otitis media

Explanation:

The correct answer is D. Anytime you see pneumonia in a cystic fibrosis patient you should
suspect Pseudomonas aeruginosa. The Gram's stain revealing aerobic, gram-negative rods confirms
your suspicion in this case. Now the question is: which of the diseases listed is also caused
by Pseudomonas? The answer is otitis externa. P. aeruginosa is often found in the external ear,
especially if the conditions are moist ("swimmer's ear") and there is any sort of inflammation.
External otitis is usually a benign process with the only symptoms being an itchy, painful ear.
If, however, the organism penetrates the epithelium and invades the soft tissue, cartilage, and
cortical bone, the process becomes malignant otitis externa, which can progress to
osteomyelitis leading to cranial nerve palsies. This condition is most common in diabetics.

Croup (choice A), also called laryngotracheobronchitis, is a respiratory disease of children


that presents with a characteristic "barking" cough. Croup is caused by parainfluenza virus.

Epiglottitis (choice B) is a potentially fatal infection in children, caused by H. influenzae,


which presents with drooling, difficulty breathing, and stridor. The incidence of this disease
has dropped dramatically with the introduction of the H. influenzae type b (Hib) vaccine.

Meningitis (choice C) is caused by numerous different bacteria, depending on the age of the
patient. The most common causes include S. pneumoniae (elderly), H. influenzae (unvaccinated
children), Group B Strep and E. Coli (neonates) and N. meningitidis (1 month - adult). While P.
aeruginosa can cause meningitis, it is not a common cause.

The most common causes of otitis media (choice E) include S. pneumoniae and H. influenzae. Even
in cases of external ear infections with P. aeruginosa, the middle ear is typically spared.

A pastry chef cut his finger while slicing a cake. After a week, the site of the injury is warm, red,
and swollen, and begins draining pus. While preparing some cream pies, he contaminates the
custard with drainage from the lesion. The pies were eaten several days later by patrons of the
restaurant. Within 4 hours they developed diarrhea and vomiting with no fever. Which of the
following organisms would be most likely to cause these symptoms?

A. Bacillus cereus

B. Clostridium perfringens
132
C. Escherichia coli

D. Shigella sonnei

E. Staphylococcus aureus

Explanation:

The correct answer is E. The chef had a staphylococcal abscess on his finger. S. aureus,
produces enterotoxin A, which was likely present in the cream pies. When ingested, the toxin
causes severe nausea and vomiting within a few hours (the average incubation time is 3-6
hours). There is little diarrhea associated with this type of food poisoning outbreak.

Bacillus cereus(choice A) is a gram-positive spore-forming rod that is associated with food


poisoning outbreaks following the ingestion of fried rice. The time of onset and symptoms would
mimic staphylococcal disease; the major differentiating feature is the food involved. The
organism survives the boiling of the rice because it is a spore-former. It germinates as the
rice cools, grows, and elaborates an enterotoxin that is responsible for the nausea and
vomiting characteristic of the disease.

Clostridium perfringens(choice B) is a gram-positive spore-forming anaerobe that can cause a


longer incubation (18-24 hour) food poisoning, typically with marked diarrhea. Once again, the
spores allow the organism to survive the heating process used in the preparation of the food.
Both Clostridium perfringens and Clostridium botulinum are associated with home-canned
vegetable and sausages.

The symptoms of Escherichia coli(choice C) food poisoning are usually watery diarrhea
(traveler's diarrhea) with minimal nausea and vomiting, or a bloody diarrhea caused by
enteroinvasive strains of the agent. Also, E. coli would be an unlikely cause of the primary
infection in the chef.

Shigella sonnei (choice D) causes enterocolitis characterized by fever, cramps, and diarrhea
after an incubation period of one to four days. Transmission is fecal-oral, associated with
poor hygiene. A wide range of foods has been implicated.

After passing his physical exam, a 19-year-old army recruit gives urine and blood samples for further
testing. Serum analysis yields elevated ALT, HBsAg, Anti-HBc, HBeAg, and bilirubin. All other
values are normal. Which of the following is the hepatitis B status of this recruit?

A. Asymptomatic carrier

B. Chronic active carrier

C. Fulminant hepatitis B

D. Recovered from acute self-limited HBV

E. Vaccinated against HBV

Explanation:

The correct answer is B. The presence of elevated ALT, HBsAg, anti-HBc, HBeAg, and bilirubin
all point to active hepatitis B.

An asymptomatic carrier (choice A) does not have elevated ALT and bilirubin.

The absence of findings on physical examination rules out fulminant hepatitis B (choice C).

Recovery from acute self-limited HBV (choice D) is associated with the presence of anti-HBs and
the decrease in HBsAg and HBeAg.
133
Someone who is vaccinated with HBV (choice E) has anti- HBs only in their serum.

A 4-year-old child living in a slum is bitten by a rat while sleeping. Two days later, the child
develops a rash characterized by discrete erythematous 1-4 mm macules on the extremities and
face, most obvious on the palms and soles. Which of the following organisms is the most likely
cause of this child's disease?

A. Borrelia burgdorferi

B. Pseudomonas mallei

C. Pseudomonas pseudomallei

D. Spirillium minus

E. Streptobacillus moniliformis

Explanation:

The correct answer is E. The child has "rat-bite fever." This occurs in two forms with somewhat
similar clinical manifestations. The form this child has is the Haverhill fever form, caused by
Streptobacillus moniliformis and characterized by a short (often 1-3 days) incubation period.
The Haverhill form is more common in the United States than the Sodoku form, which is caused by
Spirillium minus, has a 1-4 week incubation period, and is most prevalent in Japan. One of the
problems with diagnosing these diseases is that the victims are usually young children, and the
bite site may be inapparent by the time the disease becomes severe enough for the child to be
taken to a doctor.

Borrelia burgdorferi(choice A) causes Lyme disease, which is characterized by an expanding


erythematous rash, arthralgias, and eventual nervous system involvement.

Pseudomonas mallei(choice B) causes glanders, which generally affects horses or humans in close
contact with equines.

Pseudomonas pseudomallei(choice C) causes melioidosis, a rare pulmonary disease found mostly in


Southeast Asia.

Spirillium minus(choice D) causes the Sodoku form of rat-bite fever.

A 35-year-old man develops hemiparesis, ataxia, homonymous hemianopia, and cognitive deterioration. An
MRI of the brain demonstrates widespread areas of abnormal T2 signal in the white matter. An
electroencephalogram is remarkable for diffuse slowing over both cerebral hemispheres. Brain
biopsy reveals demyelination with abnormal giant oligodendrocytes, some of which contain
eosinophilic inclusions. This patient's condition is most closely related to which of the
following diseases?

A. AIDS

B. Chickenpox

C. Measles

D. Syphilis

E. Tuberculosis

Explanation:

The correct answer is A. The condition is progressive multifocal leukoencephalopathy, which is


a rapidly progressive demyelinating disorder in which the JC virus (a papovavirus) infects
oligodendroglial cells in the brain. The eosinophilic inclusions represent accumulations of JC
134
virus. PML occurs in about 1% of AIDS patients, and is the AIDS-defining illness in half of the
patients who develop the condition. There is no effective treatment for this disorder.

Shingles and post-infectious encephalitis can follow chickenpox (choice B), but varicella is
not associated specifically with demyelination.

Measles (choice C) can cause an encephalitis, and in some cases, subacute sclerosing
panencephalitis (SSPE) may follow previous measles infection. These disorders would affect not
only white matter, but gray matter as well. Also, SSPE usually occurs before the age of 18.

Syphilis (choice D) can cause meningitis, encephalitis, and spinal cord damage, but the disease
process would not be limited to the white matter.

Tuberculosis (choice E) can cause a granulomatous meningitis, typically affecting the base of
the brain.

An otherwise healthy patient who has just received a prosthetic aortic valve develops postoperative
fever. Blood cultures are done and she is placed on broad-spectrum antibiotics. Two days later
she is still febrile and clinically deteriorating. Which of the following organisms is the most
probable etiologic agent?

A. Actinomyces israelii

B. Candida albicans

C. Histoplasma capsulatum

D. Nocardia asteroides

E. Trichophyton rubrum

Explanation:

The correct answer is B. The patient likely has a candidal infection of the prosthetic aortic
valve. That is why she did not respond favorably to antibacterial therapy, which is known to
promote fungal infection. Note that Candida tends to colonize foreign bodies such as IV and
Foley catheters, prosthetic valves, and ventricular shunts.

Actinomyces israelii(choice A) is known to cause cervicofacial infections in patients having


undergone dental work.

Histoplasma capsulatum(choice C) causes histoplasmosis, a pulmonary infection common in the


midwestern river valleys. Multi-organ involvement is usually seen only in the
immunocompromised. Transmission of the organism occurs through the inhalation of airborne
microconidia (infectious) spores. The organism is found in bird and bat droppings and in the
soil.

Nocardia asteroides(choice D) is an actinomycete that causes a chronic lobar pneumonia that may
metastasize to the brain. It is more common in the immunocompromised. It is found in soil and
aquatic environments.

Trichophyton rubrum(choice E) is one of the organisms that commonly produces a variety of


cutaneous mycoses, including tinea corporis (ringworm), tinea cruris (jock itch), and tinea
pedis (athlete's foot).

An 18-year-old college student presents to the student health center complaining of a sore throat and
fever. He describes feeling tired for the past few days and reports a loss of appetite. On
examination, he has pharyngitis with cervical lymphadenopathy. Blood tests reveal lymphocytosis
and the presence of heterophil antibodies. Which of the following best describes the virus
responsible for his illness?
135
A. Double-stranded, enveloped DNA virus

B. Double-stranded, nonenveloped DNA virus

C. Single-stranded, enveloped RNA virus

D. Single-stranded, nonenveloped DNA virus

E. Single-stranded, nonenveloped RNA virus

Explanation:

The correct answer is A. This is one of those USMLE-style items where figuring out the
diagnosis is the easy part and remembering the basic science details is much tougher. In this
case, the patient has all the hallmarks of mononucleosis (the heterophil antibodies should have
confirmed your suspicion from the history and physical). Mononucleosis is caused by the
Epstein-Barr virus, which, in turn, belongs to the herpesvirus family. The herpesviruses are
enveloped viruses with double-stranded DNA. Remember that in addition to Epstein-Barr virus,
the herpesvirus family also includes herpes simplex (1 and 2), varicella-zoster (chickenpox,
shingles), and cytomegalovirus (infection in immunocompromised). Cytomegalovirus also causes
infectious mononucleosis, but in these patients the heterophil test is negative.

There are two families of viruses that are nonenveloped with double-stranded DNA (choice B):
papovaviruses and adenoviruses.

There are many families of viruses that are enveloped with single-stranded RNA (choice C):
arenaviruses, bunyaviruses, coronaviruses, filoviruses, flaviviruses, paramyxoviruses,
orthomyxoviruses, retroviruses, rhabdoviruses, and togaviruses.

Parvoviruses are the only family of DNA virus with single-stranded DNA. They do not have an
envelope (choice D).

There are two families of RNA virus that are single-stranded without an envelope (choice E):
caliciviruses and picornaviruses.

A 4-year-old African child develops an infection with Chlamydia trachomatis. How does infection with
this organism cause blindness?

A. Cataract formation

B. Hemorrhage into the anterior chamber

C. Hemorrhage into the posterior chamber

D. Retinal detachment

E. Scarring of the cornea

Explanation:

The correct answer is E.Chlamydia trachomatis (serotypes A, B, and C) causes a conjunctival and
corneal infection that is spread in developing countries by eye-seeking flies. The lesions
begin with formation of lymphoid follicles in the conjunctiva. With disease progression, there
is tissue necrosis, granulation tissue deposition, and scar formation, leading to lacrimal duct
obstruction and distortion of the eyelids. With the loss of an adequate tear system, the cornea
becomes vulnerable to dehydration and opacification. Also, the vigorous inflammatory response
can directly involve the cornea, with resulting opacity. In developed countries, chlamydial eye
infections are often transmitted venereally rather than by flies, and may cause conjunctivitis
in the newborn and in sexually active young adults.

A mother brings her 3-year-old boy into the emergency room because he has developed a harsh, "barking"
136
cough with hoarseness. The virus responsible for this child's illness belongs to which of the
following families?

A. Papovavirus

B. Paramyxovirus

C. Parvovirus

D. Picornavirus

E. Poxvirus

Explanation:

The correct answer is B. This question is difficult for two reasons&mdash;it asks for the viral
family instead of the virus itself, and it includes distracters that all look alike. You
probably realized that the child in question suffers from croup (laryngotracheobronchitis); the
classic clue here is the "barking cough." You might have remembered that the virus responsible
for croup is the parainfluenza virus. The toughest part was remembering that parainfluenza
virus (along with measles virus, mumps virus, and respiratory syncytial virus) belongs to the
paramyxovirus family. These viruses all have negative-strand RNA and an enveloped helical
nucleocapsid.

The papovaviruses (choice A) are DNA viruses with a naked icosahedral nucleocapsid. This family
includes the human papilloma viruses (which cause warts and are associated with penile,
laryngeal, and especially, cervical cancer), the BK virus that can affect immunosuppressed
patients, the JC virus associated with progressive multifocal leukoencephalopathy, and the
simian SV40 virus.

The parvoviruses (choice C) are small, single-stranded DNA viruses. Only one serotype (B19)
causes diseases in humans. It causes erythema infectiosum in children (characteristic "slapped
cheek" rash), aplastic crises in patients with hemolytic diseases, and hydrops fetalis or
stillbirth in anemic fetuses.

The picornaviruses (choice D) are positive single-stranded RNA viruses with a naked icosahedral
nucleocapsid. This family includes the polioviruses, echoviruses, coxsackieviruses, enterovirus
72 (HepA virus), and the rhinoviruses (common cold).

The poxviruses (choice E) are double-stranded DNA viruses. This family includes the viruses
responsible for smallpox and molluscum contagiosum.

A 24-year-old man presents with fever, rash, a mild headache, and a sore throat. He denies HIV risk
factors, although he is sexually active. On examination, his temperature is 100.8°F and his
pulse is 90/min. There is a diffuse, erythematous, maculopapular rash over most of his body.
Generalized adenopathy is appreciated, and photophobia is noted when funduscopic examination is
attempted. If this man is not treated, which of the following changes in his serologic status
will most likely occur?

A. The FTA-ABS titer would fall

B. The FTA-ABS titer would rise

C. The VDRL titer would fall

D. The VDRL titer would rise

E. Both FTA-ABS and VDRL would fall

F. Both FTA-ABS and VDRL would rise


137
Explanation:

The correct answer is C. This is a case of syphilis, which is diagnosed serologically, using
either treponemal or nontreponemal tests. The FTA-ABS (fluorescent treponemal antibody-absorbed
test) is the most widely used of the specific tests and depends on fluorescent labeling of the
organisms with anti-treponemal antibody. The specific tests tend to rise early in titer and
stay elevated throughout the lifetime of the host, or at least until well after drug therapy
has been completed. The VDRL (Venereal Disease Research Laboratory) is a nontreponemal test
that detects antibodies that cross-react with mammalian cardiolipid called reaginic antibodies.
This test will become positive after the specific test, and its titer will fall late in
infection, with or without drug therapy.

The FTA-ABS titer would fall (choice A) is incorrect because this titer will remain high
throughout the life of the host, falling off only very slowly after drug cure.

The FTA-ABS titer would rise (choice B) is incorrect because the specific antibody test will
reach a high positive titer and remain at that level in untreated late stage syphilis.

The VDRL titer would rise (choice D) is incorrect because reaginic antibody levels will fall in
late stage syphilis, with or without drug treatment.

Both FTA-ABS and VDRL would fall (choice E) is incorrect because specific antibody levels would
remain high, while reaginic antibodies would fall.

Both FTA-ABS and VDRL would rise (choice F) is incorrect because specific antibody levels would
remain high, while reaginic antibodies would fall.

A 22-year-old male military recruit complains of a headache and stiff neck. He is examined, blood is
drawn, and a lumbar puncture performed. The glucose in the CSF is 100 mg/dL and the serum
glucose is 120 mg/dL. The CSF shows 3 lymphocytes and 0 neutrophils/microliter. Which of the
following conclusions concerning the interpretation of these findings is most accurate?

A. The CSF glucose level suggests bacterial meningitis

B. The CSF glucose level suggests viral meningitis

C. The lymphocytes suggest bacterial meningitis

D. The lymphocytes suggest viral meningitis

E. There is no evidence for meningitis

Explanation:

The correct answer is E. Both the CSF glucose level and the small number of lymphocytes present
are within normal limits. It is normal for the CSF glucose to be less than serum glucose, often
about 2/3 of the serum value.

Bacterial meningitis (choices A and C) can profoundly lower CSF glucose levels. CSF from
bacterial meningitis cases also usually shows large numbers of neutrophils.

Viral meningitis (choices B and D) will not alter the CSF glucose level, but the CSF often
contains many lymphocytes.

Autopsy of a 23-year-old male victim of a motor vehicle accident reveals a small cluster of caseating
granulomas in the right lung just above the interlobar fissure and similar granulomas in the
hilar lymph nodes. Acid-fast staining demonstrates acid-fast bacilli within these lesions. No
other lesions were found in the remaining organs and systems. Which of the following is the most
accurate interpretation of these findings?

A. Cavitary tuberculosis
138

B. Ghon complex

C. Histoplasma infection

D. Miliary tuberculosis

E. Remote healed tuberculosis

Explanation:

The correct answer is B. The Ghon complex is the most frequent pathologic form of primary
pulmonary tuberculosis. Mycobacterium tuberculosis first localizes in the lung parenchyma, then
in the hilar lymph nodes. In both these locations, a granulomatous reaction takes place. These
lesions usually heal by fibrosis, leaving only small scars at the sites of remote tuberculous
infection. In some cases, owing to immunosuppression (e.g., AIDS, immunosuppressant treatment,
and lymphomas), reactivation of dormant bacilli in old lesions or additional re-exposure leads
to secondary tuberculosis, with progression of lesions. Sometimes, active lesions of the Ghon
complex are discovered by chance at autopsy. More frequently, scars due to remote healed
tuberculosis (choice E) are found postmortem and listed in autopsy reports as incidental
findings

Cavitary tuberculosis (choice A) and miliary tuberculosis (choice D) are expressions of


secondary infection, following reactivation of old, usually clinically silent, lesions. The
cavitary form is characterized by development of large areas of liquefactive necrosis that
empty into the airspaces, leading to cavities within the lung parenchyma. The miliary form is
due to lymphohematogenous dissemination and subsequent seeding of tubercle bacilli throughout
the body, with myriad small granulomas forming in the lungs, spleen, liver, bone marrow,
retina, and adrenals, for example.

Acid-fast bacilli suggest tuberculosis, rather than an infection with a fungus such as
Histoplasma(choice C).

A 53-year-old woman with diarrhea and lower abdominal pain of 3 days duration comes to her physician
after failing to relieve her symptoms with various home remedies. She denies any recent travel
and states that there is blood and pus in her stool. Fecal cultures yield several flagellated,
curved, oxidase-positive, gram-negative rods. The organism isolated is most likely

A. Campylobacter jejuni

B. Escherichia coli

C. Salmonella

D. Shigella

E. Vibrio cholera

Explanation:

The correct choice is A. Campylobacter is a motile, curved, oxidase-positive, gram-negative rod


with a polar flagella. The illness typically begins 1 to 7 days following ingestion of the
organism. The presentation is usually lower abdominal pain and diarrhea with blood and pus. The
illness is self-limited after 3 to 5 days and can last up to 2 weeks. The organisms grow
optimally at 42° C under microaerophilic conditions.

Escherichia coli(choice B), although a flagellated gram-negative organism, is not the correct
choice since it is not as common a cause of bloody diarrhea in this age group and is not
oxidase positive.

Salmonella (choice C) is incorrect because it is oxidase negative.


139

Shigella (choice D) is incorrect because it is oxidase negative.

Vibrio cholera(choice E) has many physical features in common with C. jejuni. However, V.
cholera is not enteroinvasive and does not produce bloody diarrhea.

A hospitalized patient develops dysuria and suprapubic pain and is treated with ciprofloxacin. What is
the mechanism of action of this antibiotic?

A. It inhibits dihydrofolate reductase

B. It inhibits DNA-dependent RNA polymerase

C. It inhibits protein synthesis by binding to the 30s ribosomal subunit

D. It inhibits protein synthesis by binding to the 50s ribosomal subunit

E. It inhibits topoisomerase II (DNA gyrase)

Explanation:

The correct answer is E. This is a straightforward question where the introductory clinical
details are really irrelevant. Ciprofloxacin and norfloxacin belong to a category of antibiotics
called the fluoroquinolones. They are bactericidal and work by inhibiting topoisomerase II (DNA
gyrase). They are effective against gram-negative rods and are the only oral agents effective
against Pseudomonas. Ciprofloxacin is effective for treating UTIs, gonorrhea, diarrheal
diseases, and soft tissue infections. It is also used to treat Pseudomonas infections in cystic
fibrosis.

The wrong answer choices provide us with the opportunity to discuss important mechanisms of
action for other antimicrobials:

Inhibition of dihydrofolate reductase (choice A) is the mechanism of action of trimethoprim.


Trimethoprim is typically used in combination with sulfonamides (trimethoprim-sulfamethoxazole).
Sulfonamides inhibit an earlier step in folate synthesis (dihydropteroate synthase), so the
combination with trimethoprim is an effective "one-two" punch. Trimethoprim-sulfa is used in the
treatment of Shigella, Salmonella, recurrent UTIs, and in Pneumocystis carinii pneumonia.

Inhibition of DNA-dependent RNA polymerase (choice B) is the mechanism of action of rifampin.


Rifampin is used (along with other drugs) in the treatment of tuberculosis. You should also
remember that rifampin can be used to treat individuals exposed to Meningococcus or H.
influenzae type B.

Inhibition of the 30s ribosomal subunit (choice C) is the mechanism of action of two important
classes of antibiotic&mdash;the tetracyclines (tetracycline, doxycycline, demeclocycline) and
the aminoglycosides (gentamicin, tobramycin, streptomycin, etc.). The tetracyclines inhibit the
attachment of the aminoacyl-tRNA to the ribosome while the aminoglycosides inhibit the formation
of the initiation complex.

Inhibition of the 50s ribosomal subunit (choice D) is the mechanism of action of the macrolides
(e.g., erythromycin), the lincosamides (e.g., lincomycin, clindamycin), and chloramphenicol.
Chloramphenicol inhibits the 50s peptidyl transferase, while erythromycin blocks translocation.

Cefuroxime is believed to exert its antibacterial effect by which of the following mechanisms?

A. Competitive inhibition of para-aminobenzoic acid (PABA)

B. Inhibition of bacterial cell wall synthesis

C. Inhibition of DNA-gyrase
140
D. Irreversible binding to the 30S subunit of bacterial ribosomes

E. Irreversible binding to the 50S subunit of bacterial ribosomes

Explanation:

The correct answer is B. Cephalosporins, such as cefuroxime, are believed to exert their
antibacterial effect by binding to one or more of the penicillin-binding proteins located on the
cell walls of susceptible organisms. This action results in the inhibition of the third, and
final, stage of bacterial cell wall synthesis. This is also the mechanism of action of the
penicillins. Cefuroxime is a second generation cephalosporin used to treat infections in the
lower respiratory and urinary tracts, as well as otitis media. It is also efficacious in the
treatment of gonorrhea and is used for perioperative prophylaxis in various surgical procedures,
such as coronary artery bypass grafting.

Sulfonamide antibiotics, such as sulfamethoxazole-trimethoprim, exert their antibacterial effect


through the competitive inhibition of para-aminobenzoic acid (PABA) (choice A), thereby
inhibiting folic acid biosynthesis required for bacterial growth.

Quinolone antibiotics, such as ciprofloxacin, inhibit DNA-gyrase (choice C), which is an enzyme
necessary for bacterial DNA replication and repair.

Aminoglycosides, such as gentamicin, irreversibly bind to the 30S subunit of bacterial ribosomes
(choice D), inhibiting bacterial protein synthesis.

Lincosamides, such as clindamycin, irreversibly bind to the 50S subunit of bacterial ribosomes
(choice E), suppressing bacterial protein synthesis. Note that macrolides, such as erythromycin,
reversibly bind to the 50S subunit of bacterial ribosome.

A 19-year-old college student presents to the student health clinic complaining of weakness, malaise,
and a chronic cough. He has a fever of 100 degrees F and a dry cough; no sputum can be obtained
for laboratory analysis, so a bronchial lavage is performed and the washings are submitted to
the laboratory. The laboratory reports that the organism is "slow-growing." Serodiagnosis
reveals Strep MG agglutinins in the patient's serum. Which of the following organisms is the
most likely cause of this student's illness?

A. Klebsiella pneumoniae

B. Mycoplasma pneumoniae

C. Parainfluenza virus

D. Respiratory syncytial virus

E. Streptococcus pneumoniae

Explanation:

The correct answer is B. The patient has primary atypical pneumonia caused by Mycoplasma
pneumoniae. This organism is fastidious and difficult to culture in the laboratory, however
serodiagnosis can be most helpful. Patients typically produce one or two heterophile antibodies
during the course of the infection; one agglutinates human O+ RBCs in the cold (the cold
hemagglutinin) while the other causes the agglutination of a strain of Streptococcus salivarius
termed strain MG (the Strep MG agglutinins).

Klebsiella pneumoniae(choice A) is readily cultured on routine laboratory media and


characteristically produces pneumonia with blood clots in the sputum (red currant jelly
sputum), which may be indicative of pulmonary abscess development.

Parainfluenza viruses (choice C) cause croup, which is characterized by a dry, "barking" cough.
It is more of a tracheitis, bronchitis, and bronchiolitis than a pneumonitis. No heterophile
141
antibodies are produced in these patients.

Respiratory syncytial virus (choice D) causes an atypical pneumonitis in infants. It is usually


diagnosed by the observation of syncytial masses in respiratory secretions. Cold hemagglutinins
and Strep MG agglutinins are absent.

Streptococcus pneumoniae (choice E) is the number one cause of pneumonia in adults. It also
causes septicemia and meningitis in the elderly. The patient has a classical acute pneumonia
with a productive cough, high fever with chills, leukocytosis, tachycardia, rapid respirations
and other signs of serious respiratory disease. A vaccine, composed of the capsular
carbohydrate of 23 serotypes of this organism, is routinely given to individuals over the age
of 60, as well as to individuals with splenic abnormalities (e.g., sickle cell disease) who are
at increased risk for pneumococcal sepsis.

An 8-month-old baby presents to the emergency room with a 1-day history of poor feeding and generalized
weakness. The mother states that she often feeds the baby honey to pacify her. The toxin
responsible for this presentation works by which of the following mechanisms?

A. It blocks the release of acetylcholine from the nerve terminal

B. It blocks the release of inhibitory neurotransmitters such as glycine and GABA

C. It has a subunit that inactivates an elongation factor by ADP-ribosylation

D. It is a lecithinase

E. It stimulates guanylate cyclase

Explanation:

The correct answer is A. The clinical history suggests infant botulism. The clue here is that
the mother feeds the baby honey. Clostridium botulinum (a common honey contaminant) was
ingested and produced toxin in the infant. The toxin, which blocks the release of acetylcholine
from nerve terminals, is responsible for the floppiness. Acetylcholine is the neurotransmitter
at the neuromuscular junction and impairment of this can lead to muscle weakness, failure to
thrive, and in more serious cases, respiratory impairment.

Glycine and GABA (choice B) release is inhibited by C. tetani. This leads to the muscular
spasms of tetanus.

ADP-ribosylation of an elongation factor (choice C) is a mechanism of action of diphtheria and


Pseudomonas exotoxins.

Alpha toxin from C. perfringens is a lecithinase (choice D) responsible for the development of
gas gangrene.

Guanylate cyclase (choice E) is stimulated by a heat-stable toxin produced by E. coli.

A 7-year-old boy presents to the pediatrician because his mother noticed a "smoky" color to his urine.
Upon questioning the mother, it is revealed that the child suffered a sore throat several weeks
ago that was left untreated. Physical examination reveals hypertension and mild generalized
edema. Urinalysis is significant for red blood cell casts. Which of the following accurately
describes the microorganism responsible for this child's illness?

A. It causes alpha-hemolysis on blood agar

B. It is catalase positive

C. It is coagulase positive

D. It is sensitive to bacitracin
142

E. It is sensitive to optochin

Explanation:

The correct answer is D. This is one of the "higher order" questions the USMLE often favors. In
this case, you need to figure out what disease the child has, what organism causes the disease,
and which of the listed features is true of the microorganism. The disease in question is
poststreptococcal glomerulonephritis, as evidenced by the smoky urine, hypertension, edema, and
red blood cell casts in the urine sediment. The history of the prior sore throat is a tip-off
that this is a nonsuppurative sequela of an infection due to Streptococcus pyogenes (group A
&beta;-hemolytic Streptococci). You must remember the features of S. pyogenes. S. pyogenes can
be differentiated from Streptococcus pneumoniae and viridans Streptococci by its hemolytic
pattern; it is beta-hemolytic, while the others are alpha-hemolytic (choice A). It can be
distinguished from the other beta-hemolytic Streptococci by its sensitivity to the antibiotic
bacitracin. Other important things to remember about S. pyogenes are its many virulence
factors, including M protein, antiphagocytic capsule, hyaluronidase, streptolysins O and S, and
erythrogenic toxins.

Choice B is incorrect because Streptococci are catalase negative, in contrast to Staphylococci,


which are catalase positive.

Choice C is incorrect because S. pyogenes is coagulase negative. (In fact, a coagulase test
should NOT be done on a catalase-negative organism.) The coagulase test is an important means
of differentiating Staphylococcus aureus, which are coagulase positive, from all other species
of Staphylococcus, which are coagulase negative.

Choice E is incorrect because optochin is used to differentiate the viridans Streptococci


(resistant) from S. pneumoniae (sensitive).

An AIDS patient with clinical pneumonia has a bronchoalveolar lavage that demonstrates small, "hat-
shaped" structures in alveoli that are about the size of an erythrocyte and stain with silver
stains. The microorganism involved is most likely which of the following?

A. Aspergillus fumigatus

B. Blastomyces dermatitidis

C. Mycobacterium avium

D. Mycobacterium tuberculosis

E. Pneumocystis carinii

Explanation:

The correct answer is E. This is the classic appearance of Pneumocystis cysts. Pneumocystis is
a common (and dangerous) cause of pneumonia in AIDS patients. The diagnosis is now frequently
made by bronchoalveolar lavage, which is much more effective at demonstrating the organism than
is either sputum or blood culture.

Aspergillus fumigatus(choice A) is a typical fungus, and hyphae would probably have been seen
in the lavage material.

Blastomycosis (choice B) is caused by a dimorphic fungus that grows in mammalian tissues as a


round, multinucleate, budding cell 8-15 micrometers in diameter.

Mycobacteria (choices C and D) are small, acid-fast rods.

Which of the following viruses is capable of replication in enucleated cells?


143
A. Adenovirus

B. Cytomegalovirus

C. Influenza virus

D. JC virus

E. Poliovirus

Explanation:

The correct answer is E. Most RNA viruses (eg, poliovirus) replicate in the cytoplasm and
therefore can replicate in enucleated cells. Poliovirus belongs to the family Picornaviridae.
These viruses are nonenveloped and have an icosahedral nucleocapsid that contains positive-
sense RNA.

The exception to the rule regarding RNA viruses is the family Orthomyxoviridae, the influenza
viruses (choice C). Orthomyxoviruses undergo transcription and RNA replication in the nucleus
of the host cell because they need to cannibalize the capped 5' termini of cellular RNAs for
use as primers for viral mRNA transcription.

For most DNA viruses, transcription and DNA replication occur in the nucleus of the host cell.
The exception to this observation is the family Poxviridae, which carries out its replication
in the cytoplasm. Poxviridae includes variola virus, vaccinia virus, molluscum contagiosum, and
orf virus.

Adenoviruses (choice A) are nonenveloped and have an icosahedral nucleocapsid that contains a
double-stranded linear DNA genome.

Cytomegalovirus (choice B) is a member of family Herpesviridae. It is an enveloped virus with


an icosahedral nucleocapsid that contains a double-stranded linear DNA genome.

JC virus (choice D) belongs to family Papovaviridae. It is nonenveloped and has an icosahedral


nucleocapsid that contains a double-stranded circular DNA genome.

A 47-year-old man with a history of sickle cell disease has had numerous hospitalizations requiring the
placement of intravenous lines. The patient has poor peripheral venous access, and a catheter is
placed in the right subclavian vein. The patient subsequently develops right arm discomfort and
swelling and a temperature of 40.1 degrees C with chills. Multiple blood cultures are taken, and
gram-positive cocci are isolated. The organism is catalase positive and grows on mannitol salt
agar, but does not turn the agar yellow; the colonies are gamma-hemolytic on a sheep blood agar
plate. Which of the following organisms is the most likely cause of this patient's symptoms?

A. Enterococcus faecalis

B. Staphylococcus aureus

C. Staphylococcus epidermidis

D. Streptococcus agalactiae

E. Streptococcus pyogenes

Explanation:

The correct answer is C. The patient has developed bacteremia; the description of the causative
agent is consistent with a staphylococcal organism (catalase positive, gram-positive cocci that
grow on mannitol salt agar). The organism is most likely S. epidermidis as it was not able to
ferment mannitol, and was not hemolytic. Both of those characteristics tend to rule out S.
aureus(choice B). Two other tests that are commonly used are coagulase production and excretion
144
of DNAse from colonies. S. aureus is positive in both tests, S. epidermidis is negative.

Enterococcus faecalis(choice A) might grow on the mannitol salt agar as it is relatively


haloduric but these organisms are catalase negative. The enterococci are extremely variable in
hemolytic ability so this characteristic is not useful in species identification.

Both streptococcal organisms (choices D and E) are catalase negative and beta-hemolytic on
sheep blood agar plates. Also, neither would grow on the mannitol salt agar. S. pyogenes is
sensitive to growth inhibition by bacitracin while S. agalactiae (group B streptococci) is not.

Microbiologic studies on a gram-positive coccus isolated from a skin abscess demonstrate a positive
catalase test and a positive coagulase test. This is most consistent with which of the following
organisms?

A. S. aureus

B. S. epidermidis

C. S. pneumoniae

D. S. pyogenes

E. S. saprophyticus

Explanation:

The correct answer is A. All of the organisms listed are gram-positive cocci. The positive
catalase test excludes the Streptococci, S. pyogenes(choice D), and S. pneumoniae(choice C). Of
the Staphylococci, only S. aureus has a positive coagulase test. S. epidermidis(choice B) and
S. saprophyticus(choice E) are coagulase-negative Staphylococci.

You might also like